exam 4 except chapter 52

अब Quizwiz के साथ अपने होमवर्क और परीक्षाओं को एस करें!

A patient's vital signs are pulse 87, respirations 24, and BP of 128/64 mm Hg and cardiac output is 4.7 L/min. The patient's stroke volume is _____ mL. (Round to the nearest whole number.)

ANS: 54 Stroke volume = cardiac output/heart rate

Which patient statement indicates a need for further teaching about extended-release zolpidem (Ambien CR)? a. "I will take the medication an hour before bedtime." b. "I should take the medication on an empty stomach." c. "I should not take this medication unless I can sleep for at least 6 hours." d. "I will schedule activities that require mental alertness for later in the day."

ANS: A Benzodiazepine receptor agonists such as zolpidem work quickly and should be taken immediately before bedtime. The other patient statements are correct

A patient with acute osteomyelitis of the left femur is hospitalized for regional antibiotic irrigation. Which intervention will be included in the initial plan of care? a. Immobilization of the left leg b. Positioning the left leg in flexion c. Assisted weight-bearing ambulation d. Quadriceps-setting exercise repetitions

ANS: A Immobilization of the affected leg helps decrease pain and reduce the risk for pathologic fractures. Weight-bearing exercise increases the risk for pathologic fractures. Flexion of the affected limb is avoided to prevent contractures

A 20-year-old woman who is being seen in the family medicine clinic for an annual physical exam reports being sexually active. The nurse will plan to teach the patient about a. testing for Chlamydia infection. b. immunization for herpes simplex. c. infertility associated with the human papillomavirus (HPV). d. the relationship between the herpes virus and cervical cancer.

ANS: A Testing for Chlamydia is recommended for all sexually active females under age 25 by the Centers for Disease Control and Prevention. HPV infection does not cause infertility. There is no vaccine available for herpes simplex, and herpes simplex infection does not cause cervical cancer

The nurse is caring for a patient with colon cancer who is scheduled for external radiation therapy to the abdomen. Which information obtained by the nurse would indicate a need for patient teaching? a. The patient swims a mile 3 days a week. b. The patient snacks frequently during the day. c. The patient showers everyday with a mild soap. d. The patient has a history of dental caries with amalgam fillings.

ANS: A The patient is instructed to avoid swimming in salt water or chlorinated pools during the treatment period. The patient does not need to change habits of eating frequently or showering with a mild soap. A history of dental caries will not impact the patient who is scheduled for abdominal radiation.

Which nursing action is correct when performing the straight-leg raising test for an ambulatory patient with back pain? a. Raise the patient's legs to a 60-degree angle from the bed. b. Place the patient initially in the prone position on the exam table. c. Have the patient dangle both legs over the edge of the exam table. d. Instruct the patient to elevate the legs and tense the abdominal muscles.

ANS: A When performing the straight leg-raising test, the patient is in the supine position and the nurse passively lifts the patient's legs to a 60-degree angle. The other actions would not be correct for this test

A 39-year-old patient with a history of IV drug use is seen at a community clinic. The patient reports difficulty walking, stating "I don't know where my feet are." Diagnostic screening reveals positive Venereal Disease Research Laboratory (VDRL) and fluorescent treponemal antibody absorption (FTA-Abs) tests. Based on the patient history, what will the nurse assess (select all that apply)? a. Heart sounds b. Genitalia for lesions c. Joints for swelling and inflammation d. Mental state for judgment and orientation e. Skin and mucous membranes for gummas

ANS: A, D, E The patient's clinical manifestations and laboratory tests are consistent with tertiary syphilis. Valvular insufficiency, gummas, and changes in mentation are other clinical manifestations of this stage

After change-of-shift report in the progressive care unit, who should the nurse care for first? a. Patient who had an inferior myocardial infarction 2 days ago and has crackles in the lung bases b. Patient with suspected urosepsis who has new orders for urine and blood cultures and antibiotics c. Patient who had a T5 spinal cord injury 1 week ago and currently has a heart rate of 54 beats/minute d. Patient admitted with anaphylaxis 3 hours ago who now has clear lung sounds and a blood pressure of 108/58 mm Hg

ANS: B Antibiotics should be administered within the first hour for patients who have sepsis or suspected sepsis in order to prevent progression to systemic inflammatory response syndrome (SIRS) and septic shock. The data on the other patients indicate that they are more stable. Crackles heard only at the lung bases do not require immediate intervention in a patient who has had a myocardial infarction. Mild bradycardia does not usually require atropine in patients who have a spinal cord injury. The findings for the patient admitted with anaphylaxis indicate resolution of bronchospasm and hypotension

When caring for a 58-year-old patient with persistent menorrhagia, the nurse will plan to monitor the a. estrogen level. b. complete blood count (CBC). c. gonadotropin-releasing hormone (GNRH) level. d. serial human chorionic gonadotropin (hCG) results.

ANS: B Because anemia is a likely complication of menorrhagia, the nurse will need to check the CBC. Estrogen and GNRH levels are checked for patients with other problems, such as infertility. Serial hCG levels are monitored in patients who may be pregnant, which is not likely for this patient

The nurse will plan to teach a 34-year-old patient diagnosed with stage 0 cervical cancer about a. radiation. b. conization. c. chemotherapy. d. radical hysterectomy.

ANS: B Because the carcinoma is in situ, conization can be used for treatment. Radical hysterectomy, chemotherapy, or radiation will not be needed

The nurse cares for a critically ill patient in the intensive care unit (ICU). Which intervention should the nurse include in the plan of care to improve this patient's sleep quality? a. Ask all visitors to leave the ICU for the night. b. Lower the level of lights from 8:00 PM until 7:00 AM. c. Avoid the use of opioids for pain relief during the evening hours. d. Schedule assessments to allow at least 4 hours of uninterrupted sleep.

ANS: B Lowering the level of light will help mimic normal day/night patterns and maximize the opportunity for sleep. Although frequent assessments and opioid use can disturb sleep patterns, these actions are necessary for the care of critically ill patients. For some patients, having a family member or friend at the bedside may decrease anxiety and improve sleep

The nurse is interviewing a patient with contact dermatitis. Which finding indicates a need for patient teaching? a. The patient applies corticosteroid cream to pruritic areas. b. The patient uses Neosporin ointment on minor cuts or abrasions. c. The patient adds oilated oatmeal (Aveeno) to the bath water every day. d. The patient takes diphenhydramine (Benadryl) at night if itching occurs.

ANS: B Neosporin can cause contact dermatitis. The other medications are being used appropriately by the patient

The following medications are prescribed by the health care provider for a middle-aged patient who uses long-acting morphine (MS Contin) for chronic back pain, but still has ongoing pain. Which medication should the nurse question? a. Morphine (Roxanol) b. Pentazocine (Talwin) c. Celecoxib (Celebrex) d. Dexamethasone (Decadron)

ANS: B Opioid agonist-antagonists can precipitate withdrawal if used in a patient who is physically dependent on mu agonist drugs such as morphine. The other medications are appropriate for the patient

The nurse instructs a patient who has osteosarcoma of the tibia about a scheduled above-the-knee amputation. Which statement by a patient indicates that additional patient teaching is needed? a. "I will need to participate in physical therapy after surgery." b. "I did not have this bone cancer until my leg broke a week ago." c. "I wish that I did not have to have chemotherapy after this surgery." d. "I can use the patient-controlled analgesia (PCA) to control postoperative pain."

ANS: B Osteogenic sarcoma may be diagnosed following a fracture, but it is not caused by the injury. The other patient statements indicate that patient teaching has been effective

The nurse performing a focused examination to determine possible causes of infertility will assess for a. hydrocele. b. varicocele. c. epididymitis. d. paraphimosis.

ANS: B Persistent varicoceles are commonly associated with infertility. Hydrocele, epididymitis, and paraphimosis are not risk factors for infertility

Which hemodynamic parameter is most appropriate for the nurse to monitor to determine the effectiveness of medications given to a patient to reduce left ventricular afterload? a. Mean arterial pressure (MAP) b. Systemic vascular resistance (SVR) c. Pulmonary vascular resistance (PVR) d. Pulmonary artery wedge pressure (PAWP)

ANS: B Systemic vascular resistance reflects the resistance to ventricular ejection, or afterload. The other parameters will be monitored, but do not reflect afterload as directly

When assessing for Tinel's sign in a patient with possible right-sided carpal tunnel syndrome, the nurse will ask the patient about a. weakness in the right little finger. b. tingling in the right thumb and fingers. c. burning in the right elbow and forearm. d. tremor when gripping with the right hand.

ANS: B Testing for Tinel's sign will cause tingling in the thumb and first three fingers of the affected hand in patients who have carpal tunnel syndrome. The median nerve does not innervate the right little finger or elbow and forearm. Tremor is not associated with carpal tunnel syndrome

A patient with a complex pelvic fracture from a motor vehicle crash is on bed rest. Which nursing assessment finding is important to report to the health care provider? a. The patient states that the pelvis feels unstable. b. Abdomen is distended and bowel sounds are absent. c. There are ecchymoses across the abdomen and hips. d. The patient complains of pelvic pain with palpation.

ANS: B The abdominal distention and absent bowel sounds may be due to complications of pelvic fractures such as paralytic ileus or hemorrhage or trauma to the bladder, urethra, or colon. Pelvic instability, abdominal pain with palpation, and abdominal bruising would be expected with this type of injury.

A patient who is anxious and has difficulty breathing seeks treatment after being stung by a wasp. What is the nurse's priority action? a. Have the patient lie down. b. Assess the patient's airway. c. Administer high-flow oxygen. d. Remove the stinger from the site.

ANS: B The initial action with any patient with difficulty breathing is to assess and maintain the airway. The other actions also are part of the emergency management protocol for anaphylaxis, but the priority is airway maintenance

The nurse is caring for a patient who is intubated and receiving positive pressure ventilation to treat acute respiratory distress syndrome (ARDS). Which finding is most important to report to the health care provider? a. Blood urea nitrogen (BUN) level 32 mg/dL b. Red-brown drainage from orogastric tube c. Scattered coarse crackles heard throughout lungs d. Arterial blood gases: pH 7.31, PaCO2 50, PaO2 68

ANS: B The nasogastric drainage indicates possible gastrointestinal bleeding and/or stress ulcer, and should be reported. The pH and PaCO2 are slightly abnormal, but current guidelines advocating for permissive hypercapnia indicate that these would not indicate an immediate need for a change in therapy. The BUN is slightly elevated but does not indicate an immediate need for action. Adventitious breath sounds are commonly heard in patients with ARDS

A patient undergoes a left above-the-knee amputation with an immediate prosthetic fitting. When the patient arrives on the orthopedic unit after surgery, the nurse should a. place the patient in a prone position. b. check the surgical site for hemorrhage. c. remove the prosthesis and wrap the site. d. keep the residual leg elevated on a pillow.

ANS: B The nurse should monitor for hemorrhage after the surgery. The prosthesis will not be removed. To avoid flexion contracture of the hip, the leg will not be elevated on a pillow. The patient is placed in a prone position after amputation to prevent hip flexion, but this would not be done during the immediate postoperative period

Before assisting a patient with ambulation 2 days after a total hip replacement, which action is most important for the nurse to take? a. Observe the status of the incisional drain device. b. Administer the ordered oral opioid pain medication. c. Instruct the patient about the benefits of ambulation. d. Change the hip dressing and document the wound appearance.

ANS: B The patient should be adequately medicated for pain before any attempt to ambulate. Instructions about the benefits of ambulation may increase the patient's willingness to ambulate, but decreasing pain with ambulation is more important. The presence of an incisional drain or timing of dressing change will not affect ambulation

The nurse will plan to teach the patient scheduled for photovaporization of the prostate (PVP) a. that urine will appear bloody for several days. b. how to care for an indwelling urinary catheter. c. that symptom improvement takes 2 to 3 weeks. d. about complications associated with urethral stenting.

ANS: B The patient will have an indwelling catheter for 24 to 48 hours and will need teaching about catheter care. There is minimal bleeding with this procedure. Symptom improvement is almost immediate after PVP. Stent placement is not included in the procedure

Following laminectomy with a spinal fusion to treat a herniated disc, a patient reports numbness and tingling of the right lower leg. The first action that the nurse should take is to a. report the patient's complaint to the surgeon. b. check the chart for preoperative assessment data. c. check the vital signs for indications of hemorrhage. d. turn the patient to the side to relieve pressure on the right leg.

ANS: B The postoperative movement and sensation of the extremities should be unchanged (or improved) from the preoperative assessment. If the numbness and tingling are new, this information should be immediately reported to the surgeon. Numbness and tingling are not symptoms associated with hemorrhage at the site. Turning the patient will not relieve the numbness

Which assessment finding in a woman who recently started taking hormone therapy (HT) is most important for the nurse to report to the health care provider? a. Breast tenderness b. Left calf swelling c. Weight gain of 3 lb d. Intermittent spotting

ANS: B Unilateral calf swelling may indicate deep vein thrombosis caused by the changes in coagulation associated with HT and would indicate that the HT should be discontinued. Breast tenderness, weight gain, and intermittent spotting are common side effects of HT and do not indicate a need for a change in therapy

Which actions should the nurse initiate to reduce the risk for ventilator-associated pneumonia (VAP) (select all that apply)? a. Obtain arterial blood gases daily. b. Provide a "sedation holiday" daily. c. Elevate the head of the bed to at least 30°. d. Give prescribed pantoprazole (Protonix). e. Provide oral care with chlorhexidine (0.12%) solution daily.

ANS: B, C, D, E All of these interventions are part of the ventilator bundle that is recommended to prevent VAP. Arterial blood gases may be done daily but are not always necessary and do not help prevent VAP

A patient with a positive rapid antibody test result for human immunodeficiency virus (HIV) is anxious and does not appear to hear what the nurse is saying. What action by the nurse is most important at this time? a. Teach the patient about the medications available for treatment. b. Inform the patient how to protect sexual and needle-sharing partners. c. Remind the patient about the need to return for retesting to verify the results. d. Ask the patient to notify individuals who have had risky contact with the patient.

ANS: C After an initial positive antibody test, the next step is retesting to confirm the results. A patient who is anxious is not likely to be able to take in new information or be willing to disclose information about HIV status of other individuals

The nurse taking a focused health history for a patient with possible testicular cancer will ask the patient about a history of a. testicular torsion. b. testicular trauma. c. undescended testicles. d. sexually transmitted infection (STI).

ANS: C Cryptorchidism is a risk factor for testicular cancer if it is not corrected before puberty. STI, testicular torsion, and testicular trauma are risk factors for other testicular conditions but not for testicular cancer

Which discharge instruction will the emergency department nurse include for a patient with a sprained ankle? a. Keep the ankle loosely wrapped with gauze. b. Apply a heating pad to reduce muscle spasms. c. Use pillows to elevate the ankle above the heart. d. Gently move the ankle through the range of motion.

ANS: C Elevation of the leg will reduce the amount of swelling and pain. Compression bandages are used to decrease swelling. For the first 24 to 48 hours, cold packs are used to reduce swelling. The ankle should be rested and kept immobile to prevent further swelling or injury

A 19-year-old patient with massive trauma and possible spinal cord injury is admitted to the emergency department (ED). Which assessment finding by the nurse will help confirm a diagnosis of neurogenic shock? a. Inspiratory crackles. b. Cool, clammy extremities. c. Apical heart rate 45 beats/min. d. Temperature 101.2° F (38.4° C).

ANS: C Neurogenic shock is characterized by hypotension and bradycardia. The other findings would be more consistent with other types of shock

When obtaining the pertinent health history for a man who is being evaluated for infertility, which question is most important for the nurse to ask? a. "Are you circumcised?" b. "Have you had surgery for phimosis?" c. "Do you use medications to improve muscle mass?" d. "Is there a history of prostate cancer in your family?"

ANS: C Testosterone or testosterone-like medications may adversely affect sperm count. The other information will be obtained in the health history but does not affect the patient's fertility.

A patient who has been involved in a motor vehicle crash arrives in the emergency department (ED) with cool, clammy skin; tachycardia; and hypotension. Which intervention ordered by the health care provider should the nurse implement first? a. Insert two large-bore IV catheters. b. Initiate continuous electrocardiogram (ECG) monitoring. c. Provide oxygen at 100% per non-rebreather mask. d. Draw blood to type and crossmatch for transfusions.

ANS: C The first priority in the initial management of shock is maintenance of the airway and ventilation. ECG monitoring, insertion of IV catheters, and obtaining blood for transfusions should also be rapidly accomplished but only after actions to maximize oxygen delivery have been implemented.

After the nurse has taught a patient with a newly diagnosed sexually transmitted infection about expedited partner therapy, which patient statement indicates that the teaching has been effective? a. "I will tell my partner that it is important to be examined at the clinic." b. "I will have my partner take the antibiotics if any STI symptoms occur." c. "I will make sure that my partner takes all of the prescribed medication." d. "I will have my partner use a condom until I have finished the antibiotics."

ANS: C With expedited partner therapy, the patient is given a prescription or medications for the partner. The partner does not need to be evaluated by the health care provider, but is presumed to be infected and should be treated concurrently with the patient. Use of a condom will not treat the presumed STI in the partner

When assessing a patient with chronic obstructive pulmonary disease (COPD), the nurse finds a new onset of agitation and confusion. Which action should the nurse take first? a. Notify the health care provider. b. Check pupils for reaction to light. c. Attempt to calm and reorient the patient. d. Assess oxygenation using pulse oximetry.

ANS: D Because agitation and confusion are frequently the initial indicators of hypoxemia, the nurse's initial action should be to assess oxygen saturation. The other actions are also appropriate, but assessment of oxygenation takes priority over other assessments and notification of the health care provider

A woman calls the clinic because she is having an unusually heavy menstrual flow. She tells the nurse that she has saturated three tampons in the past 2 hours. The nurse estimates that the amount of blood loss over the past 2 hours is _____ mL. a. 20 to 30 b. 30 to 40 c. 40 to 60 d. 60 to 90

ANS: D The average tampon absorbs 20 to 30 mL.

A 32-year-old patient has oral contraceptives prescribed for endometriosis. The nurse will teach the patient to a. expect to experience side effects such as facial hair. b. take the medication every day for the next 9 months. c. take calcium supplements to prevent developing osteoporosis during therapy. d. use a second method of contraception to ensure that she will not become pregnant.

When oral contraceptives are prescribed to treat endometriosis, the patient should take the medications continuously for 9 months. Facial hair is a side effect of synthetic androgens. The patient does not need to use additional contraceptive methods. The hormones in oral contraceptives will protect against osteoporosis

Which information should the nurse include when teaching a patient who has just received a prescription for ciprofloxacin (Cipro) to treat a urinary tract infection? a. Use a sunscreen with a high SPF when exposed to the sun. b. Sun exposure may decrease the effectiveness of the medication. c. Photosensitivity may result in an artificial-looking tan appearance. d. Wear sunglasses to avoid eye damage while taking this medication.

The patient should stay out of the sun. If that is not possible, teach them to wear sunscreen when taking medications that can cause photosensitivity. The other statements are not accurate

The health care provider orders the following interventions for a 67-kg patient who has septic shock with a BP of 70/42 mm Hg and oxygen saturation of 90% on room air. In which order will the nurse implement the actions? (Put a comma and a space between each answer choice [A, B, C, D, E].) a. Obtain blood and urine cultures. b. Give vancomycin (Vancocin) 1 g IV. c. Start norepinephrine (Levophed) 0.5 mcg/min. d. Infuse normal saline 2000 mL over 30 minutes. e. Titrate oxygen administration to keep O2 saturation >95%.

ANS: E, D, C, A, B The initial action for this hypotensive and hypoxemic patient should be to improve the oxygen saturation, followed by infusion of IV fluids and vasopressors to improve perfusion. Cultures should be obtained before administration of antibiotics

A patient undergoing external radiation has developed a dry desquamation of the skin in the treatment area. The nurse teaches the patient about management of the skin reaction. Which statement, if made by the patient, indicates the teaching was effective? a. "I can buy some aloe vera gel to use on the area." b. "I will expose the treatment area to a sun lamp daily." c. "I can use ice packs to relieve itching in the treatment area." d. "I will scrub the area with warm water to remove the scales."

ANS: A Aloe vera gel and cream may be used on the radiated skin area. Ice and sunlamps may injure the skin. Treatment areas should be cleaned gently to avoid further injury

When performing discharge teaching for a patient after a vasectomy, the nurse instructs the patient that he a. should continue to use other methods of birth control for 6 weeks. b. should not have sexual intercourse until his 6-week follow-up visit. c. may have temporary erectile dysfunction (ED) because of swelling. d. will notice a decrease in the appearance and volume of his ejaculate.

ANS: A Because it takes about 6 weeks to evacuate sperm that are distal to the vasectomy site, the patient should use contraception for 6 weeks. ED that occurs after vasectomy is psychologic in origin and not related to postoperative swelling. The patient does not need to abstain from intercourse. The appearance and volume of the ejaculate are not changed because sperm are a minor component of the ejaculate

A 72-year-old patient with kyphosis is scheduled for dual-energy x-ray absorptiometry (DXA) testing. The nurse will plan to a. explain the procedure. b. start an IV line for contrast medium injection. c. give an oral sedative 60 to 90 minutes before the procedure. d. screen the patient for allergies to shellfish or iodine products.

ANS: A DXA testing is painless and noninvasive. No IV access is necessary. Contrast medium is not used. Because the procedure is painless, no antianxiety medications are required

A patient is admitted to the emergency department (ED) for shock of unknown etiology. The first action by the nurse should be to a. administer oxygen. b. obtain a 12-lead electrocardiogram (ECG). c. obtain the blood pressure. d. check the level of consciousness.

ANS: A The initial actions of the nurse are focused on the ABCs—airway, breathing, and circulation—and administration of oxygen should be done first. The other actions should be accomplished as rapidly as possible after oxygen administration

A patient with acute respiratory distress syndrome (ARDS) is placed in the prone position. When prone positioning is used, which information obtained by the nurse indicates that the positioning is effective? a. The patient's PaO2 is 89 mm Hg, and the SaO2 is 91%. b. Endotracheal suctioning results in clear mucous return. c. Sputum and blood cultures show no growth after 48 hours. d. The skin on the patient's back is intact and without redness.

ANS: A The purpose of prone positioning is to improve the patient's oxygenation as indicated by the PaO2 and SaO2. The other information will be collected but does not indicate whether prone positioning has been effective

When caring for a patient who is pancytopenic, which action by unlicensed assistive personnel (UAP) indicates a need for the nurse to intervene? a. The UAP assists the patient to use dental floss after eating. b. The UAP adds baking soda to the patient's saline oral rinses. c. The UAP puts fluoride toothpaste on the patient's toothbrush. d. The UAP has the patient rinse after meals with a saline solution.

ANS: A Use of dental floss is avoided in patients with pancytopenia because of the risk for infection and bleeding. The other actions are appropriate for oral care of a pancytopenic patient

After scheduling a patient with a possible ovarian cyst for ultrasound, the nurse will teach the patient that she should a. expect to receive IV contrast during the procedure. b. drink several glasses of fluids before the procedure. c. experience mild abdominal cramps after the procedure. d. discontinue taking aspirin for 7 days before the procedure.

ANS: B A full bladder is needed for many ultrasound procedures, so the nurse will have the patient drink fluids before arriving for the ultrasound. The other instructions are not accurate for this procedure

The nurse is caring for a patient who has septic shock. Which assessment finding is most important for the nurse to report to the health care provider? a. Blood pressure (BP) 92/56 mm Hg b. Skin cool and clammy c. Oxygen saturation 92% d. Heart rate 118 beats/minute

ANS: B Because patients in the early stage of septic shock have warm and dry skin, the patient's cool and clammy skin indicates that shock is progressing. The other information will also be reported, but does not indicate deterioration of the patient's status

The nurse notes crackling sounds and a grating sensation with palpation of an older patient's elbow. How will this finding be documented? a. Torticollis b. Crepitation c. Subluxation d. Epicondylitis

ANS: B Crackling sounds and a grating sensation that accompany movement are described as crepitus or crepitation. Torticollis is a twisting of the neck to one side, subluxation is a partial dislocation of the joint, and epicondylitis is an inflammation of the elbow that causes a dull ache that increases with movement

A patient is being discharged 4 days after hip replacement surgery using the posterior approach. Which patient action requires immediate intervention by the nurse? a. The patient uses crutches with a swing-to gait. b. The patient leans over to pull shoes and socks on. c. The patient sits straight up on the edge of the bed. d. The patient bends over the sink while brushing teeth.

ANS: B Leaning over would flex the hip at greater than 90 degrees and predispose the patient to hip dislocation. The other patient actions are appropriate and do not require any immediate action by the nurse to protect the patient

A patient who uses injectable illegal drugs asks the nurse about preventing acquired immunodeficiency syndrome (AIDS). Which response by the nurse is best? a. "Avoid sexual intercourse when using injectable drugs." b. "It is important to participate in a needle-exchange program." c. "You should ask those who share equipment to be tested for HIV." d. "I recommend cleaning drug injection equipment before each use."

ANS: B Participation in needle-exchange programs has been shown to decrease and control the rate of HIV infection. Cleaning drug equipment before use also reduces risk, but it might not be consistently practiced. HIV antibodies do not appear for several weeks to months after exposure, so testing drug users would not be very effective in reducing risk for HIV exposure. It is difficult to make appropriate decisions about sexual activity when under the influence of drugs

The nurse manager of a medical/surgical unit wants to improve the alertness of nurses who work the night shift. Which action will be the most helpful? a. Arrange for older staff members to work most night shifts. b. Provide a sleeping area for staff to use for napping at night. c. Post reminders about the relationship of sleep and alertness. d. Schedule nursing staff to rotate day and night shifts monthly.

ANS: B Short on-site naps will improve alertness. Rotating shifts causes the most disruption in sleep habits. Reminding staff members about the impact of lack of sleep on alertness will not improve sleep or alertness. It is not feasible to schedule nurses based on their ages

The nurse should include which food choice when providing dietary teaching for a patient scheduled to receive external beam radiation for abdominal cancer? a. Fresh fruit salad b. Roasted chicken c. Whole wheat toast d. Cream of potato soup

ANS: B To minimize the diarrhea that is commonly associated with bowel radiation, the patient should avoid foods high in roughage, such as fruits and whole grains. Lactose intolerance may develop secondary to radiation, so dairy products should also be avoided

To evaluate the effectiveness of ordered interventions for a patient with ventilatory failure, which diagnostic test will be most useful to the nurse? a. Chest x-ray b. Oxygen saturation c. Arterial blood gas analysis d. Central venous pressure monitoring

ANS: C Arterial blood gas (ABG) analysis is most useful in this setting because ventilatory failure causes problems with CO2 retention, and ABGs provide information about the PaCO2 and pH. The other tests may also be done to help in assessing oxygenation or determining the cause of the patient's ventilatory failure

The nurse is caring for a patient who has an intraaortic balloon pump in place. Which action should be included in the plan of care? a. Position the patient supine at all times. b. Avoid the use of anticoagulant medications. c. Measure the patient's urinary output every hour. d. Provide passive range of motion for all extremities.

ANS: C Monitoring urine output will help determine whether the patient's cardiac output has improved and also help monitor for balloon displacement. The head of the bed can be elevated up to 30 degrees. Heparin is used to prevent thrombus formation. Limited movement is allowed for the extremity with the balloon insertion site to prevent displacement of the balloon

The nurse is caring for a patient with left-sided lung cancer. Which finding would be most important for the nurse to report to the health care provider? a. Hematocrit 32% b. Pain with deep inspiration c. Serum sodium 126 mEq/L d. Decreased breath sounds on left side

ANS: C Syndrome of inappropriate antidiuretic hormone (and the resulting hyponatremia) is an oncologic metabolic emergency and will require rapid treatment in order to prevent complications such as seizures and coma. The other findings also require intervention, but are common in patients with lung cancer and not immediately life threatening

A patient complains of difficulty falling asleep and daytime fatigue for the past 6 weeks. What is the best initial action for the nurse to take in determining whether this patient has chronic insomnia? a. Schedule a polysomnography (PSG) study. b. Arrange for the patient to have a sleep study. c. Ask the patient to keep a 2-week sleep diary. d. Teach the patient about the use of an actigraph.

ANS: C The diagnosis of insomnia is made on the basis of subjective complaints and an evaluation of a 1- to 2-week sleep diary completed by the patient. Actigraphy and PSG studies/sleep studies may be used for determining specific sleep disorders but are not necessary to make an initial insomnia diagnosis

The nurse notes that a patient's endotracheal tube (ET), which was at the 22-cm mark, is now at the 25-cm mark and the patient is anxious and restless. Which action should the nurse take next? a. Offer reassurance to the patient. b. Bag the patient at an FIO2 of 100%. c. Listen to the patient's breath sounds. d. Notify the patient's health care provider.

ANS: C The nurse should first determine whether the ET tube has been displaced into the right mainstem bronchus by listening for unilateral breath sounds. If so, assistance will be needed to reposition the tube immediately. The other actions are also appropriate, but detection and correction of tube malposition are the most critical actions.

When assessing a new patient at the outpatient clinic, the nurse notes dry, scaly skin; thin hair; and thick, brittle nails. What is the nurse's best action? a. Instruct the patient about the importance of nutrition in skin health. b. Make a referral to a podiatrist so that the nails can be safely trimmed. c. Consult with the health care provider about the need for further diagnostic testing. d. Teach the patient about using moisturizing creams and lotions to decrease dry skin.

ANS: C The patient has clinical manifestations that could be caused by systemic problems such as malnutrition or hypothyroidism, so further diagnostic evaluation is indicated. Patient teaching about nutrition, addressing the patient's dry skin, and referral to a podiatrist may also be needed, but the priority is to rule out underlying disease that may be causing these manifestations

A middle-aged patient tells the nurse, "My mother died 4 months ago, and I just can't seem to get over it. I'm not sure it is normal to still think about her every day." Which nursing diagnosis is most appropriate? a. Hopelessness related to inability to resolve grief b. Complicated grieving related to unresolved issues c. Anxiety related to lack of knowledge about normal grieving d. Chronic sorrow related to ongoing distress about loss of mother

ANS: C The patient should be reassured that grieving activities such as frequent thoughts about the deceased are considered normal for months or years after a death. The other nursing diagnoses imply that the patient's grief is unusual or pathologic, which is not the case

A patient with cancer has a nursing diagnosis of imbalanced nutrition: less than body requirements related to altered taste sensation. Which nursing action is most appropriate? a. Add strained baby meats to foods such as casseroles. b. Teach the patient about foods that are high in nutrition. c. Avoid giving the patient foods that are strongly disliked. d. Add extra spice to enhance the flavor of foods that are served.

ANS: C The patient will eat more if disliked foods are avoided and foods that the patient likes are included instead. Additional spice is not usually an effective way to enhance taste. Adding baby meats to foods will increase calorie and protein levels, but does not address the issue of taste. The patient's poor intake is not caused by a lack of information about nutrition

A nursing diagnosis that is likely to be appropriate for a 67-year-old woman who has just been diagnosed with stage III ovarian cancer is a. sexual dysfunction related to loss of vaginal sensation. b. risk for infection related to impaired immune function. c. anxiety related to cancer diagnosis and need for treatment decisions. d. situational low self-esteem related to guilt about delaying medical care.

ANS: C The patient with stage III ovarian cancer is likely to be anxious about the poor prognosis and about the need to make decisions about the multiple treatments that may be used. Decreased vaginal sensation does not occur with ovarian cancer. The patient may develop immune dysfunction when she receives chemotherapy, but she is not currently at risk. It is unlikely that the patient has delayed seeking medical care because the symptoms of ovarian cancer are vague and occur late in the course of the cancer

The nurse has just received change-of-shift report about the following four patients. Which patient should be assessed first? a. A patient with a cervical radium implant in place who is crying in her room b. A patient who is complaining of 5/10 pain after an abdominal hysterectomy c. A patient with a possible ectopic pregnancy who is complaining of shoulder pain d. A patient in the fifteenth week of gestation who has uterine cramping and spotting

ANS: C The patient with the ectopic pregnancy has symptoms consistent with rupture and needs immediate assessment for signs of hemorrhage and possible transfer to surgery. The other patients should also be assessed as quickly as possible but do not have symptoms of life-threatening complications

Which information obtained during the nurse's assessment of a 30-year-old patient's nutritional-metabolic pattern may indicate the risk for musculoskeletal problems? a. The patient takes a multivitamin daily. b. The patient dislikes fruits and vegetables. c. The patient is 5 ft 2 in and weighs 180 lb. d. The patient prefers whole milk to nonfat milk.

ANS: C The patient's height and weight indicate obesity, which places stress on weight-bearing joints. The use of whole milk, avoiding fruits and vegetables, and use of a daily multivitamin are not risk factors for musculoskeletal problems

The second day after admission with a fractured pelvis, a 64-year-old patient suddenly develops confusion. Which action should the nurse take first? a. Take the blood pressure. b. Assess patient orientation. c. Check the oxygen saturation. d. Observe for facial asymmetry.

ANS: C The patient's history and clinical manifestations suggest a fat embolus. The most important assessment is oxygenation. The other actions are also appropriate but will be done after the nurse assesses gas exchange

The nurse is caring for a patient who has diabetes and complains of chronic burning leg pain even when taking oxycodone (OxyContin) twice daily. When reviewing the orders, which prescribed medication is the best choice for the nurse to administer as an adjuvant to decrease the patient's pain? a. Aspirin (Ecotrin) b. Celecoxib (Celebrex) c. Amitriptyline (Elavil) d. Acetaminophen (Tylenol)

ANS: C The patient's pain symptoms are consistent with neuropathic pain and the tricyclic antidepressants are effective for treating this type of pain. The other medications are more effective for nociceptive pain

After completing the health history, the nurse assessing the musculoskeletal system will begin by a. having the patient move the extremities against resistance. b. feeling for the presence of crepitus during joint movement. c. observing the patient's body build and muscle configuration. d. checking active and passive range of motion for the extremities.

ANS: C The usual technique in the physical assessment is to begin with inspection. Abnormalities in muscle mass or configuration will allow the nurse to perform a more focused assessment of abnormal areas. The other assessments are also included in the assessment but are usually done after inspection

A nurse develops a teaching plan for a patient diagnosed with basal cell carcinoma (BCC). Which information should the nurse include in the teaching plan? a. Treatment plans include watchful waiting. b. Screening for metastasis will be important. c. Low dose systemic chemotherapy is used to treat BCC. d. Minimizing sun exposure will reduce risk for future BCC.

ANS: D BCC is frequently associated with sun exposure and preventive measures should be taken for future sun exposure. BCC spreads locally, and does not metastasize to distant tissues. Since BCC can cause local tissue destruction, treatment is indicated. Local (not systemic) chemotherapy may be used to treat BCC

What is the best method to prevent the spread of infection when the nurse is changing the dressing over a wound infected with Staphylococcus aureus? a. Change the dressing using sterile gloves. b. Soak the dressing in sterile normal saline. c. Apply antibiotic ointment over the wound. d. Wash hands and properly dispose of soiled dressings.

ANS: D Careful hand washing and the safe disposal of soiled dressings are the best means of preventing the spread of skin problems. Sterile glove and sterile saline use during wound care will not necessarily prevent spread of infection. Applying antibiotic ointment will treat the bacteria but not necessarily prevent the spread of infection

The nurse is caring for a patient with a subarachnoid hemorrhage who is intubated and placed on a mechanical ventilator with 10 cm H2O of peak end-expiratory pressure (PEEP). When monitoring the patient, the nurse will need to notify the health care provider immediately if the patient develops a. oxygen saturation of 93%. b. respirations of 20 breaths/minute. c. green nasogastric tube drainage. d. increased jugular venous distention.

ANS: D Increases in jugular venous distention in a patient with a subarachnoid hemorrhage may indicate an increase in intracranial pressure (ICP) and that the PEEP setting is too high for this patient. A respiratory rate of 20, O2 saturation of 93%, and green nasogastric tube drainage are within normal limits

The nurse is caring for a patient undergoing plasmapheresis. The nurse should assess the patient for which clinical manifestation? a. Shortness of breath b. High blood pressure c. Transfusion reaction d. Numbness and tingling

ANS: D Numbness and tingling may occur as the result of the hypocalcemia caused by the citrate used to prevent coagulation. The other clinical manifestations are not associated with plasmapheresis

To determine the severity of the symptoms for a 68-year-old patient with benign prostatic hyperplasia (BPH) the nurse will ask the patient about a. blood in the urine. b. lower back or hip pain. c. erectile dysfunction (ED). d. force of the urinary stream.

ANS: D The American Urological Association (AUA) Symptom Index for a patient with BPH asks questions about the force and frequency of urination, nocturia, etc. Blood in the urine, ED, and back or hip pain are not typical symptoms of BPH

The nurse is caring for a patient receiving intravesical bladder chemotherapy. The nurse should monitor for which adverse effect? a. Nausea b. Alopecia c. Mucositis d. Hematuria

ANS: D The adverse effects of intravesical chemotherapy are confined to the bladder. The other adverse effects are associated with systemic chemotherapy

After a transurethral resection of the prostate (TURP), a 64-year-old patient with continuous bladder irrigation complains of painful bladder spasms. The nurse observes clots in the urine. Which action should the nurse take first? a. Increase the flow rate of the bladder irrigation. b. Administer the prescribed IV morphine sulfate. c. Give the patient the prescribed belladonna and opium suppository. d. Manually instill and then withdraw 50 mL of saline into the catheter.

ANS: D The assessment suggests that obstruction by a clot is causing the bladder spasms, and the nurse's first action should be to irrigate the catheter manually and to try to remove the clots. IV morphine will not decrease the spasm, although pain may be reduced. Increasing the flow rate of the irrigation will further distend the bladder and may increase spasms. The belladonna and opium suppository will decrease bladder spasms but will not remove the obstructing blood clot

Which assessment information is most important for the nurse to report to the health care provider when a patient asks for a prescription for testosterone replacement therapy (TRT)? a. The patient has noticed a decrease in energy level for a few years. b. The patient's symptoms have increased steadily over the last few years. c. The patient has been using sildenafil (Viagra) several times every week. d. The patient has had a gradual decrease in the force of his urinary stream.

ANS: D The decrease in urinary stream may indicate benign prostatic hyperplasia (BPH) or prostate cancer, which are contraindications to the use of testosterone replacement therapy (TRT). The other patient data indicate that TRT may be a helpful therapy for the patient

The nurse who notes that a 59-year-old female patient has lost 1 inch in height over the past 2 years will plan to teach the patient about a. discography studies. b. myelographic testing. c. magnetic resonance imaging (MRI). d. dual-energy x-ray absorptiometry (DXA).

ANS: D The decreased height and the patient's age suggest that the patient may have osteoporosis and that bone density testing is needed. Discography, MRI, and myelography are typically done for patients with current symptoms caused by musculoskeletal dysfunction and are not the initial diagnostic tests for osteoporosis

The nurse will determine that more teaching is needed if a patient with discomfort from a bunion says, "I will a. give away my high-heeled shoes." b. take ibuprofen (Motrin) if I need it." c. use the bunion pad to cushion the area." d. only wear sandals, no closed-toe shoes."

ANS: D The patient can wear shoes that have a wide forefoot. The other patient statements indicate that the teaching has been effective

The nurse notes darker skin pigmentation in the skinfolds of a middle-aged patient who has a body mass index of 40 kg/m2. What is the nurse's best action? a. Teach the patient about the treatment of fungal infection. b. Discuss the use of drying agents to minimize infection risk. c. Instruct the patient about the use of mild soap to clean skinfolds. d. Ask the patient about type 2 diabetes or if there is a family history of it.

ANS: D The presence of acanthosis nigricans in skinfolds suggests either having type 2 diabetes or being at an increased risk for it. The description of the patient's skin does not indicate problems with fungal infection, poor hygiene, or the need to dry the skinfolds better

A factory line worker has repetitive strain syndrome in the left elbow. The nurse will plan to teach the patient about a. surgical options. b. elbow injections. c. wearing a left wrist splint. d. modifying arm movements.

ANS: D Treatment for repetitive strain syndrome includes changing the ergonomics of the activity. Elbow injections and surgery are not initial options for this type of injury. A wrist splint might be used for hand or wrist pain

A patient with osteomyelitis is to receive vancomycin (Vancocin) 500 mg IV every 6 hours. The vancomycin is diluted in 100 mL of normal saline and needs to be administered over 1 hour. The nurse will set the IV pump for how many mL/minute? (Round to the nearest hundredth.)

ANS: 1.67 To administer 100 mL in 60 minutes, the IV pump will need to provide 1.67 mL/minute.

A 198-lb patient is to receive a dobutamine infusion at 5 mcg/kg/minute. The label on the infusion bag states: dobutamine 250 mg in 250 mL normal saline. When setting the infusion pump, the nurse will set the infusion rate at how many mL per hour?

ANS: 27 In order to administer the dobutamine at the prescribed rate of 5 mcg/kg/minute from a concentration of 250 mg in 250 mL, the nurse will need to infuse 27 mL/hour.

The nurse is caring for a patient who has an intraortic balloon pump (IABP) following a massive heart attack. When assessing the patient, the nurse notices blood backing up into the IABP catheter. In which order should the nurse take the following actions? (Put a comma and a space between each answer choice [A, B, C, D].) a. Ensure that the IABP console has turned off. b. Assess the patient's vital signs and orientation. c. Obtain supplies for insertion of a new IABP catheter. d. Notify the health care provider of the IABP malfunction

ANS: A, B, D, C Blood in the IABP catheter indicates a possible tear in the balloon. The console will shut off automatically to prevent complications such as air embolism. Next, the nurse will assess the patient and communicate with the health care provider about the patient's assessment and the IABP problem. Finally, supplies for insertion of a new IABP catheter may be needed, based on the patient assessment and the decision of the health care provider.

In which order will the nurse implement these collaborative interventions prescribed for a patient being admitted who has acute osteomyelitis with a temperature of 101.2° F? (Put a comma and a space between each answer choice [A, B, C, D].) a. Obtain blood cultures from two sites. b. Send to radiology for computed tomography (CT) scan of right leg. c. Administer gentamicin (Garamycin) 60 mg IV. d. Administer acetaminophen (Tylenol) now and every 4 hours PRN for fever

ANS: A, C, D, B The highest priority for possible osteomyelitis is initiation of antibiotic therapy, but cultures should be obtained before administration of antibiotics. Addressing the discomfort of the fever is the next highest priority. Because the purpose of the CT scan is to determine the extent of the infection, it can be done last

A patient who is receiving an IV antibiotic develops wheezes and dyspnea. In which order should the nurse implement these prescribed actions? (Put a comma and a space between each answer choice [A, B, C, D, E]). a. Discontinue the antibiotic infusion. b. Give diphenhydramine (Benadryl) IV. c. Inject epinephrine (Adrenalin) IM or IV. d. Prepare an infusion of dopamine (Intropin). e. Start 100% oxygen using a nonrebreather mask

ANS: A, E, C, B, D The nurse should initially discontinue the antibiotic because it is the likely cause of the allergic reaction. Next, oxygen delivery should be maximized, followed by treatment of bronchoconstriction with epinephrine administered IM or IV. Diphenhydramine will work more slowly than epinephrine, but will help prevent progression of the reaction. Because the patient currently does not have evidence of hypotension, the dopamine infusion can be prepared last

In which order will the nurse take these actions when caring for a patient in the emergency department with a right leg fracture after a motor vehicle accident? (Put a comma and a space between each answer choice [A, B, C, D, E, F].) a. Obtain x-rays. b. Check pedal pulses. c. Assess lung sounds. d. Take blood pressure. e. Apply splint to the leg. f. Administer tetanus prophylaxis

ANS: C, D, B, E, A, F The initial actions should be to ensure that airway, breathing, and circulation are intact. This should be followed by checking the neurovascular status of the leg (before and after splint application). Application of a splint to immobilize the leg should be done before sending the patient for x-rays. The tetanus prophylaxis is the least urgent of the actions

When assisting with oral intubation of a patient who is having respiratory distress, in which order will the nurse take these actions? (Put a comma and a space between each answer choice [A, B, C, D, E].) a. Obtain a portable chest-x-ray. b. Position the patient in the supine position. c. Inflate the cuff of the endotracheal tube after insertion. d. Attach an end-tidal CO2 detector to the endotracheal tube. e. Oxygenate the patient with a bag-valve-mask device for several minutes

ANS: E, B, C, D, A The patient is pre-oxygenated with a bag-valve-mask system for 3 to 5 minutes before intubation and then placed in a supine position. Following the intubation, the cuff on the endotracheal tube is inflated to occlude and protect the airway. Tube placement is assessed first with an end-tidal CO2 sensor, then with a chest x-ray

A patient with chronic pain who has been receiving morphine sulfate 20 mg IV over 24 hours is to be discharged home on oral sustained-release morphine (MS Contin), which will be administered twice a day. What dosage of MS Contin will be needed for each dose to obtain an equianalgesic dose for the patient? (Morphine sulfate 10 mg IV is equianalgesic to morphine sulfate 30 mg orally.)

ANS: MS Contin 30 mg/dose Morphine sulfate 20 mg IV over 24 hours will be equianalgesic to MS Contin 60 mg in 24 hours. Since the total dose needs to be divided into two doses, each dose should be 30 mg.

As the nurse admits a patient in end-stage kidney disease to the hospital, the patient tells the nurse, "If my heart or breathing stop, I do not want to be resuscitated." Which action is best for the nurse to take? a. Ask if these wishes have been discussed with the health care provider. b. Place a "Do Not Resuscitate" (DNR) notation in the patient's care plan. c. Inform the patient that a notarized advance directive must be included in the record or resuscitation must be performed. d. Advise the patient to designate a person to make health care decisions when the patient is not able to make them independently.

ANS: A A health care provider's order should be written describing the actions that the nurses should take if the patient requires CPR, but the primary right to decide belongs to the patient or family. The nurse should document the patient's request but does not have the authority to place the DNR order in the care plan. A notarized advance directive is not needed to establish the patient's wishes. The patient may need a durable power of attorney for health care (or the equivalent), but this does not address the patient's current concern with possible resuscitation

A 56-year-old woman is concerned about having a moderate amount of vaginal bleeding after 4 years of menopause. The nurse will anticipate teaching the patient about a. endometrial biopsy. b. endometrial ablation. c. uterine balloon therapy. d. dilation and curettage (D&C).

ANS: A A postmenopausal woman with vaginal bleeding should be evaluated for endometrial cancer, and endometrial biopsy is the primary test for endometrial cancer. D&C will be needed only if the biopsy does not provide sufficient information to make a diagnosis. Endometrial ablation and balloon therapy are used to treat menorrhagia, which is unlikely in this patient

A patient is admitted to the hospital with acute rejection of a kidney transplant. Which intervention will the nurse prepare for this patient? a. Administration of immunosuppressant medications b. Insertion of an arteriovenous graft for hemodialysis c. Placement of the patient on the transplant waiting list d. A blood draw for human leukocyte antigen (HLA) matching

ANS: A Acute rejection is treated with the administration of additional immunosuppressant drugs such as corticosteroids. Because acute rejection is potentially reversible, there is no indication that the patient will require another transplant or hemodialysis. There is no indication for repeat HLA testing

Norepinephrine (Levophed) has been prescribed for a patient who was admitted with dehydration and hypotension. Which patient data indicate that the nurse should consult with the health care provider before starting the norepinephrine? a. The patient's central venous pressure is 3 mm Hg. b. The patient is in sinus tachycardia at 120 beats/min. c. The patient is receiving low dose dopamine (Intropin). d. The patient has had no urine output since being admitted.

ANS: A Adequate fluid administration is essential before administration of vasopressors to patients with hypovolemic shock. The patient's low central venous pressure indicates a need for more volume replacement. The other patient data are not contraindications to norepinephrine administration

A patient who has vague symptoms of fatigue, headaches, and a positive test for human immunodeficiency virus (HIV) antibodies using an enzyme immunoassay (EIA) test. What instructions should the nurse give to this patient? a. "The EIA test will need to be repeated to verify the results." b. "A viral culture will be done to determine the progression of the disease." c. "It will probably be 10 or more years before you develop acquired immunodeficiency syndrome (AIDS)." d. "The Western blot test will be done to determine whether acquired immunodeficiency syndrome (AIDS) has developed."

ANS: A After an initial positive EIA test, the EIA is repeated before more specific testing such as the Western blot is done. Viral cultures are not usually part of HIV testing. It is not appropriate for the nurse to predict the time frame for AIDS development. The Western blot tests for HIV antibodies, not for AIDS

Which action is best for the nurse to take to ensure culturally competent care for an alert, terminally ill Filipino patient? a. Ask the patient and family about their preferences for care during this time. b. Let the family decide whether to tell the patient about the terminal diagnosis. c. Obtain information from Filipino staff members about possible cultural needs. d. Remind family members that dying patients prefer to have someone at the bedside.

ANS: A Because cultural beliefs may vary among people of the same ethnicity, the nurse's best action is to assess the expectations of both the patient and family. The other actions may be appropriate, but the nurse can only plan for individualized culturally competent care after assessment of this patient and family.

A 70-year-old patient who has had a transurethral resection of the prostate (TURP) for benign prostatic hyperplasia (BPH) is being discharged from the hospital today, The nurse determines that additional instruction is needed when the patient says which of the following? a. "I should call the doctor if I have incontinence at home." b. "I will avoid driving until I get approval from my doctor." c. "I will increase fiber and fluids in my diet to prevent constipation." d. "I should continue to schedule yearly appointments for prostate exams."

ANS: A Because incontinence is common for several weeks after a TURP, the patient does not need to call the health care provider if this occurs. The other patient statements indicate that the patient has a good understanding of post-TURP instructions

The nurse working in the dermatology clinic assesses a young adult female patient who is taking isotretinoin (Accutane) to treat severe cystic acne. Which assessment finding is most indicative of a need for further questioning of the patient? a. The patient recently had an intrauterine device removed. b. The patient already has some acne scarring on her forehead. c. The patient has also used topical antibiotics to treat the acne. d. The patient has a strong family history of rheumatoid arthritis.

ANS: A Because isotretinoin is teratogenic, contraception is required for women who are using this medication. The nurse will need to determine whether the patient is using other birth control methods. More information about the other patient data may also be needed, but the other data do not indicate contraindications to isotretinoin use

Which information about a patient who is receiving cisatracurium (Nimbex) to prevent asynchronous breathing with the positive pressure ventilator requires immediate action by the nurse? a. Only continuous IV opioids have been ordered. b. The patient does not respond to verbal stimulation. c. There is no cough or gag when the patient is suctioned. d. The patient's oxygen saturation fluctuates between 90% to 93%.

ANS: A Because neuromuscular blockade is extremely anxiety provoking, it is essential that patients who are receiving neuromuscular blockade receive concurrent sedation and analgesia. Absence of response to stimuli is expected in patients receiving neuromuscular blockade. The oxygen saturation is adequate

A patient with septic shock has a BP of 70/46 mm Hg, pulse 136, respirations 32, temperature 104° F, and blood glucose 246 mg/dL. Which intervention ordered by the health care provider should the nurse implement first? a. Give normal saline IV at 500 mL/hr. b. Give acetaminophen (Tylenol) 650 mg rectally. c. Start insulin drip to maintain blood glucose at 110 to 150 mg/dL. d. Start norepinephrine (Levophed) to keep systolic blood pressure >90 mm Hg.

ANS: A Because of the low systemic vascular resistance (SVR) associated with septic shock, fluid resuscitation is the initial therapy. The other actions also are appropriate, and should be initiated quickly as well

A patient who uses a fentanyl (Duragesic) patch for chronic abdominal pain caused by ovarian cancer asks the nurse to administer the prescribed hydrocodone (Vicodin) tablets, but the patient is asleep when the nurse returns with the medication. Which action is best for the nurse to take? a. Wake the patient and administer the hydrocodone. b. Wait until the patient wakes up and reassess the pain. c. Suggest the use of nondrug therapies for pain relief instead of additional opioids. d. Consult with the health care provider about changing the fentanyl (Duragesic) dose.

ANS: A Because patients with chronic pain frequently use withdrawal and decreased activity as coping mechanisms for pain, sleep is not an indicator that the patient is pain free. The nurse should wake the patient and administer the hydrocodone

A patient in the dermatology clinic has a thin, scaly erythematous plaque on the right cheek. Which action should the nurse take? a. Prepare the patient for a biopsy. b. Teach about the use of corticosteroid creams. c. Explain how to apply tretinoin (Retin-A) to the face. d. Discuss the need for topical application of antibiotics.

ANS: A Because the appearance of the lesion suggests actinic keratosis or possible squamous cell carcinoma (SCC), the appropriate treatment would be excision and biopsy. Over-the-counter (OTC) corticosteroids, topical antibiotics, and Retin-A would not be used for this lesion

A patient with urinary obstruction from benign prostatic hyperplasia (BPH) tells the nurse, "My symptoms are much worse this week." Which response by the nurse is most appropriate? a. "Have you been taking any over-the-counter (OTC) medications recently?" b. "I will talk to the doctor about ordering a prostate specific antigen (PSA) test." c. "Have you talked to the doctor about surgery such as transurethral resection of the prostate (TURP)?" d. "The prostate gland changes in size from day to day, and this may be making your symptoms worse."

ANS: A Because the patient's increase in symptoms has occurred abruptly, the nurse should ask about OTC medications that might cause contraction of the smooth muscle in the prostate and worsen obstruction. The prostate gland does not vary in size from day to day. A TURP may be needed, but more assessment about possible reasons for the sudden symptom change is a more appropriate first response by the nurse. PSA testing is done to differentiate BPH from prostatic cancer

The nurse evaluating effectiveness of prescribed calcitonin (Cibacalcin) and ibandronate (Boniva) for a patient with Paget's disease will consider the patient's a. pain level. b. oral intake. c. daily weight. d. grip strength.

ANS: A Bone pain is one of the common early manifestations of Paget's disease, and the nurse should assess the pain level to determine whether the treatment is effective. The other information will also be collected by the nurse, but will not be used in evaluating the effectiveness of the therapy

A patient who is diagnosed with cervical cancer that is classified as Tis, N0, M0 asks the nurse what the letters and numbers mean. Which response by the nurse is most appropriate? a. "The cancer involves only the cervix." b. "The cancer cells look almost like normal cells." c. "Further testing is needed to determine the spread of the cancer." d. "It is difficult to determine the original site of the cervical cancer."

ANS: A Cancer in situ indicates that the cancer is localized to the cervix and is not invasive at this time. Cell differentiation is not indicated by clinical staging. Because the cancer is in situ, the origin is the cervix. Further testing is not indicated given that the cancer has not spread

Which statement by the patient indicates that the nurse's teaching about treating vaginal candidiasis has been effective? a. "I should clean carefully after each urination and bowel movement." b. "I can douche with warm water if the itching continues to bother me." c. "I will insert the antifungal cream right before I get up in the morning." d. "I will tell my husband that we cannot have intercourse for the next month."

ANS: A Cleaning of the perineal area will decrease itching caused by contact of the irritated tissues with urine and reduce the chance of further infection of irritated tissues by bacteria in the stool. Sexual intercourse should be avoided for 1 week. Douching will disrupt normal protective mechanisms in the vagina. The cream should be used at night so that it will remain in the vagina for longer periods of time

A patient with chronic back pain has learned to control the pain with the use of imagery and hypnosis. The patient's spouse asks the nurse how these techniques work. Which response by the nurse is best? a. "The strategies work by affecting the perception of pain." b. "These techniques block the pain pathways of the nerves." c. "Both strategies prevent transmission of painful stimuli to the brain." d. "The therapies slow the release of chemicals in the spinal cord that cause pain."

ANS: A Cognitive therapies affect the perception of pain by the brain rather than affecting efferent or afferent pathways or influencing the release of chemical transmitters in the dorsal horn

A 58-year-old patient who has undergone a radical vulvectomy for vulvar carcinoma returns to the medical-surgical unit after the surgery. The priority nursing diagnosis for the patient at this time is a. risk for infection related to contact of the wound with urine and stool. b. self-care deficit: bathing/hygiene related to pain and difficulty moving. c. imbalanced nutrition: less than body requirements related to low-residue diet. d. risk for ineffective sexual pattern related to disfiguration caused by the surgery.

ANS: A Complex and meticulous wound care is needed to prevent infection and delayed wound healing. The other nursing diagnoses may also be appropriate for the patient but are not the highest priority immediately after surgery

Which infection, reported in the health history of a woman who is having difficulty conceiving, will the nurse identify as a risk factor for infertility? a. N. gonorrhoeae b. Treponema pallidum c. Condyloma acuminatum d. Herpes simplex virus type 2

ANS: A Complications of gonorrhea include scarring of the fallopian tubes, which can lead to tubal pregnancies and infertility. Syphilis, genital warts, and genital herpes do not lead to problems with conceiving, although transmission to the fetus (syphilis) or newborn (genital warts or genital herpes) is a concern

The nurse teaches a patient about drug therapy after a kidney transplant. Which statement by the patient would indicate a need for further instructions? a. "After a couple of years, it is likely that I will be able to stop taking the cyclosporine." b. "If I develop an acute rejection episode, I will need to have other types of drugs given IV." c. "I need to be monitored closely because I have a greater chance of developing malignant tumors." d. "The drugs are given in combination because they inhibit different ways the kidney can be rejected."

ANS: A Cyclosporine, a calcineurin inhibitor, will need to be continued for life. The other patient statements are accurate and indicate that no further teaching is necessary about those topics

A 28-year-old patient with endometriosis asks why she is being treated with medroxyprogesterone (Depo-Provera), a medication that she thought was an oral contraceptive. The nurse explains that this therapy a. suppresses the menstrual cycle by mimicking pregnancy. b. will relieve symptoms such as vaginal atrophy and hot flashes. c. prevents a pregnancy that could worsen the menstrual bleeding. d. will lead to permanent suppression of abnormal endometrial tissues.

ANS: A Depo-Provera induces a pseudopregnancy, which suppresses ovulation and causes shrinkage of endometrial tissue. Menstrual bleeding does not occur during pregnancy. Vaginal atrophy and hot flashes are caused by synthetic androgens such as danazol or gonadotropin-releasing hormone agonists (GNRH) such as leuprolide. Although hormonal therapies will control endometriosis while the therapy is used, endometriosis will recur once the menstrual cycle is reestablished

A nurse is caring for a patient who is orally intubated and receiving mechanical ventilation. To decrease the risk for ventilator-associated pneumonia, which action will the nurse include in the plan of care? a. Elevate head of bed to 30 to 45 degrees. b. Suction the endotracheal tube every 2 to 4 hours. c. Limit the use of positive end-expiratory pressure. d. Give enteral feedings at no more than 10 mL/hr.

ANS: A Elevation of the head decreases the risk for aspiration. Positive end-expiratory pressure is frequently needed to improve oxygenation in patients receiving mechanical ventilation. Suctioning should be done only when the patient assessment indicates that it is necessary. Enteral feedings should provide adequate calories for the patient's high energy needs

A 78-kg patient with septic shock has a urine output of 30 mL/hr for the past 3 hours. The pulse rate is 120/minute and the central venous pressure and pulmonary artery wedge pressure are low. Which order by the health care provider will the nurse question? a. Give PRN furosemide (Lasix) 40 mg IV. b. Increase normal saline infusion to 250 mL/hr. c. Administer hydrocortisone (Solu-Cortef) 100 mg IV. d. Titrate norepinephrine (Levophed) to keep systolic BP >90 mm Hg.

ANS: A Furosemide will lower the filling pressures and renal perfusion further for the patient with septic shock. The other orders are appropriate

Which patient will the nurse plan on teaching about the Gardasil vaccine? a. A 24-year-old female who has not been sexually active b. A 34-year-old woman who has multiple sexual partners c. A 19-year-old woman who is pregnant for the first time d. A 29-year-old woman who is in a monogamous relationship

ANS: A Gardasil is recommended for females ages 9 through 26, preferably those who have never been sexually active. It is not recommended for women during pregnancy or for older women

The nurse will plan to teach the female patient with genital warts about the a. importance of regular Pap tests. b. increased risk for endometrial cancer. c. appropriate use of oral contraceptives. d. symptoms of pelvic inflammatory disease (PID).

ANS: A Genital warts are caused by the human papillomavirus (HPV) and increase the risk for cervical cancer. There is no indication that the patient needs teaching about PID, oral contraceptives, or endometrial cancer

The nurse will inform a patient with cancer of the prostate that side effects of leuprolide (Lupron) may include a. flushing. b. dizziness. c. infection. d. incontinence.

ANS: A Hot flashes may occur with decreased testosterone production. Dizziness may occur with the alpha-blockers used for benign prostatic hyperplasia (BPH). Urinary incontinence may occur after prostate surgery, but it is not an expected side effect of medication. Risk for infection is increased in patients receiving chemotherapy

Interleukin-2 (IL-2) is used as adjuvant therapy for a patient with metastatic renal cell carcinoma. Which information should the nurse include when explaining the purpose of this therapy to the patient? a. IL-2 enhances the immunologic response to tumor cells. b. IL-2 stimulates malignant cells in the resting phase to enter mitosis. c. IL-2 prevents the bone marrow depression caused by chemotherapy. d. IL-2 protects normal cells from the harmful effects of chemotherapy.

ANS: A IL-2 enhances the ability of the patient's own immune response to suppress tumor cells. IL-2 does not protect normal cells from damage caused by chemotherapy, stimulate malignant cells to enter mitosis, or prevent bone marrow depression

While caring for a patient who has been admitted with a pulmonary embolism, the nurse notes a change in the patient's oxygen saturation (SpO2) from 94% to 88%. Which action should the nurse take next? a. Increase the oxygen flow rate. b. Suction the patient's oropharynx. c. Instruct the patient to cough and deep breathe. d. Help the patient to sit in a more upright position.

ANS: A Increasing oxygen flow rate will usually improve oxygen saturation in patients with ventilation-perfusion mismatch, as occurs with pulmonary embolism. Because the problem is with perfusion, actions that improve ventilation, such as deep breathing and coughing, sitting upright, and suctioning, are not likely to improve oxygenation

When admitting a patient with possible respiratory failure with a high PaCO2, which assessment information should be immediately reported to the health care provider? a. The patient is somnolent. b. The patient complains of weakness. c. The patient's blood pressure is 164/98. d. The patient's oxygen saturation is 90%.

ANS: A Increasing somnolence will decrease the patient's respiratory rate and further increase the PaCO2 and respiratory failure. Rapid action is needed to prevent respiratory arrest. An SpO2 of 90%, weakness, and elevated blood pressure all require ongoing monitoring but are not indicators of possible impending respiratory arrest.

A 50-year-old patient is diagnosed with uterine bleeding caused by a leiomyoma. Which information will the nurse include in the patient teaching plan? a. The symptoms may decrease after the patient undergoes menopause. b. The tumor size is likely to increase throughout the patient's lifetime. c. Aspirin or acetaminophen may be used to control mild to moderate pain. d. The patient will need frequent monitoring to detect any malignant changes.

ANS: A Leiomyomas appear to depend on ovarian hormones and will atrophy after menopause, leading to a decrease in symptoms. Aspirin use is discouraged because the antiplatelet effects may lead to heavier uterine bleeding. The size of the tumor will shrink after menopause. Leiomyomas are benign tumors that do not undergo malignant changes.

A patient who is receiving positive pressure ventilation is scheduled for a spontaneous breathing trial (SBT). Which finding by the nurse is most important to discuss with the health care provider before starting the SBT? a. New ST segment elevation is noted on the cardiac monitor. b. Enteral feedings are being given through an orogastric tube. c. Scattered rhonchi are heard when auscultating breath sounds. d. HYDROmorphone (Dilaudid) is being used to treat postoperative pain.

ANS: A Myocardial ischemia is a contraindication for ventilator weaning. The ST segment elevation is an indication that weaning should be postponed until further investigation and/or treatment for myocardial ischemia can be done. The other information will also be shared with the health care provider, but ventilator weaning can proceed when opioids are used for pain management, abnormal lung sounds are present, or enteral feedings are being used

A patient who has just started taking sustained-release morphine sulfate (MS Contin) for chronic arthritic joint pain following a traumatic injury complains of nausea and abdominal fullness. Which action should the nurse take initially? a. Administer the ordered antiemetic medication. b. Tell the patient that the nausea will subside in about a week. c. Order the patient a clear liquid diet until the nausea decreases. d. Consult with the health care provider about using a different opioid.

ANS: A Nausea is frequently experienced with the initiation of opioid therapy, and antiemetics usually are prescribed to treat this expected side effect. There is no indication that a different opioid is needed, although if the nausea persists, the health care provider may order a change of opioid. Although tolerance develops and the nausea will subside in about a week, it is not appropriate to allow the patient to continue to be nauseated. A clear liquid diet may decrease the nausea, but the best choice would be to administer the antiemetic medication and allow the patient to eat

A 46-year-old man who has had blood drawn for an insurance screening has a positive Venereal Disease Research Laboratory (VDRL) test. Which action should the nurse take next? a. Ask the patient about past treatment for syphilis. b. Explain the need for blood and spinal fluid cultures. c. Obtain a specimen for fluorescent treponemal antibody absorption (FAT-Abs) testing. d. Assess for the presence of chancres, flulike symptoms, or a bilateral rash on the trunk.

ANS: A Once antibody testing is positive for syphilis, the antibodies remain present for an indefinite period of time even after successful treatment, so the nurse should inquire about previous treatment before doing other assessments or testing. Culture, FAT-Abs testing, and assessment for symptoms may be appropriate, based on whether the patient has been previously treated for syphilis

The nurse will plan to provide teaching for a 67-year-old patient who has been diagnosed with orchitis about a. pain management. b. emergency surgical repair. c. application of heat to the scrotum. d. aspiration of fluid from the scrotal sac.

ANS: A Orchitis is very painful, and effective pain management will be needed. Heat, aspiration, and surgery are not used to treat orchitis

Which action will the nurse take when caring for a patient with osteomalacia? a. Teach about the use of vitamin D supplements. b. Educate about the need for weight-bearing exercise. c. Discuss the use of medications such as bisphosphonates. d. Emphasize the importance of sunscreen use when outside.

ANS: A Osteomalacia is caused by inadequate intake or absorption of vitamin D. Weight-bearing exercise and bisphosphonate administration may be used for osteoporosis but will not be beneficial for osteomalacia. Because ultraviolet light is needed for the body to synthesize vitamin D, the patient might be taught that 20 minutes/day of sun exposure is beneficial

An assessment finding for a 55-year-old patient that alerts the nurse to the presence of osteoporosis is a. a measurable loss of height. b. the presence of bowed legs. c. the aversion to dairy products. d. a statement about frequent falls.

ANS: A Osteoporosis occurring in the vertebrae produces a gradual loss of height. Bowed legs are associated with osteomalacia. Low intake of dairy products is a risk factor for osteoporosis, but it does not indicate that osteoporosis is present. Frequent falls increase the risk for fractures but are not an indicator of osteoporosis

A patient has a new order for magnetic resonance imaging (MRI) to evaluate for left femur osteomyelitis after a hip replacement surgery. Which information indicates that the nurse should consult with the health care provider before scheduling the MRI? a. The patient has a pacemaker. b. The patient is claustrophobic. c. The patient wears a hearing aid. d. The patient is allergic to shellfish.

ANS: A Patients with permanent pacemakers cannot have MRI because of the force exerted by the magnetic field on metal objects. An open MRI will not cause claustrophobia. The patient will need to be instructed to remove the hearing aid before the MRI, but this does not require consultation with the health care provider. Because contrast medium will not be used, shellfish allergy is not a contraindication to MRI

A patient with a right lower leg fracture will be discharged home with an external fixation device in place. Which information will the nurse teach? a. "You will need to check and clean the pin insertion sites daily." b. "The external fixator can be removed for your bath or shower." c. "You will need to remain on bed rest until bone healing is complete." d. "Prophylactic antibiotics are used until the external fixator is removed."

ANS: A Pin insertion sites should be cleaned daily to decrease the risk for infection at the site. An external fixator allows the patient to be out of bed and avoid the risks of prolonged immobility. The device is surgically placed and is not removed until the bone is stable. Prophylactic antibiotics are not routinely given when an external fixator is used

A 22-year-old tells the nurse that she has not had a menstrual period for the last 2 months. Which action is most important for the nurse to take? a. Obtain a urine specimen for a pregnancy test. b. Ask about any recent stressful lifestyle changes. c. Measure the patient's current height and weight. d. Question the patient about prescribed medications.

ANS: A Pregnancy should always be considered a possible cause of amenorrhea in women of childbearing age. The other actions are also appropriate, but it is important to check for pregnancy in this patient because pregnancy will require rapid implementation of actions to promote normal fetal development such as changes in lifestyle, folic acid intake, etc.

Which patient exposure by the nurse is most likely to require postexposure prophylaxis when the patient's human immunodeficiency virus (HIV) status is unknown? a. Needle stick with a needle and syringe used to draw blood b. Splash into the eyes when emptying a bedpan containing stool c. Contamination of open skin lesions with patient vaginal secretions d. Needle stick injury with a suture needle during a surgical procedure

ANS: A Puncture wounds are the most common means for workplace transmission of blood-borne diseases, and a needle with a hollow bore that had been contaminated with the patient's blood would be a high-risk situation. The other situations described would be much less likely to result in transmission of the virus

The occupational health nurse will teach the patient whose job involves many hours of typing about the need to a. obtain a keyboard pad to support the wrist. b. do stretching exercises before starting work. c. wrap the wrists with compression bandages every morning. d. avoid using nonsteroidal antiinflammatory drugs (NSAIDs) for pain.

ANS: A Repetitive strain injuries caused by prolonged times working at a keyboard can be prevented by the use of a pad that will keep the wrists in a straight position. Stretching exercises during the day may be helpful, but these would not be needed before starting. Use of a compression bandage is not needed, although a splint may be used for carpal tunnel syndrome. NSAIDs are appropriate to use to decrease swelling

Which nursing action for a patient who has had right hip replacement surgery can the nurse delegate to experienced unlicensed assistive personnel (UAP)? a. Reposition the patient every 1 to 2 hours. b. Assess for skin irritation on the patient's back. c. Teach the patient quadriceps-setting exercises. d. Determine the patient's pain level and tolerance.

ANS: A Repositioning of orthopedic patients is within the scope of practice of UAP (after they have been trained and evaluated in this skill). The other actions should be done by licensed nursing staff members

The nurse assesses a patient with non-Hodgkin's lymphoma who is receiving an infusion of rituximab (Rituxan). Which assessment finding would require the most rapid action by the nurse? a. Shortness of breath b. Temperature 100.2° F (37.9° C) c. Shivering and complaint of chills d. Generalized muscle aches and pains

ANS: A Rituximab (Rituxan) is a monoclonal antibody. Shortness of breath should be investigated rapidly because anaphylaxis is a possible reaction to monoclonal antibody administration. The nurse will need to rapidly take actions such as stopping the infusion, assessing the patient further, and notifying the health care provider. The other findings will also require action by the nurse, but are not indicative of life-threatening complications

A patient is being evaluated for possible atopic dermatitis. The nurse expects elevation of which laboratory value? a. IgE b. IgA c. Basophils d. Neutrophils

ANS: A Serum IgE is elevated in an allergic response (type 1 hypersensitivity disorders). The eosinophil level will be elevated rather than neutrophil or basophil counts. IgA is located in body secretions and would not be tested when evaluating a patient who has symptoms of atopic dermatitis

A 29-year-old female patient is diagnosed with Chlamydia during a routine pelvic examination. The nurse knows that teaching regarding the management of the condition has been effective when the patient says which of the following? a. "My partner will need to take antibiotics at the same time I do." b. "Go ahead and give me the antibiotic injection, so I will be cured." c. "I will use condoms during sex until I finish taking all the antibiotics." d. "I do not plan on having children, so treating the infection is not important."

ANS: A Sex partners should be treated simultaneously to prevent reinfection. Chlamydia is treated with oral antibiotics. Abstinence from sexual intercourse is recommended for 7 days after treatment, and condoms should be recommended during all sexual contacts to prevent infection. Chronic pelvic pain, as well as infertility, can result from untreated Chlamydia

A nurse is caring for a patient with acute respiratory distress syndrome (ARDS) who is receiving mechanical ventilation using synchronized intermittent mandatory ventilation (SIMV). The settings include fraction of inspired oxygen (FIO2) 80%, tidal volume 450, rate 16/minute, and positive end-expiratory pressure (PEEP) 5 cm. Which assessment finding is most important for the nurse to report to the health care provider? a. Oxygen saturation 99% b. Respiratory rate 22 breaths/minute c. Crackles audible at lung bases d. Heart rate 106 beats/minute

ANS: A The FIO2 of 80% increases the risk for oxygen toxicity. Because the patient's O2 saturation is 99%, a decrease in FIO2 is indicated to avoid toxicity. The other patient data would be typical for a patient with ARDS and would not need to be urgently reported to the health care provider

The nurse is caring for a 33-year-old patient who arrived in the emergency department with acute respiratory distress. Which assessment finding by the nurse requires the most rapid action? a. The patient's PaO2 is 45 mm Hg. b. The patient's PaCO2 is 33 mm Hg. c. The patient's respirations are shallow. d. The patient's respiratory rate is 32 breaths/minute.

ANS: A The PaO2 indicates severe hypoxemia and respiratory failure. Rapid action is needed to prevent further deterioration of the patient. Although the shallow breathing, rapid respiratory rate, and low PaCO2 also need to be addressed, the most urgent problem is the patient's poor oxygenation

What teaching should be included in the plan of care for a patient with narcolepsy? a. Driving an automobile may be possible with appropriate treatment of narcolepsy. b. Changes in sleep hygiene are ineffective in improving sleep quality in narcolepsy. c. Antidepressant drugs are prescribed to treat the depression caused by the disorder. d. Stimulant drugs should be used for only a short time because of the risk for abuse.

ANS: A The accident rate for patients with narcolepsy who are receiving appropriate treatment is similar to the general population. Stimulant medications are used on an ongoing basis for patients with narcolepsy. The purpose of antidepressant drugs in the treatment of narcolepsy is the management of cataplexy, not to treat depression. Changes in sleep hygiene are recommended for patients with narcolepsy to improve sleep quality

A 48-year-old woman in the emergency department reports that she has been sexually assaulted. Which action by the nurse will be most important in maintaining the medicolegal chain of evidence? a. Labeling all specimens and other materials obtained from the patient. b. Assisting the patient in filling out the application for financial compensation. c. Discussing the availability of the "morning-after pill" for pregnancy prevention. d. Educating the patient about baseline sexually transmitted infection (STI) testing.

ANS: A The careful labeling of specimens and materials will assist in maintaining the chain of evidence. Assisting with paperwork, and discussing STIs and pregnancy prevention are interventions that might be appropriate after sexual assault, but they do not help maintain the legal chain of evidence

Which assessment finding obtained by the nurse when caring for a patient with a right radial arterial line indicates a need for the nurse to take immediate action? a. The right hand is cooler than the left hand. b. The mean arterial pressure (MAP) is 77 mm Hg. c. The system is delivering 3 mL of flush solution per hour. d. The flush bag and tubing were last changed 3 days previously.

ANS: A The change in temperature of the left hand suggests that blood flow to the left hand is impaired. The flush system needs to be changed every 96 hours. A mean arterial pressure (MAP) of 75 mm Hg is normal. Flush systems for hemodynamic monitoring are set up to deliver 3 to 6 mL/hour of flush solution

During change-of-shift report, the nurse is told that a patient has been admitted with dehydration and hypotension after having vomiting and diarrhea for 4 days. Which finding is most important for the nurse to report to the health care provider? a. New onset of confusion b. Heart rate 112 beats/minute c. Decreased bowel sounds d. Pale, cool, and dry extremities

ANS: A The changes in mental status are indicative that the patient is in the progressive stage of shock and that rapid intervention is needed to prevent further deterioration. The other information is consistent with compensatory shock

A 19-year-old visits the health clinic for a routine checkup. Which question should the nurse ask to determine whether a Pap test is needed? a. "Have you had sexual intercourse?" b. "Do you use any illegal substances?" c. "Do you have cramping with your periods?" d. "At what age did your menstrual periods start?"

ANS: A The current American Cancer Society recommendation is that a Pap test be done every 3 years, starting 3 years after the first sexual intercourse and no later than age 21. The information about menstrual periods and substance abuse will not help determine whether the patient requires a Pap test

Which information would be most important to help the nurse determine if the patient needs human immunodeficiency virus (HIV) testing? a. Patient age b. Patient lifestyle c. Patient symptoms d. Patient sexual orientation

ANS: A The current Center for Disease Control (CDC) policy is to offer routine testing for HIV to all individuals age 13 to 64. Although lifestyle, symptoms, and sexual orientation may suggest increased risk for HIV infection, the goal is to test all individuals in this age range

A 54-year-old patient is on the surgical unit after a radical abdominal hysterectomy. Which finding is most important to report to the health care provider? a. Urine output of 125 mL in the first 8 hours after surgery b. Decreased bowel sounds in all four abdominal quadrants c. One-inch area of bloody drainage on the abdominal dressing d. Complaints of abdominal pain at the incision site with coughing

ANS: A The decreased urine output indicates possible low blood volume and further assessment is needed to assess for possible internal bleeding. Decreased bowel sounds, minor drainage on the dressing, and abdominal pain with coughing are expected after this surgery

There is one opening in the schedule at the dermatology clinic, and 4 patients are seeking appointments today. Which patient will the nurse schedule for the available opening? a. 38-year old with a 7-mm nevus on the face that has recently become darker b. 62-year-old with multiple small, soft, pedunculated papules in both axillary areas c. 42-year-old with complaints of itching after using topical fluorouracil on the nose d. 50-year-old with concerns about skin redness after having a chemical peel 3 days ago

ANS: A The description of the lesion is consistent with possible malignant melanoma. This patient should be assessed as soon as possible by the health care provider. Itching is common after using topical fluorouracil and redness is an expected finding a few days after a chemical peel. Skin tags are common, benign lesions after midlife

To evaluate the effectiveness of antiretroviral therapy (ART), which laboratory test result will the nurse review? a. Viral load testing b. Enzyme immunoassay c. Rapid HIV antibody testing d. Immunofluorescence assay

ANS: A The effectiveness of ART is measured by the decrease in the amount of virus detectable in the blood. The other tests are used to detect HIV antibodies, which remain positive even with effective ART

Which data collected by the nurse caring for a patient who has cardiogenic shock indicate that the patient may be developing multiple organ dysfunction syndrome (MODS)? a. The patient's serum creatinine level is elevated. b. The patient complains of intermittent chest pressure. c. The patient's extremities are cool and pulses are weak. d. The patient has bilateral crackles throughout lung fields.

ANS: A The elevated serum creatinine level indicates that the patient has renal failure as well as heart failure. The crackles, chest pressure, and cool extremities are all consistent with the patient's diagnosis of cardiogenic shock

A 32-year-old woman brought to the emergency department reports being sexually assaulted. The patient is confused about where she is and she has a large laceration above the right eye. Which action should the nurse take first? a. Assess the patient's neurologic status. b. Assist the patient to remove her clothing. c. Contact the sexual assault nurse examiner (SANE). d. Ask the patient to describe what occurred during the assault.

ANS: A The first priority is to treat urgent medical problems associated with the sexual assault. The patient's head injury may be associated with a head trauma such as a skull fracture or subdural hematoma. Therefore her neurologic status should be assessed first. The other nursing actions are also appropriate, but they are not as high in priority as assessment and treatment for acute physiologic injury

A patient with respiratory failure has arterial pressure-based cardiac output (APCO) monitoring and is receiving mechanical ventilation with peak end-expiratory pressure (PEEP) of 12 cm H2O. Which information indicates that a change in the ventilator settings may be required? a. The arterial pressure is 90/46. b. The heart rate is 58 beats/minute. c. The stroke volume is increased. d. The stroke volume variation is 12%.

ANS: A The hypotension suggests that the high intrathoracic pressure caused by the PEEP may be decreasing venous return and (potentially) cardiac output. The other assessment data would not be a direct result of PEEP and mechanical ventilation

A patient who is orally intubated and receiving mechanical ventilation is anxious and is "fighting" the ventilator. Which action should the nurse take next? a. Verbally coach the patient to breathe with the ventilator. b. Sedate the patient with the ordered PRN lorazepam (Ativan). c. Manually ventilate the patient with a bag-valve-mask device. d. Increase the rate for the ordered propofol (Diprivan) infusion.

ANS: A The initial response by the nurse should be to try to decrease the patient's anxiety by coaching the patient about how to coordinate respirations with the ventilator. The other actions may also be helpful if the verbal coaching is ineffective in reducing the patient's anxiety.

A patient who has received allergen testing using the cutaneous scratch method has developed itching and swelling at the skin site. Which action should the nurse take first? a. Administer epinephrine. b. Apply topical hydrocortisone. c. Monitor the patient for lower extremity edema. d. Ask the patient about exposure to any new lotions or soaps.

ANS: A The initial symptoms of anaphylaxis are itching and edema at the site of the exposure. Hypotension, tachycardia, dilated pupils, and wheezes occur later. Rapid administration of epinephrine when excessive itching or swelling at the skin site is observed can prevent the progression to anaphylaxis. Topical hydrocortisone would not deter an anaphylactic reaction. Exposure to lotions and soaps does not address the immediate concern of a possible anaphylactic reaction. The nurse should not wait and observe for edema. The nurse should act immediately in order to prevent progression to anaphylaxis

An older adult who takes medications for coronary artery disease has just been diagnosed with asymptomatic chronic human immunodeficiency virus (HIV) infection. Which information will the nurse include in patient teaching? a. Many medications have interactions with antiretroviral drugs. b. Less frequent CD4+ level monitoring is needed in older adults. c. Hospice care is available for patients with terminal HIV infection. d. Progression of HIV infection occurs more rapidly in older patients.

ANS: A The nurse will teach the patient about potential interactions between antiretrovirals and the medications that the patient is using for chronic health problems. Treatment and monitoring of HIV infection is not affected by age. A patient with asymptomatic HIV infection is not a candidate for hospice. Progression of HIV is not affected by age, although it may be affected by chronic disease

A patient complains of insomnia and daytime fatigue. What is the first action the nurse should take in addressing the patient's concerns? a. Question the patient about the use of over-the-counter (OTC) sleep aids. b. Suggest that the patient decrease intake of caffeine-containing beverages. c. Advise the patient to get out of bed if unable to fall asleep in 10 to 20 minutes. d. Recommend that the patient use any prescribed sleep aids for only 2 to 3 weeks.

ANS: A The nurse's first action should be assessment of the patient for factors that may contribute to poor sleep quality or daytime fatigue such as the use of OTC medications. The other actions may be appropriate, but assessment is needed first to choose appropriate interventions to improve the patient's sleep.

The nurse admits a terminally ill patient to the hospital. What is the first action that the nurse should complete when planning this patient's care? a. Determine the patient's wishes regarding end-of-life care. b. Emphasize the importance of addressing any family issues. c. Discuss the normal grief process with the patient and family. d. Encourage the patient to talk about any fears or unresolved issues.

ANS: A The nurse's initial action should be to assess the patient's wishes at this time. The other actions may be implemented if the patient or the family express a desire to discuss fears, understand the grief process, or address family issues, but they should not be implemented until the assessment indicates that they are appropriate

A nurse is caring for an obese patient with right lower lobe pneumonia. Which position will be best to improve gas exchange? a. On the left side b. On the right side c. In the tripod position d. In the high-Fowler's position

ANS: A The patient should be positioned with the "good" lung in the dependent position to improve the match between ventilation and perfusion. The obese patient's abdomen will limit respiratory excursion when sitting in the high-Fowler's or tripod positions

A patient with sleep apnea who uses a continuous positive airway pressure (CPAP) device is preparing to have inpatient surgery. Which instructions should the nurse provide to the patient? a. Remind the patient to take the CPAP device to the hospital. b. Plan to schedule a nighttime polysomnography (PSG) study before surgery. c. Discourage the patient from requesting pain medication while hospitalized. d. Call the hospital to ensure that mechanical ventilation will be available for the patient.

ANS: A The patient should be told to take the CPAP device to the hospital if an overnight stay is expected. Many patients will be able to use their own CPAP equipment, but hospital policy should be checked to make sure it can be used. Patients should be treated for pain and monitored for respiratory depression. Another PSG is not required before surgery. There is no need to call the hospital if the patient takes the CPAP device to the hospital

Several patients call the urology clinic requesting appointments with the health care provider as soon as possible. Which patient will the nurse schedule to be seen first? a. 22-year-old who has noticed a firm, nontender lump on his scrotum b. 35-year-old who is concerned that his scrotum "feels like a bag of worms" c. 40-year-old who has pelvic pain while being treated for chronic prostatitis d. 70-year-old who is reporting frequent urinary dribbling after a prostatectomy

ANS: A The patient's age and symptoms suggest possible testicular cancer. Some forms of testicular cancer can be very aggressive, so the patient should be evaluated by the health care provider as soon as possible. Varicoceles do require treatment, but not emergently. Ongoing pelvic pain is common with chronic prostatitis. Urinary dribbling is a common problem after prostatectomy

The following male patients recently arrived in the emergency department. Which one should the nurse assess first? a. 19-year-old who is complaining of severe scrotal pain b. 60-year-old with a nontender ulceration of the glans penis c. 22-year-old who has purulent urethral drainage and back pain d. 64-year-old who has dysuria after brachytherapy for prostate cancer

ANS: A The patient's age and symptoms suggest possible testicular torsion, which will require rapid treatment in order to prevent testicular necrosis. The other patients also require assessment by the nurse, but their history and symptoms indicate nonemergent problems (acute prostatitis, cancer of the penis, and radiation-associated urinary tract irritation)

A 32-year-old patient who has had an open reduction and internal fixation (ORIF) of left lower leg fractures continues to complain of severe pain in the leg 15 minutes after receiving the prescribed IV morphine. Pulses are faintly palpable and the foot is cool. Which action should the nurse take next? a. Notify the health care provider. b. Assess the incision for redness. c. Reposition the left leg on pillows. d. Check the patient's blood pressure.

ANS: A The patient's clinical manifestations suggest compartment syndrome and delay in diagnosis and treatment may lead to severe functional impairment. The data do not suggest problems with blood pressure or infection. Elevation of the leg will decrease arterial flow and further reduce perfusion

A nurse is caring for a patient with shock of unknown etiology whose hemodynamic monitoring indicates BP 92/54, pulse 64, and an elevated pulmonary artery wedge pressure. Which collaborative intervention ordered by the health care provider should the nurse question? a. Infuse normal saline at 250 mL/hr. b. Keep head of bed elevated to 30 degrees. c. Hold nitroprusside (Nipride) if systolic BP <90 mm Hg. d. Titrate dobutamine (Dobutrex) to keep systolic BP >90 mm Hg.

ANS: A The patient's elevated pulmonary artery wedge pressure indicates volume excess. A saline infusion at 250 mL/hr will exacerbate the volume excess. The other actions are appropriate for the patient

Ten days after receiving a bone marrow transplant, a patient develops a skin rash. What would the nurse suspect is the cause of this patient's skin rash? a. The donor T cells are attacking the patient's skin cells. b. The patient's antibodies are rejecting the donor bone marrow. c. The patient is experiencing a delayed hypersensitivity reaction. d. The patient will need treatment to prevent hyperacute rejection.

ANS: A The patient's history and symptoms indicate that the patient is experiencing graft-versus-host disease, in which the donated T cells attack the patient's tissues. The history and symptoms are not consistent with rejection or delayed hypersensitivity

The nurse is caring for a 1-day postoperative patient who is receiving morphine through patient-controlled analgesia (PCA). What action by the nurse is a priority? a. Check the respiratory rate. b. Assess for nausea after eating. c. Inspect the abdomen and auscultate bowel sounds. d. Evaluate the sacral and heel areas for signs of redness.

ANS: A The patient's respiratory rate is the highest priority of care while using PCA medication because of the possible respiratory depression. The other information may also require intervention but is not as urgent to report as the respiratory rate

A 19-year-old patient has genital warts around her external genitalia and perianal area. She tells the nurse that she has not sought treatment until now because "the warts are so disgusting." Which nursing diagnosis is most appropriate? a. Disturbed body image related to feelings about the genital warts b. Ineffective coping related to denial of increased risk for infection c. Risk for infection related to lack of knowledge about transmission d. Anxiety related to impact of condition on interpersonal relationships

ANS: A The patient's statement that her lesions are disgusting suggests that disturbed body image is the major concern. There is no evidence to indicate ineffective coping or lack of knowledge about mode of transmission. The patient may be experiencing anxiety, but there is nothing in the data indicating that the genital warts are impacting interpersonal relationships.

A 28-year-old patient reports anxiety, headaches with dizziness, and abdominal bloating occurring before her menstrual periods. Which action is best for the nurse to take at this time? a. Ask the patient to keep track of her symptoms in a diary for 3 months. b. Suggest that the patient try aerobic exercise to decrease her symptoms. c. Teach the patient about appropriate lifestyle changes to reduce premenstrual syndrome (PMS) symptoms. d. Advise the patient to use nonsteroidal antiinflammatory drugs (NSAIDs) such as ibuprofen (Advil) to control symptoms.

ANS: A The patient's symptoms indicate possible PMS, but they also may be associated with other diagnoses. Having the patient keep a symptom diary for 2 or 3 months will help in confirming a diagnosis of PMS. The nurse should not implement interventions for PMS until a diagnosis is made

Which finding from a patient's right knee arthrocentesis will be of concern to the nurse? a. Cloudy fluid b. Scant thin fluid c. Pale yellow fluid d. Straw-colored fluid

ANS: A The presence of purulent fluid suggests a possible joint infection. Normal synovial fluid is scant in amount and pale yellow/straw-colored

An older adult patient who is having an annual check-up tells the nurse, "I feel fine, and I don't want to pay for all these unnecessary cancer screening tests!" Which information should the nurse plan to teach this patient? a. Consequences of aging on cell-mediated immunity b. Decrease in antibody production associated with aging c. Impact of poor nutrition on immune function in older people d. Incidence of cancer-stimulating infections in older individuals

ANS: A The primary impact of aging on immune function is on T cells, which are important for immune surveillance and tumor immunity. Antibody function is not affected as much by aging. Poor nutrition can also contribute to decreased immunity, but there is no evidence that it is a contributing factor for this patient. Although some types of cancer are associated with specific infections, this patient does not have an active infection

The nurse cares for a terminally ill patient who is experiencing pain that is continuous and severe. How should the nurse schedule the administration of opioid pain medications? a. Give around-the-clock routine administration of analgesics. b. Provide PRN doses of medication whenever the patient requests. c. Offer enough pain medication to keep the patient sedated and unaware of stimuli. d. Suggest analgesic doses that provide pain control without decreasing respiratory rate.

ANS: A The principles of beneficence and nonmaleficence indicate that the goal of pain management in a terminally ill patient is adequate pain relief even if the effect of pain medications could hasten death. Administration of analgesics on a PRN basis will not provide the consistent level of analgesia the patient needs. Patients usually do not require so much pain medication that they are oversedated and unaware of stimuli. Adequate pain relief may require a dosage that will result in a decrease in respiratory rate.

The spouse of a patient with terminal cancer visits daily and cheerfully talks with the patient about wedding anniversary plans for the next year. When the nurse asks about any concerns, the spouse says, "I'm busy at work, but otherwise things are fine." Which nursing diagnosis is most appropriate? a. Ineffective coping related to lack of grieving b. Anxiety related to complicated grieving process c. Caregiver role strain related to feeling overwhelmed d. Hopelessness related to knowledge deficit about cancer

ANS: A The spouse's behavior and statements indicate the absence of anticipatory grieving, which may lead to impaired adjustment as the patient progresses toward death. The spouse does not appear to feel overwhelmed, hopeless, or anxious

Which information will the nurse plan to include when teaching a community health group about testicular self-examination? a. Testicular self-examination should be done in a warm room. b. The only structure normally felt in the scrotal sac is the testis. c. Testicular self-examination should be done at least every week. d. Call the health care provider if one testis is larger than the other.

ANS: A The testes will hang lower in the scrotum when the temperature is warm (e.g., during a shower), and it will be easier to palpate. The epididymis is also normally palpable in the scrotum. One testis is normally larger. The patient should perform testicular self-examination monthly

The health care provider diagnoses impetigo in a patient who has crusty vesicopustular lesions on the lower face. Which instructions should the nurse include in the teaching plan? a. Clean the infected areas with soap and water. b. Apply alcohol-based cleansers on the lesions. c. Avoid use of antibiotic ointments on the lesions. d. Use petroleum jelly (Vaseline) to soften crusty areas.

ANS: A The treatment for impetigo includes softening of the crusts with warm saline soaks and then soap-and-water removal. Alcohol-based cleansers and use of petroleum jelly are not recommended for impetigo. Antibiotic ointments, such as mupirocin (Bactroban), may be applied to the lesions

A patient with atopic dermatitis has been using a high-potency topical corticosteroid ointment for several weeks. The nurse should assess for which adverse effect? a. Thinning of the affected skin b. Alopecia of the affected areas c. Reddish-brown discoloration of the skin d. Dryness and scaling in the areas of treatment

ANS: A Thinning of the skin indicates that atrophy, a possible adverse effect of topical corticosteroids, is occurring. The health care provider should be notified so that the medication can be changed or tapered. Alopecia, red-brown discoloration, and dryness/scaling of the skin are not adverse effects of topical corticosteroid use

The nurse is assessing the sexual-reproductive functional health pattern of a 32-year-old woman. Which question is most useful in determining the patient's sexual orientation and risk factors? a. "Do you have sex with men, women, or both?" b. "Which gender do you prefer to have sex with?" c. "What types of sexual activities do you prefer?" d. "Are you heterosexual, homosexual, or bisexual?"

ANS: A This question is the most simply stated and will increase the likelihood of obtaining the relevant information about sexual orientation and possible risk factors associated with sexual activity. A patient who prefers sex with women may also have intercourse at times with men. The types of sexual activities engaged in may not indicate sexual orientation. Many patients who have sex with both men and women do not identify themselves as homosexual or bisexual.

A widowed mother of four school-age children is hospitalized with metastatic ovarian cancer. The patient is crying and tells the nurse that she does not know what will happen to her children when she dies. Which response by the nurse is most appropriate? a. "Why don't we talk about the options you have for the care of your children?" b. "I'm sure you have friends that will take the children when you can't care for them." c. "For now you need to concentrate on getting well and not worrying about your children." d. "Many patients with cancer live for a long time, so there is still time to plan for your children."

ANS: A This response expresses the nurse's willingness to listen and recognizes the patient's concern. The responses beginning "Many patients with cancer live for a long time" and "For now you need to concentrate on getting well" close off discussion of the topic and indicate that the nurse is uncomfortable with the topic. In addition, the patient with metastatic ovarian cancer may not have a long time to plan. Although it is possible that the patient's friends will take the children, more assessment information is needed before making plans

Which topic will the nurse include in the preoperative teaching for a patient admitted for an abdominal hysterectomy? a. Purpose of ambulation and leg exercises b. Adverse effects of systemic chemotherapy c. Decrease in vaginal sensation after surgery d. Symptoms caused by the drop in estrogen level

ANS: A Venous thromboembolism (VTE) is a potential complication after the surgery, and the nurse will instruct the patient about ways to prevent it. Vaginal sensation is decreased after a vaginal hysterectomy but not after abdominal hysterectomy. Leiomyomas are benign tumors, so chemotherapy and radiation will not be prescribed. Because the patient will still have her ovaries, the estrogen level will not decrease

The nurse cares for a patient infected with human immunodeficiency virus (HIV) who has just been diagnosed with asymptomatic chronic HIV infection. Which prophylactic measures will the nurse include in the plan of care (select all that apply)? a. Hepatitis B vaccine b. Pneumococcal vaccine c. Influenza virus vaccine d. Trimethoprim-sulfamethoxazole e. Varicella zoster immune globulin

ANS: A, B, C Asymptomatic chronic HIV infection is a stage between acute HIV infection and a diagnosis of symptomatic chronic HIV infection. Although called asymptomatic, symptoms (e.g., fatigue, headache, low-grade fever, night sweats) often occur. Prevention of other infections is an important intervention in patients who are HIV positive, and these vaccines are recommended as soon as the HIV infection is diagnosed. Antibiotics and immune globulin are used to prevent and treat infections that occur later in the course of the disease when the CD4+ counts have dropped or when infection has occurred

Which preventive actions by the nurse will help limit the development of systemic inflammatory response syndrome (SIRS) in patients admitted to the hospital (select all that apply)? a. Use aseptic technique when caring for invasive lines or devices. b. Ambulate postoperative patients as soon as possible after surgery. c. Remove indwelling urinary catheters as soon as possible after surgery. d. Advocate for parenteral nutrition for patients who cannot take oral feedings. e. Administer prescribed antibiotics within 1 hour for patients with possible sepsis.

ANS: A, B, C, E Because sepsis is the most frequent etiology for SIRS, measures to avoid infection such as removing indwelling urinary catheters as soon as possible, use of aseptic technique, and early ambulation should be included in the plan of care. Adequate nutrition is important in preventing SIRS. Enteral, rather than parenteral, nutrition is preferred when patients are unable to take oral feedings because enteral nutrition helps maintain the integrity of the intestine, thus decreasing infection risk. Antibiotics should be administered within 1 hour after being prescribed to decrease the risk of sepsis progressing to SIRS

Which nonhormonal therapies will the nurse suggest for a healthy perimenopausal woman who prefers not to use hormone therapy (HT) (select all that apply)? a. Reduce coffee intake. b. Exercise several times a week. c. Take black cohosh supplements. d. Have a glass of wine in the evening. e. Increase intake of dietary soy products.

ANS: A, B, C, E Reduction in caffeine intake, use of black cohosh, increasing dietary soy intake, and exercising three to four times weekly are recommended to reduce symptoms associated with menopause. Alcohol intake in the evening may increase the sleep problems associated with menopause

The nurse in the outpatient clinic notes that the following patients have not received the human papillomavirus (HPV) vaccine. Which patients should the nurse plan to teach about benefits of the vaccine (select all that apply)? a. 24-year-old man who has a history of genital warts b. 18-year-old man who has had one male sexual partner c. 28-year-old woman who has never been sexually active d. 20-year-old woman who has a newly diagnosed Chlamydia infection e. 30-year-old woman whose sexual partner has a history of genital warts

ANS: A, B, D The HPV vaccines are recommended for male and female patients between ages 9 through 26. Ideally, the vaccines are administered before patients are sexually active, but they offer benefit even to those who already have HPV infection

A patient with suspected neurogenic shock after a diving accident has arrived in the emergency department. A cervical collar is in place. Which actions should the nurse take (select all that apply)? a. Prepare to administer atropine IV. b. Obtain baseline body temperature. c. Infuse large volumes of lactated Ringer's solution. d. Provide high-flow oxygen (100%) by non-rebreather mask. e. Prepare for emergent intubation and mechanical ventilation.

ANS: A, B, D, E All of the actions are appropriate except to give large volumes of lactated Ringer's solution. The patient with neurogenic shock usually has a normal blood volume, and it is important not to volume overload the patient. In addition, lactated Ringer's solution is used cautiously in all shock situations because the failing liver cannot convert lactate to bicarbonate

The nurse plans a presentation for community members about how to decrease the risk for antibiotic-resistant infections. Which information will the nurse include in the teaching plan (select all that apply)? a. Continue taking antibiotics until all the medication is gone. b. Antibiotics may sometimes be prescribed to prevent infection. c. Unused antibiotics that are more than a year old should be discarded. d. Antibiotics are effective in treating influenza associated with high fevers. e. Hand washing is effective in preventing many viral and bacterial infections.

ANS: A, B, E All prescribed doses of antibiotics should be taken. In some situations, such as before surgery, antibiotics are prescribed to prevent infection. There should not be any leftover antibiotics because all prescribed doses should be taken. However, if there are leftover antibiotics, they should be discarded immediately because the number left will not be enough to treat a future infection. Hand washing is generally considered the single most effective action in decreasing infection transmission. Antibiotics are ineffective in treating viral infections such as influenza

A nurse is teaching a patient with contact dermatitis of the arms and legs about ways to decrease pruritus. Which information should the nurse include in the teaching plan (select all that apply)? a. Cool, wet cloths or dressings can be used to reduce itching. b. Take cool or tepid baths several times daily to decrease itching. c. Add oil to your bath water to aid in moisturizing the affected skin. d. Rub yourself dry with a towel after bathing to prevent skin maceration. e. Use of an over-the-counter (OTC) antihistamine can reduce scratching.

ANS: A, B, E Cool or tepid baths, cool dressings, and OTC antihistamines all help reduce pruritus and scratching. Adding oil to bath water is not recommended because of the increased risk for falls. The patient should use the towel to pat (not rub) the skin dry

Which nursing actions for the care of a dying patient can the nurse delegate to a licensed practical/vocational nurse (LPN/LVN) (select all that apply)? a. Provide postmortem care to the patient. b. Encourage the family members to talk with and reassure the patient. c. Determine how frequently physical assessments are needed for the patient. d. Teach family members about commonly occurring signs of approaching death. e. Administer the prescribed morphine sulfate sublingual as necessary for pain control.

ANS: A, B, E Medication administration, psychosocial care, and postmortem care are included in LPN/LVN education and scope of practice. Patient and family teaching and assessment and planning of frequency for assessments are skills that require registered nurse level education and scope of practice

A nurse assesses a postoperative patient 2 days after chest surgery. What findings indicate that the patient requires better pain management (select all that apply)? a. Confusion b. Hypoglycemia c. Poor cough effort d. Shallow breathing e. Elevated temperature

ANS: A, C, D, E Inadequate pain control can decrease tidal volume and cough effort, leading to complications such as pneumonia with increases in temperature. Poor pain control may lead to confusion through a variety of mechanism, including hypoventilation and poor sleep quality. Stressors such as pain cause increased release of corticosteroids that can result in hyperglycemia.

The nurse at the clinic is interviewing a 64-year-old woman who is 5 feet, 3 inches tall and weighs 125 pounds (57 kg). The patient has not seen a health care provider for 20 years. She walks 5 miles most days and has a glass of wine 2 or 3 times a week. Which topics will the nurse plan to include in patient teaching about cancer screening and decreasing cancer risk (select all that apply)? a. Pap testing b. Tobacco use c. Sunscreen use d. Mammography e. Colorectal screening

ANS: A, C, D, E The patient's age, gender, and history indicate a need for screening and/or teaching about colorectal cancer, mammography, Pap smears, and sunscreen. The patient does not use excessive alcohol or tobacco, she is physically active, and her body weight is healthy

A patient develops neutropenia after receiving chemotherapy. Which information about ways to prevent infection will the nurse include in the teaching plan (select all that apply)? a. Cook food thoroughly before eating. b. Choose low fiber, low residue foods. c. Avoid public transportation such as buses. d. Use rectal suppositories if needed for constipation. e. Talk to the oncologist before having any dental work done.

ANS: A, C, E Eating only cooked food and avoiding public transportation will decrease infection risk. A high-fiber diet is recommended for neutropenic patients to decrease constipation. Because bacteria may enter the circulation during dental work or oral surgery, the patient may need to postpone dental work or take antibiotics

Which actions will the nurse include in the plan of care when caring for a patient with metastatic bone cancer of the left femur (select all that apply)? a. Monitor serum calcium level. b. Teach about the need for strict bed rest. c. Avoid use of sustained-release opioids for pain. d. Support the left leg when repositioning the patient. e. Support family as they discuss the prognosis of patient

ANS: A, D, E The nurse will monitor for hypercalcemia caused by bone decalcification. Support of the leg helps reduce the risk for pathologic fractures. Although the patient may be reluctant to exercise, activity is important to maintain function and avoid the complications associated with immobility. Adequate pain medication, including sustained-release and rapidly acting opioids, is needed for the severe pain that is frequently associated with bone cancer. The prognosis for metastatic bone cancer is poor so the patient and family need to be supported as they deal with the reality of the situation.

A 22-year-old man tells the nurse at the health clinic that he has recently had some problems with erectile dysfunction. Which question should the nurse ask first to assess for possible etiologic factors? a. "Do you experience an unusual amount of stress?" b. "Do you use any recreational drugs or drink alcohol?" c. "Do you have chronic cardiovascular or peripheral vascular disease?" d. "Do you have a history of an erection that lasted for 6 hours or more?"

ANS: B A common etiologic factor for erectile dysfunction (ED) in younger men is use of recreational drugs or alcohol. Stress, priapism, and cardiovascular illness also contribute to ED, but they are not common etiologic factors in younger men

The health care provider prescribes finasteride (Proscar) for a 67-year-old patient who has benign prostatic hyperplasia (BPH). When teaching the patient about the drug, the nurse informs him that a. he should change position from lying to standing slowly to avoid dizziness. b. his interest in sexual activity may decrease while he is taking the medication. c. improvement in the obstructive symptoms should occur within about 2 weeks. d. he will need to monitor his blood pressure frequently to assess for hypertension.

ANS: B A decrease in libido is a side effect of finasteride because of the androgen suppression that occurs with the drug. Although orthostatic hypotension may occur if the patient is also taking a medication for erectile dysfunction (ED), it should not occur with finasteride alone. Improvement in symptoms of obstruction takes about 6 months. The medication does not cause hypertension

Which information in a 67-year-old woman's health history will alert the nurse to the need for a more focused assessment of the musculoskeletal system? a. The patient sprained her ankle at age 13. b. The patient's mother became shorter with aging. c. The patient takes ibuprofen (Advil) for occasional headaches. d. The patient's father died of complications of miliary tuberculosis.

ANS: B A family history of height loss with aging may indicate osteoporosis, and the nurse should perform a more thorough assessment of the patient's current height and other risk factors for osteoporosis. A sprained ankle during adolescence does not place the patient at increased current risk for musculoskeletal problems. A family history of tuberculosis is not a risk factor. Occasional nonsteroidal antiinflammatory drug (NSAID) use does not indicate any increased musculoskeletal risk.

Following surgery for an abdominal aortic aneurysm, a patient's central venous pressure (CVP) monitor indicates low pressures. Which action is a priority for the nurse to take? a. Administer IV diuretic medications. b. Increase the IV fluid infusion per protocol. c. Document the CVP and continue to monitor. d. Elevate the head of the patient's bed to 45 degrees.

ANS: B A low CVP indicates hypovolemia and a need for an increase in the infusion rate. Diuretic administration will contribute to hypovolemia and elevation of the head may decrease cerebral perfusion. Documentation and continued monitoring is an inadequate response to the low CVP

The registered nurse (RN) caring for an HIV-positive patient admitted with tuberculosis can delegate which action to unlicensed assistive personnel (UAP)? a. Teach the patient about how to use tissues to dispose of respiratory secretions. b. Stock the patient's room with all the necessary personal protective equipment. c. Interview the patient to obtain the names of family members and close contacts. d. Tell the patient's family members the reason for the use of airborne precautions.

ANS: B A patient diagnosed with tuberculosis would be placed on airborne precautions. Because all health care workers are taught about the various types of infection precautions used in the hospital, the UAP can safely stock the room with personal protective equipment. Obtaining contact information and patient teaching are higher-level skills that require RN education and scope of practice

A hospice patient is manifesting a decrease in all body system functions except for a heart rate of 124 and a respiratory rate of 28. Which statement, if made by the nurse to the patient's family member, is most appropriate? a. "These symptoms will continue to increase until death finally occurs." b. "These symptoms are a normal response before these functions decrease." c. "These symptoms indicate a reflex response to the slowing of other body systems." d. "These symptoms may be associated with an improvement in the patient's condition."

ANS: B An increase in heart and respiratory rate may occur before the slowing of these functions in the dying patient. Heart and respiratory rate typically slow as the patient progresses further toward death. In a dying patient, high respiratory and pulse rates do not indicate improvement, and it would be inappropriate for the nurse to indicate this to the family. The changes in pulse and respirations are not reflex responses

Which nursing intervention will be included in the plan of care after a patient with a right femur fracture has a hip spica cast applied? a. Avoid placing the patient in prone position. b. Ask the patient about abdominal discomfort. c. Discuss remaining on bed rest for several weeks. d. Use the cast support bar to reposition the patient.

ANS: B Assessment of bowel sounds, abdominal pain, and nausea and vomiting will detect the development of cast syndrome. To avoid breakage, the support bar should not be used for repositioning. After the cast dries, the patient can begin ambulating with the assistance of physical therapy personnel and may be turned to the prone position

Which finding is the best indicator that the fluid resuscitation for a patient with hypovolemic shock has been effective? a. Hemoglobin is within normal limits. b. Urine output is 60 mL over the last hour. c. Central venous pressure (CVP) is normal. d. Mean arterial pressure (MAP) is 72 mm Hg.

ANS: B Assessment of end organ perfusion, such as an adequate urine output, is the best indicator that fluid resuscitation has been successful. The hemoglobin level, CVP, and MAP are useful in determining the effects of fluid administration, but they are not as useful as data indicating good organ perfusion

A 31-year-old patient who has been diagnosed with human papillomavirus (HPV) infection gives a health history that includes smoking tobacco, taking oral contraceptives, and having been treated twice for vaginal candidiasis. Which topic will the nurse include in patient teaching? a. Use of water-soluble lubricants b. Risk factors for cervical cancer c. Antifungal cream administration d. Possible difficulties with conception

ANS: B Because HPV infection and smoking are both associated with increased cervical cancer risk, the nurse should emphasize the importance of avoiding smoking. An HPV infection does not decrease vaginal lubrication, decrease ability to conceive, or require the use of antifungal creams.

A 24-year-old female says she wants to begin using oral contraceptives. Which information from the nursing assessment is most important to report to the health care provider? a. The patient quit smoking 5 months previously. b. The patient's blood pressure is 154/86 mm Hg. c. The patient has not been vaccinated for rubella. d. The patient has chronic iron-deficiency anemia.

ANS: B Because hypertension increases the risk for morbidity and mortality in women taking oral contraceptives, the patient's blood pressure should be controlled before oral contraceptives are prescribed. The other information also will be reported but will not affect the choice of contraceptive

A 25-year-old woman has an induced abortion with suction curettage at an ambulatory surgical center. Which instructions will the nurse include when discharging the patient? a. "Heavy vaginal bleeding is expected for about 2 weeks." b. "You should abstain from sexual intercourse for 2 weeks." c. "Contraceptives should be avoided until your reexamination." d. "Irregular menstrual periods are expected for the next few months."

ANS: B Because infection is a possible complication of this procedure, the patient is advised to avoid intercourse until the reexamination in 2 weeks. Patients may be started on contraceptives on the day of the procedure. The patient should call the doctor if heavy vaginal bleeding occurs. No change in the regularity of the menstrual periods is expected

Which question asked by the nurse will give the most information about the patient's metastatic bone cancer pain? a. "How long have you had this pain?" b. "How would you describe your pain?" c. "How much medication do you take for the pain?" d. "How many times a day do you take medication for the pain?"

ANS: B Because pain is a multidimensional experience, asking a question that addresses the patient's experience with the pain will elicit more information than the more specific information asked in the other three responses. All of these questions are appropriate, but the response beginning "How would you describe your pain?" is the best initial question

The nurse supervises the care of a patient with a temporary radioactive cervical implant. Which action by unlicensed assistive personnel (UAP), if observed by the nurse, would require an intervention? a. The UAP flushes the toilet once after emptying the patient's bedpan. b. The UAP stands by the patient's bed for 30 minutes talking with the patient. c. The UAP places the patient's bedding in the laundry container in the hallway. d. The UAP gives the patient an alcohol-containing mouthwash to use for oral care.

ANS: B Because patients with temporary implants emit radioactivity while the implants are in place, exposure to the patient is limited. Laundry and urine/feces do not have any radioactivity and do not require special precautions. Cervical radiation will not affect the oral mucosa, and alcohol-based mouthwash is not contraindicated

A 58-year-old man with erectile dysfunction (ED) tells the nurse he is interested in using sildenafil (Viagra). Which action should the nurse take first? a. Assure the patient that ED is common with aging. b. Ask the patient about any prescription drugs he is taking. c. Tell the patient that Viagra does not always work for ED. d. Discuss the common adverse effects of erectogenic drugs.

ANS: B Because some medications can cause ED and patients using nitrates should not take sildenafil, the nurse should first assess for prescription drug use. The nurse may want to teach the patient about realistic expectations and adverse effects of sildenafil therapy, but this should not be the first action. Although ED does increase with aging, it may be secondary to medication use or cardiovascular disease

A 23-year-old patient with a history of muscular dystrophy is hospitalized with pneumonia. Which nursing action will be included in the plan of care? a. Logroll the patient every 2 hours. b. Assist the patient with ambulation. c. Discuss the need for genetic testing with the patient. d. Teach the patient about the muscle biopsy procedure.

ANS: B Because the goal for the patient with muscular dystrophy is to keep the patient active for as long as possible, assisting the patient to ambulate will be part of the care plan. The patient will not require logrolling. Muscle biopsies are necessary to confirm the diagnosis but are not necessary for a patient who already has a diagnosis. There is no need for genetic testing because the patient already knows the diagnosis

A 32-year-old who was admitted to the emergency department with severe abdominal pain is diagnosed with an ectopic pregnancy. The patient begins to cry and asks the nurse to leave her alone to grieve. Which action should the nurse take next? a. Stay with the patient and encourage her to discuss her feelings. b. Explain the reason for taking vital signs every 15 to 30 minutes. c. Close the door to the patient's room and minimize disturbances. d. Provide teaching about options for termination of the pregnancy.

ANS: B Because the patient is at risk for rupture of the fallopian tube and hemorrhage, frequent monitoring of vital signs is needed. The patient has asked to be left alone, so staying with her and encouraging her to discuss her feelings are inappropriate actions. Minimizing contact with her and closing the door of the room is unsafe because of the risk for hemorrhage. Because the patient has requested time to grieve, it would be inappropriate to provide teaching about options for pregnancy termination

A patient admitted with acute respiratory failure has a nursing diagnosis of ineffective airway clearance related to thick, secretions. Which action is a priority for the nurse to include in the plan of care? a. Encourage use of the incentive spirometer. b. Offer the patient fluids at frequent intervals. c. Teach the patient the importance of ambulation. d. Titrate oxygen level to keep O2 saturation >93%.

ANS: B Because the reason for the poor airway clearance is the thick secretions, the best action will be to encourage the patient to improve oral fluid intake. Patients should be instructed to use the incentive spirometer on a regular basis (e.g., every hour) in order to facilitate the clearance of the secretions. The other actions may also be helpful in improving the patient's gas exchange, but they do not address the thick secretions that are causing the poor airway clearance

A 22-year-old patient with gonorrhea is treated with a single IM dose of ceftriaxone (Rocephin) and is given a prescription for doxycycline (Vibramycin) 100 mg bid for 7 days. The nurse explains to the patient that this combination of antibiotics is prescribed to a. prevent reinfection during treatment. b. treat any coexisting chlamydial infection. c. eradicate resistant strains of N. gonorrhoeae. d. prevent the development of resistant organisms.

ANS: B Because there is a high incidence of co-infection with gonorrhea and chlamydia, patients are usually treated for both. The other explanations about the purpose of the antibiotic combination are not accurate

The nurse will instruct the patient with a fractured left radius that the cast will need to remain in place a. for several months. b. for at least 3 weeks. c. until swelling of the wrist has resolved. d. until x-rays show complete bony union.

ANS: B Bone healing starts immediately after the injury, but since ossification does not begin until 3 weeks postinjury, the cast will need to be worn for at least 3 weeks. Complete union may take up to a year. Resolution of swelling does not indicate bone healing

Which action will the nurse take in order to evaluate the effectiveness of Buck's traction for a 62-year-old patient who has an intracapsular fracture of the right femur? a. Check peripheral pulses. b. Ask about hip pain level. c. Assess for hip contractures. d. Monitor for hip dislocation.

ANS: B Buck's traction keeps the leg immobilized and reduces painful muscle spasm. Hip contractures and dislocation are unlikely to occur in this situation. The peripheral pulses will be assessed, but this does not help in evaluating the effectiveness of Buck's traction

A patient with left knee pain is diagnosed with bursitis. The nurse will explain that bursitis is an inflammation of a. the synovial membrane that lines the joint. b. a small, fluid-filled sac found at some joints. c. the fibrocartilage that acts as a shock absorber in the knee joint. d. any connective tissue that is found supporting the joints of the body.

ANS: B Bursae are fluid-filled sacs that cushion joints and bony prominences. Fibrocartilage is a solid tissue that cushions some joints. Bursae are a specific type of connective tissue. The synovial membrane lines many joints but is not a bursa

Which statement by the nurse when explaining the purpose of positive end-expiratory pressure (PEEP) to the family members of a patient with ARDS is accurate? a. "PEEP will push more air into the lungs during inhalation." b. "PEEP prevents the lung air sacs from collapsing during exhalation." c. "PEEP will prevent lung damage while the patient is on the ventilator." d. "PEEP allows the breathing machine to deliver 100% oxygen to the lungs."

ANS: B By preventing alveolar collapse during expiration, PEEP improves gas exchange and oxygenation. PEEP will not prevent lung damage (e.g., fibrotic changes that occur with ARDS), push more air into the lungs, or change the fraction of inspired oxygen (FIO2) delivered to the patient

The nurse provides discharge instructions to a patient who has an immune deficiency involving the T lymphocytes. Which screening should the nurse include in the teaching plan for this patient? a. Screening for allergies b. Screening for malignancy c. Antibody deficiency screening d. Screening for autoimmune disorders

ANS: B Cell-mediated immunity is responsible for the recognition and destruction of cancer cells. Allergic reactions, autoimmune disorders, and antibody deficiencies are mediated primarily by B lymphocytes and humoral immunity

Which information obtained by the nurse about a 29-year-old patient with a lumbar vertebral compression fracture is most important to report to the health care provider? a. Patient refuses to be turned due to back pain. b. Patient has been incontinent of urine and stool. c. Patient reports lumbar area tenderness to palpation. d. Patient frequently uses oral corticosteroids to treat asthma.

ANS: B Changes in bowel or bladder function indicate possible spinal cord compression and should be reported immediately because surgical intervention may be needed. The other findings are also pertinent but are consistent with the patient's diagnosis and do not require immediate intervention

A 68-year-old patient has been in the intensive care unit for 4 days and has a nursing diagnosis of disturbed sensory perception related to sleep deprivation. Which action should the nurse include in the plan of care? a. Administer prescribed sedatives or opioids at bedtime to promote sleep. b. Cluster nursing activities so that the patient has uninterrupted rest periods. c. Silence the alarms on the cardiac monitors to allow 30- to 40-minute naps. d. Eliminate assessments between 0100 and 0600 to allow uninterrupted sleep.

ANS: B Clustering nursing activities and providing uninterrupted rest periods will minimize sleep-cycle disruption. Sedative and opioid medications tend to decrease the amount of rapid eye movement (REM) sleep and can contribute to sleep disturbance and disturbed sensory perception. Silencing the alarms on the cardiac monitors would be unsafe in a critically ill patient, as would discontinuing assessments during the night.

Which action can the nurse delegate to unlicensed assistive personnel (UAP) who are working in the orthopedic clinic? a. Grade leg muscle strength for a patient with back pain. b. Obtain blood sample for uric acid from a patient with gout. c. Perform straight-leg-raise testing for a patient with sciatica. d. Check for knee joint crepitation before arthroscopic surgery.

ANS: B Drawing blood specimens is a common skill performed by UAP in clinic settings. The other actions are assessments and require registered nurse (RN)-level judgment and critical thinking.

A 58-year-old patient who has been recently diagnosed with benign prostatic hyperplasia (BPH) tells the nurse that he does not want to have a transurethral resection of the prostate (TURP) because it might affect his ability to maintain an erection during intercourse. Which action should the nurse take? a. Provide teaching about medications for erectile dysfunction (ED). b. Discuss that TURP does not commonly affect erectile function. c. Offer reassurance that sperm production is not affected by TURP. d. Discuss alternative methods of sexual expression besides intercourse.

ANS: B ED is not a concern with TURP, although retrograde ejaculation is likely and the nurse should discuss this with the patient. Erectile function is not usually affected by a TURP, so the patient will not need information about penile implants or reassurance that other forms of sexual expression may be used. Because the patient has not asked about fertility, reassurance about sperm production does not address his concerns

The central venous oxygen saturation (ScvO2) is decreasing in a patient who has severe pancreatitis. To determine the possible cause of the decreased ScvO2, the nurse assesses the patient's a. lipase. b. temperature. c. urinary output. d. body mass index.

ANS: B Elevated temperature increases metabolic demands and oxygen use by tissues, resulting in a drop in oxygen saturation of central venous blood. Information about the patient's body mass index, urinary output, and lipase will not help in determining the cause of the patient's drop in ScvO2

The following interventions are ordered by the health care provider for a patient who has respiratory distress and syncope after eating strawberries. Which will the nurse complete first? a. Start a normal saline infusion. b. Give epinephrine (Adrenalin). c. Start continuous ECG monitoring. d. Give diphenhydramine (Benadryl).

ANS: B Epinephrine rapidly causes peripheral vasoconstriction, dilates the bronchi, and blocks the effects of histamine and reverses the vasodilation, bronchoconstriction, and histamine release that cause the symptoms of anaphylaxis. The other interventions are also appropriate but would not be the first ones completed

A 22-year-old patient reports her concern about not having a menstrual period for the past 7 months. Which statement by the patient indicates a possible related factor to the amenorrhea? a. "I drink at least 3 glasses of nonfat milk every day." b. "I run 7 to 8 miles every day to keep my weight down." c. "I was treated for a sexually transmitted infection 2 years ago." d. "I am not sexually active but currently I have an IUD."

ANS: B Excessive exercise can cause amenorrhea. The other statements by the patient do not suggest any urgent teaching needs

A 39-year-old patient whose work involves frequent lifting has a history of chronic back pain. After the nurse has taught the patient about correct body mechanics, which patient statement indicates that the teaching has been effective? a. "I will keep my back straight to lift anything higher than my waist." b. "I will begin doing exercises to strengthen the muscles of my back." c. "I can try to sleep with my hips and knees extended to prevent back strain." d. "I can tell my boss that I need to change to a job where I can work at a desk."

ANS: B Exercises can help strengthen the muscles that support the back. Flexion of the hips and knees places less strain on the back. Modifications in the way the patient lifts boxes are needed, but sitting for prolonged periods can aggravate back pain. The patient should not lift above the level of the elbows

A patient admitted with chest pain is also found to have positive Venereal Disease Research Laboratory (VDRL) and fluorescent treponemal antibody absorption (FAT-Abs) tests, rashes on the palms and the soles of the feet, and moist papules in the anal and vulvar area. Which action will the nurse include in the plan of care? a. Assess for arterial aneurysms. b. Wear gloves for patient contact. c. Place the patient in a private room. d. Apply antibiotic ointment to the perineum.

ANS: B Exudate from any lesions with syphilis is highly contagious. Systemic antibiotics, rather than local treatment of lesions, are used to treat syphilis. The patient does not require a private room because the disease is spread through contact with the lesions. This patient has clinical manifestations of secondary syphilis and does not need to be monitored for manifestations of tertiary syphilis

The home health nurse cares for a patient who has been receiving interferon therapy for treatment of cancer. Which statement by the patient indicates a need for further assessment? a. "I have frequent muscle aches and pains." b. "I rarely have the energy to get out of bed." c. "I experience chills after I inject the interferon." d. "I take acetaminophen (Tylenol) every 4 hours."

ANS: B Fatigue can be a dose-limiting toxicity for use of biologic therapies. Flulike symptoms, such as muscle aches and chills, are common side effects with interferon use. Patients are advised to use acetaminophen every 4 hours

The intensive care unit (ICU) nurse educator will determine that teaching about arterial pressure monitoring for a new staff nurse has been effective when the nurse a. balances and calibrates the monitoring equipment every 2 hours. b. positions the zero-reference stopcock line level with the phlebostatic axis. c. ensures that the patient is supine with the head of the bed flat for all readings. d. rechecks the location of the phlebostatic axis when changing the patient's position.

ANS: B For accurate measurement of pressures, the zero-reference level should be at the phlebostatic axis. There is no need to rebalance and recalibrate monitoring equipment hourly. Accurate hemodynamic readings are possible with the patient's head raised to 45 degrees or in the prone position. The anatomic position of the phlebostatic axis does not change when patients are repositioned

A couple is scheduled to have a Huhner test for infertility. In preparation for the test, the nurse will instruct the couple about a. being sedated during the procedure. b. determining the estimated time of ovulation. c. experiencing shoulder pain after the procedure. d. refraining from intercourse before the appointment.

ANS: B For the Huhner test, the couple should have intercourse at the estimated time of ovulation and then arrive for the test 2 to 8 hours after intercourse. The other instructions would be used for other types of fertility testing

A hospitalized patient who has received chemotherapy for leukemia develops neutropenia. Which observation by the nurse would indicate a need for further teaching? a. The patient ambulates several times a day in the room. b. The patient's visitors bring in some fresh peaches from home. c. The patient cleans with a warm washcloth after having a stool. d. The patient uses soap and shampoo to shower every other day.

ANS: B Fresh, thinned-skin fruits are not permitted in a neutropenic diet because of the risk of bacteria being present. The patient should ambulate in the room rather than the hospital hallway to avoid exposure to other patients or visitors. Because overuse of soap can dry the skin and increase infection risk, showering every other day is acceptable. Careful cleaning after having a bowel movement will help prevent skin breakdown and infection

Which action should the nurse take before administering gentamicin (Garamycin) to a patient who has acute osteomyelitis? a. Ask the patient about any nausea. b. Review the patient's creatinine level. c. Obtain the patient's oral temperature. d. Change the prescribed wet-to-dry dressing.

ANS: B Gentamicin is nephrotoxic and can cause renal failure. Monitoring the patient's temperature before gentamicin administration is not necessary. Nausea is not a common side effect of IV gentamicin. There is no need to change the dressing before gentamicin administration

A patient with acute respiratory distress syndrome (ARDS) and acute kidney injury has the following medications ordered. Which medication should the nurse discuss with the health care provider before giving? a. Pantoprazole (Protonix) 40 mg IV b. Gentamicin (Garamycin) 60 mg IV c. Sucralfate (Carafate) 1 g per nasogastric tube d. Methylprednisolone (Solu-Medrol) 60 mg IV

ANS: B Gentamicin, which is one of the aminoglycoside antibiotics, is potentially nephrotoxic, and the nurse should clarify the drug and dosage with the health care provider before administration. The other medications are appropriate for the patient with ARDS

A young man arrives in the emergency department with ankle swelling and severe pain after twisting his ankle playing basketball. Which of these prescribed collaborative interventions will the nurse implement first? a. Take the patient to have x-rays. b. Wrap the ankle and apply an ice pack. c. Administer naproxen (Naprosyn) 500 mg PO. d. Give acetaminophen with codeine (Tylenol #3).

ANS: B Immediate care after a sprain or strain injury includes the application of cold and compression to the injury to minimize swelling. The other actions should be taken after the ankle is wrapped with a compression bandage and ice is applied

A hospice nurse who has become close to a terminally ill patient is present in the home when the patient dies and feels saddened and tearful as the family members begin to cry. Which action should the nurse take at this time? a. Contact a grief counselor as soon as possible. b. Cry along with the patient's family members. c. Leave the home as soon as possible to allow the family to grieve privately. d. Consider whether working in hospice is desirable because patient losses are common.

ANS: B It is appropriate for the nurse to cry and express sadness in other ways when a patient dies, and the family is likely to feel that this is therapeutic. Contacting a grief counselor, leaving the family to grieve privately, and considering whether hospice continues to be a satisfying place to work are all appropriate actions as well, but the nurse's initial action at this time should be to share the grieving process with the family

The nurse prepares to administer the following medications to a hospitalized patient with human immunodeficiency (HIV). Which medication is most important to administer at the right time? a. Oral acyclovir (Zovirax) b. Oral saquinavir (Invirase) c. Nystatin (Mycostatin) tablet d. Aerosolized pentamidine (NebuPent)

ANS: B It is important that antiretrovirals be taken at the prescribed time every day to avoid developing drug-resistant HIV. The other medications should also be given as close as possible to the correct time, but they are not as essential to receive at the same time every day

A 48-year-old male patient who has been diagnosed with gonococcal urethritis tells the nurse he had recent sexual contact with a woman but says she did not appear to have any disease. In responding to the patient, the nurse explains that a. women do not develop gonorrhea infections but can serve as carriers to spread the disease to males. b. women may not be aware they have gonorrhea because they often do not have symptoms of infection. c. women develop subclinical cases of gonorrhea that do not cause tissue damage or clinical manifestations. d. when gonorrhea infections occur in women, the disease affects only the ovaries and not the genital organs.

ANS: B Many women with gonorrhea are asymptomatic or have minor symptoms that are overlooked. The disease may affect both the genitals and the other reproductive organs and cause complications such as pelvic inflammatory disease (PID). Women who can transmit the disease have active infections

Which nursing action could the nurse delegate to unlicensed assistive personnel (UAP) when caring for a patient who is using a fentanyl (Duragesic) patch and a heating pad for treatment of chronic back pain? a. Check the skin under the heating pad. b. Take the respiratory rate every 2 hours. c. Monitor sedation using the sedation assessment scale. d. Ask the patient about whether pain control is effective.

ANS: B Obtaining the respiratory rate is included in UAP education and scope of practice. Assessment for sedation, pain control, and skin integrity requires more education and scope of practice

A pregnant woman with a history of asymptomatic chronic human immunodeficiency virus (HIV) infection is seen at the clinic. The patient states, "I am very nervous about making my baby sick." Which information will the nurse include when teaching the patient? a. The antiretroviral medications used to treat HIV infection are teratogenic. b. Most infants born to HIV-positive mothers are not infected with the virus. c. Because she is at an early stage of HIV infection, the infant will not contract HIV. d. It is likely that her newborn will become infected with HIV unless she uses antiretroviral therapy (ART).

ANS: B Only 25% of infants born to HIV-positive mothers develop HIV infection, even when the mother does not use ART during pregnancy. The percentage drops to 2% when ART is used. Perinatal transmission can occur at any stage of HIV infection (although it is less likely to occur when the viral load is lower). ART can safely be used in pregnancy, although some ART drugs should be avoided

A 29-year-old patient who is trying to become pregnant asks the nurse how to determine when she is most likely to conceive. The nurse explains that a. ovulation is unpredictable unless there are regular menstrual periods. b. ovulation prediction kits provide accurate information about ovulation. c. she will need to bring a specimen of cervical mucus to the clinic for testing. d. she should take her body temperature daily and have intercourse when it drops.

ANS: B Ovulation prediction kits indicate when luteinizing hormone (LH) levels first rise. Ovulation occurs about 28 to 36 hours after the first rise of LH. This information can be used to determine the best time for intercourse. Body temperature rises at ovulation. Postcoital cervical smears are used in infertility testing, but they do not predict the best time for conceiving and are not obtained by the patient. Determination of the time of ovulation can be predicted by basal body temperature charts or ovulation prediction kits and is not dependent on regular menstrual periods

A patient with second-degree burns has been receiving hydromorphone through patient-controlled analgesia (PCA) for a week. The patient wakes up frequently during the night complaining of pain. What action by the nurse is most appropriate? a. Administer a dose of morphine every 1 to 2 hours from the PCA machine while the patient is sleeping. b. Consult with the health care provider about using a different treatment protocol to control the patient's pain. c. Request that the health care provider order a bolus dose of morphine to be given when the patient awakens with pain. d. Teach the patient to push the button every 10 minutes for an hour before going to sleep, even if the pain is minimal.

ANS: B PCAs are best for controlling acute pain. This patient's history indicates chronic pain and a need for a pain management plan that will provide adequate analgesia while the patient is sleeping. Administering a dose of morphine when the patient already has severe pain will not address the problem. Teaching the patient to administer unneeded medication before going to sleep can result in oversedation and respiratory depression. It is illegal for the nurse to administer the morphine for a patient through PCA.

A patient who is to have no weight bearing on the left leg is learning to walk using crutches. Which observation by the nurse indicates that the patient can safely ambulate independently? a. The patient moves the right crutch with the right leg and then the left crutch with the left leg. b. The patient advances the left leg and both crutches together and then advances the right leg. c. The patient uses the bedside chair to assist in balance as needed when ambulating in the room. d. The patient keeps the padded area of the crutch firmly in the axillary area when ambulating.

ANS: B Patients are usually taught to move the crutches and the injured leg forward at the same time and then to move the unaffected leg. Patients are discouraged from using furniture to assist with ambulation. The patient is taught to place weight on the hands, not in the axilla, to avoid nerve damage. If the 2- or 4-point gaits are to be used, the crutch and leg on opposite sides move forward, not the crutch and same-side leg.

A teenaged male patient who wrestles in high school is examined by the nurse in the clinic. Which assessment finding would prompt the nurse to teach the patient about the importance of not sharing headgear to prevent the spread of pediculosis? a. Ringlike rashes with red, scaly borders over the entire scalp b. Papular, wheal-like lesions with white deposits on the hair shaft c. Patchy areas of alopecia with small vesicles and excoriated areas d. Red, hivelike papules and plaques with sharply circumscribed borders

ANS: B Pediculosis is characterized by wheal-like lesions with parasites that attach eggs to the base of the hair shaft. The other descriptions are more characteristic of other types of skin disorders

A 57-year-old patient is incontinent of urine following a radical retropubic prostatectomy. The nurse will plan to teach the patient a. to restrict oral fluid intake. b. pelvic floor muscle exercises. c. to perform intermittent self-catheterization. d. the use of belladonna and opium suppositories.

ANS: B Pelvic floor muscle training (Kegel) exercises are recommended to strengthen the pelvic floor muscles and improve urinary control. Belladonna and opium suppositories are used to reduce bladder spasms after surgery. Intermittent self-catheterization may be taught before surgery if the patient has urinary retention, but it will not be useful in reducing incontinence after surgery. The patient should have a daily oral intake of 2 to 3 L.

The day after a having a right below-the-knee amputation, a patient complains of pain in the right foot. Which action is best for the nurse to take? a. Explain the reasons for the phantom limb pain. b. Administer prescribed analgesics to relieve the pain. c. Loosen the compression bandage to decrease incisional pressure. d. Inform the patient that this phantom pain will diminish over time.

ANS: B Phantom limb sensation is treated like any other type of postoperative pain would be treated. Explanations of the reason for the pain may be given, but the nurse should still medicate the patient. The compression bandage is left in place except during physical therapy or bathing. Although the pain may decrease over time, it still requires treatment now

A 22-year-old tennis player has an arthroscopic repair of a rotator cuff injury performed in same-day surgery. When the nurse plans postoperative teaching for the patient, which information will be included? a. "You will not be able to serve a tennis ball again." b. "You will work with a physical therapist tomorrow." c. "The doctor will use the drop-arm test to determine the success of surgery." d. "Leave the shoulder immobilizer on for the first 4 days to minimize pain."

ANS: B Physical therapy after a rotator cuff repair begins on the first postoperative day to prevent "frozen shoulder." A shoulder immobilizer is used immediately after the surgery, but leaving the arm immobilized for several days would lead to loss of range of motion (ROM). The drop-arm test is used to test for rotator cuff injury, but not after surgery. The patient may be able to return to pitching after rehabilitation

The nurse teaches a patient diagnosed with systemic lupus erythematosus (SLE) about plasmapheresis. What instructions about plasmapheresis should the nurse include in the teaching plan? a. Plasmapheresis will eliminate eosinophils and basophils from blood. b. Plasmapheresis will remove antibody-antigen complexes from circulation. c. Plasmapheresis will prevent foreign antibodies from damaging various body tissues. d. Plasmapheresis will decrease the damage to organs caused by attacking T lymphocytes.

ANS: B Plasmapheresis is used in SLE to remove antibodies, antibody-antigen complexes, and complement from blood. T lymphocytes, foreign antibodies, eosinophils, and basophils do not directly contribute to the tissue damage in SLE

department after a motor vehicle crash is scheduled for chest and abdominal x-rays. Which information is most important to report to the health care provider before the x-rays are obtained? a. Severity of abdominal pain b. Positive result of hCG test c. Blood pressure 172/88 mm Hg d. Temperature 102.1° F (38.9° C)

ANS: B Positive hCG testing indicates that the patient is pregnant and that unnecessary abdominal x-rays should be avoided. The other information is also important to report, but it will not affect whether the x-rays should be done

The nurse in a health clinic receives requests for appointments from several patients. Which patient should be seen by the health care provider first? a. A 48-year-old man who has perineal pain and a temperature of 100.4° F b. A 58-year-old man who has a painful erection that has lasted over 6 hours c. A 38-year-old man who states he had difficulty maintaining an erection last night d. A 68-year-old man who has pink urine after a transurethral resection of the prostate (TURP) 3 days ago

ANS: B Priapism can cause complications such as necrosis or hydronephrosis, and this patient should be treated immediately. The other patients do not require immediate action to prevent serious complications

When caring for a patient who has an arterial catheter in the left radial artery for arterial pressure-based cardiac output (APCO) monitoring, which information obtained by the nurse is most important to report to the health care provider? a. The patient has a positive Allen test. b. There is redness at the catheter insertion site. c. The mean arterial pressure (MAP) is 86 mm Hg. d. The dicrotic notch is visible in the arterial waveform.

ANS: B Redness at the catheter insertion site indicates possible infection. The Allen test is performed before arterial line insertion, and a positive test indicates normal ulnar artery perfusion. A MAP of 86 is normal and the dicrotic notch is normally present on the arterial waveform

While family members are visiting, a patient has a respiratory arrest and is being resuscitated. Which action by the nurse is best? a. Tell the family members that watching the resuscitation will be very stressful. b. Ask family members if they wish to remain in the room during the resuscitation. c. Take the family members quickly out of the patient room and remain with them. d. Assign a staff member to wait with family members just outside the patient room.

ANS: B Research indicates that family members want the option of remaining in the room during procedures such as cardiopulmonary resuscitation (CPR) and that this decreases anxiety and facilitates grieving. The other options may be appropriate if the family decides not to remain with the patient

Which intervention will the nurse include in the plan of care for a patient who has cardiogenic shock? a. Check temperature every 2 hours. b. Monitor breath sounds frequently. c. Maintain patient in supine position. d. Assess skin for flushing and itching.

ANS: B Since pulmonary congestion and dyspnea are characteristics of cardiogenic shock, the nurse should assess the breath sounds frequently. The head of the bed is usually elevated to decrease dyspnea in patients with cardiogenic shock. Elevated temperature and flushing or itching of the skin are not typical of cardiogenic shock

A patient who is being treated for stage IV lung cancer tells the nurse about new-onset back pain. Which action should the nurse take first? a. Give the patient the prescribed PRN opioid. b. Assess for sensation and strength in the legs. c. Notify the health care provider about the symptoms. d. Teach the patient how to use relaxation to reduce pain.

ANS: B Spinal cord compression, an oncologic emergency, can occur with invasion of tumor into the epidural space. The nurse will need to assess the patient further for symptoms such as decreased leg sensation and strength and then notify the health care provider. Administration of opioids or use of relaxation may be appropriate but only after the nurse has assessed for possible spinal cord compression

A young adult patient with metastatic cancer, who is very close to death, appears restless. The patient keeps repeating, "I am not ready to die." Which action is best for the nurse to take? a. Remind the patient that no one feels ready for death. b. Sit at the bedside and ask if there is anything the patient needs. c. Insist that family members remain at the bedside with the patient. d. Tell the patient that everything possible is being done to delay death.

ANS: B Staying at the bedside and listening allows the patient to discuss any unresolved issues or physical discomforts that should be addressed. Stating that no one feels ready for death fails to address the individual patient's concerns. Telling the patient that everything is being done does not address the patient's fears about dying, especially since the patient is likely to die soon. Family members may not feel comfortable staying at the bedside of a dying patient, and the nurse should not insist that they remain there

The nurse administers an IV vesicant chemotherapeutic agent to a patient. Which action is most important for the nurse to take? a. Infuse the medication over a short period of time. b. Stop the infusion if swelling is observed at the site. c. Administer the chemotherapy through a small-bore catheter. d. Hold the medication unless a central venous line is available.

ANS: B Swelling at the site may indicate extravasation, and the IV should be stopped immediately. The medication generally should be given slowly to avoid irritation of the vein. The size of the catheter is not as important as administration of vesicants into a running IV line to allow dilution of the chemotherapeutic drug. These medications can be given through peripheral lines, although central vascular access devices (CVADs) are preferred

Eight years after seroconversion, a human immunodeficiency virus (HIV)-infected patient has a CD4+ cell count of 800/µL and an undetectable viral load. What is the priority nursing intervention at this time? a. Teach about the effects of antiretroviral agents. b. Encourage adequate nutrition, exercise, and sleep. c. Discuss likelihood of increased opportunistic infections. d. Monitor for symptoms of acquired immunodeficiency syndrome (AIDS).

ANS: B The CD4+ level for this patient is in the normal range, indicating that the patient is the stage of asymptomatic chronic infection, when the body is able to produce enough CD4+ cells to maintain a normal CD4+ count. AIDS and increased incidence of opportunistic infections typically develop when the CD4+ count is much lower than normal. Although the initiation of ART is highly individual, it would not be likely that a patient with a normal CD4+ level would receive ART

A patient with cardiogenic shock has the following vital signs: BP 102/50, pulse 128, respirations 28. The pulmonary artery wedge pressure (PAWP) is increased and cardiac output is low. The nurse will anticipate an order for which medication? a. 5% human albumin b. Furosemide (Lasix) IV c. Epinephrine (Adrenalin) drip d. Hydrocortisone (Solu-Cortef)

ANS: B The PAWP indicates that the patient's preload is elevated, and furosemide is indicated to reduce the preload and improve cardiac output. Epinephrine would further increase heart rate and myocardial oxygen demand. 5% human albumin would also increase the PAWP. Hydrocortisone might be considered for septic or anaphylactic shock

A patient who is using a fentanyl (Duragesic) patch and immediate-release morphine for chronic cancer pain develops new-onset confusion, dizziness, and a decrease in respiratory rate. Which action should the nurse take first? a. Obtain vital signs. b. Remove the fentanyl patch. c. Notify the health care provider. d. Administer the prescribed PRN naloxone (Narcan).

ANS: B The assessment data indicate a possible overdose of opioid. The first action should be to remove the patch. Naloxone administration in a patient who has been chronically using opioids can precipitate withdrawal and would not be the first action. Notification of the health care provider and continued monitoring are also needed, but the patient's data indicate that more rapid action is needed. The respiratory rate alone is an indicator for immediate action before obtaining blood pressure, pulse, and temperature

The nurse educator is evaluating the performance of a new registered nurse (RN) who is providing care to a patient who is receiving mechanical ventilation with 15 cm H2O of peak end-expiratory pressure (PEEP). Which action indicates that the new RN is safe? a. The RN plans to suction the patient every 1 to 2 hours. b. The RN uses a closed-suction technique to suction the patient. c. The RN tapes connection between the ventilator tubing and the ET. d. The RN changes the ventilator circuit tubing routinely every 48 hours.

ANS: B The closed-suction technique is used when patients require high levels of PEEP (>10 cm H2O) to prevent the loss of PEEP that occurs when disconnecting the patient from the ventilator. Suctioning should not be scheduled routinely, but it should be done only when patient assessment data indicate the need for suctioning. Taping connections between the ET and the ventilator tubing would restrict the ability of the tubing to swivel in response to patient repositioning. Ventilator tubing changes increase the risk for ventilator-associated pneumonia (VAP) and are not indicated routinely

A healthy 28-year-old who has been vaccinated against human papillomavirus (HPV) has a normal Pap test. Which information will the nurse include in patient teaching when calling the patient with the results of the Pap test? a. You can wait until age 30 before having another Pap test. b. Pap testing is recommended every 3 years for women your age. c. No further Pap testing is needed until you decide to become pregnant. d. Yearly Pap testing is suggested for women with multiple sexual partners.

ANS: B The current national guidelines suggest Pap testing every 3 years for patients between ages 21 to 65. Although HPV immunization does protect against cervical cancer, the recommendations are unchanged for individuals who have received the HPV vaccination

The nurse in the clinic notes elevated prostate specific antigen (PSA) levels in the laboratory results of these patients. Which patient's PSA result is most important to report to the health care provider? a. A 38-year-old who is being treated for acute prostatitis b. A 48-year-old whose father died of metastatic prostate cancer c. A 52-year-old who goes on long bicycle rides every weekend d. A 75-year-old who uses saw palmetto to treat benign prostatic hyperplasia (BPH)

ANS: B The family history of prostate cancer and elevation of PSA indicate that further evaluation of the patient for prostate cancer is needed. Elevations in PSA for the other patients are not unusual

When evaluating a patient with a central venous catheter, the nurse observes that the insertion site is red and tender to touch and the patient's temperature is 101.8° F. What should the nurse plan to do next? a. Give analgesics and antibiotics as ordered. b. Discontinue the catheter and culture the tip. c. Change the flush system and monitor the site. d. Check the site more frequently for any swelling.

ANS: B The information indicates that the patient has a local and systemic infection caused by the catheter, and the catheter should be discontinued. Changing the flush system, giving analgesics, and continued monitoring will not help prevent or treat the infection. Administration of antibiotics is appropriate, but the line should still be discontinued to avoid further complications such as endocarditis

A 27-year-old man who has testicular cancer is being admitted for a unilateral orchiectomy. The patient does not talk to his wife and speaks to the nurse only to answer the admission questions. Which action is best for the nurse to take? a. Teach the patient and the wife that impotence is unlikely after unilateral orchiectomy. b. Ask the patient if he has any questions or concerns about the diagnosis and treatment. c. Document the patient's lack of communication on the chart and continue preoperative care. d. Inform the patient's wife that concerns about sexual function are common with this diagnosis.

ANS: B The initial action by the nurse should be assessment for any anxiety or questions about the surgery or postoperative care. The nurse should address the patient, not the spouse, when discussing the diagnosis and any possible concerns. Without further assessment of patient concerns, the nurse should not offer teaching about complications after orchiectomy. Documentation of the patient's lack of interaction is not an adequate nursing action in this situation

When doing discharge teaching for a 19-year-old patient who has had a repair of a fractured mandible, the nurse will include information about a. administration of nasogastric tube feedings. b. how and when to cut the immobilizing wires. c. the importance of high-fiber foods in the diet. d. the use of sterile technique for dressing changes.

ANS: B The jaw will be wired for stabilization, and the patient should know what emergency situations require that the wires be cut to protect the airway. There are no dressing changes for this procedure. The diet is liquid, and patients are not able to chew high-fiber foods. Initially, the patient may receive nasogastric tube feedings, but by discharge, the patient will swallow liquid through a straw

A patient who has had an open reduction and internal fixation (ORIF) of a hip fracture tells the nurse that he is ready to get out of bed for the first time. Which action should the nurse take? a. Use a mechanical lift to transfer the patient from the bed to the chair. b. Check the postoperative orders for the patient's weight-bearing status. c. Avoid administration of pain medications before getting the patient up. d. Delegate the transfer of the patient to nursing assistive personnel (NAP).

ANS: B The nurse should be familiar with the weight-bearing orders for the patient before attempting the transfer. Mechanical lifts are not typically needed after this surgery. Pain medications should be given because the movement is likely to be painful for the patient. The registered nurse (RN) should supervise the patient during the initial transfer to evaluate how well the patient is able to accomplish this skill

Which action will the nurse take first when a patient is seen in the outpatient clinic with neck pain? a. Provide information about therapeutic neck exercises. b. Ask about numbness or tingling of the hands and arms. c. Suggest that the patient alternate the use of heat and cold to the neck to treat the pain. d. Teach about the use of nonsteroidal antiinflammatory drugs such as ibuprofen (Advil).

ANS: B The nurse's initial action should be further assessment of the pain because cervical nerve root compression will require different treatment than musculoskeletal neck pain. The other actions may also be appropriate, depending on the assessment findings.

A 71-year-old patient who has benign prostatic hyperplasia (BPH) with urinary retention is admitted to the hospital with elevated blood urea nitrogen (BUN) and creatinine. Which prescribed therapy should the nurse implement first? a. Infuse normal saline at 50 mL/hr. b. Insert a urinary retention catheter. c. Draw blood for a complete blood count. d. Schedule a pelvic computed tomography (CT) scan.

ANS: B The patient data indicate that the patient may have acute kidney injury caused by the BPH. The initial therapy will be to insert a catheter. The other actions are also appropriate, but they can be implemented after the acute urinary retention is resolved

A 24-year-old patient with pelvic inflammatory disease (PID) is being treated with oral antibiotics as an outpatient. Which instruction will be included in patient teaching? a. Abdominal pain may persist for several weeks. b. Return for a follow-up appointment in 2 to 3 days. c. Instruct a male partner to use a condom during sexual intercourse for the next week. d. Nonsteroidal antiinflammatory drug (NSAID) use may prevent pelvic organ scarring

ANS: B The patient is instructed to return for follow-up in 48 to 72 hours. The patient should abstain from intercourse for 3 weeks. Abdominal pain should subside with effective antibiotic therapy. Corticosteroids may help prevent inflammation and scarring, but NSAIDs will not decrease scarring

A chemotherapy drug that causes alopecia is prescribed for a patient. Which action should the nurse take to maintain the patient's self-esteem? a. Tell the patient to limit social contacts until regrowth of the hair occurs. b. Encourage the patient to purchase a wig or hat and wear it once hair loss begins. c. Teach the patient to gently wash hair with a mild shampoo to minimize hair loss. d. Inform the patient that hair usually grows back once the chemotherapy is complete.

ANS: B The patient is taught to anticipate hair loss and to be prepared with wigs, scarves, or hats. Limiting social contacts is not appropriate at a time when the patient is likely to need a good social support system. The damage occurs at the hair follicle and will occur regardless of gentle washing or use of a mild shampoo. The information that the hair will grow back is not immediately helpful in maintaining the patient's self-esteem

Which statement by a 62-year-old patient who has had an above-the-knee amputation indicates that the nurse's discharge teaching has been effective? a. "I should elevate my residual limb on a pillow 2 or 3 times a day." b. "I should lay flat on my abdomen for 30 minutes 3 or 4 times a day." c. "I should change the limb sock when it becomes soiled or each week." d. "I should use lotion on the stump to prevent skin drying and cracking."

ANS: B The patient lies in the prone position several times daily to prevent flexion contractures of the hip. The limb sock should be changed daily. Lotion should not be used on the stump. The residual limb should not be elevated because this would encourage flexion contracture

An adolescent patient seeks care in the emergency department after sharing needles for heroin injection with a friend who has hepatitis B. To provide immediate protection from infection, what medication will the nurse administer? a. Corticosteroids b. Gamma globulin c. Hepatitis B vaccine d. Fresh frozen plasma

ANS: B The patient should first receive antibodies for hepatitis B from injection of gamma globulin. The hepatitis B vaccination series should be started to provide active immunity. Fresh frozen plasma and corticosteroids will not be effective in preventing hepatitis B in the patient

A 50-year-old patient is being discharged after a week of IV antibiotic therapy for acute osteomyelitis in the right leg. Which information will be included in the discharge teaching? a. How to apply warm packs to the leg to reduce pain b. How to monitor and care for the long-term IV catheter c. The need for daily aerobic exercise to help maintain muscle strength d. The reason for taking oral antibiotics for 7 to 10 days after discharge

ANS: B The patient will be on IV antibiotics for several months, and the patient will need to recognize signs of infection at the IV site and how to care for the catheter during daily activities such as bathing. IV antibiotics rather than oral antibiotics are used for acute osteomyelitis. Patients are instructed to avoid exercise and heat application because these will increase swelling and the risk for spreading infection

A 63-year-old woman undergoes an anterior and posterior (A&P) colporrhaphy for repair of a cystocele and rectocele. Which nursing action will be included in the postoperative care plan? a. Encourage a high-fiber diet. b. Perform indwelling catheter care. c. Repack the vagina with gauze daily. d. Teach the patient to insert a pessary.

ANS: B The patient will have a retention catheter for several days after surgery to keep the bladder empty and decrease strain on the suture. A pessary will not be needed after the surgery. Vaginal wound packing is not usually used after an A&P repair. A low-residue diet will be ordered after posterior colporrhaphy

The oxygen saturation (SpO2) for a patient with left lower lobe pneumonia is 90%. The patient has rhonchi, a weak cough effort, and complains of fatigue. Which action is a priority for the nurse to take? a. Position the patient on the left side. b. Assist the patient with staged coughing. c. Place a humidifier in the patient's room. d. Schedule a 2-hour rest period for the patient.

ANS: B The patient's assessment indicates that assisted coughing is needed to help remove secretions, which will improve oxygenation. A 2-hour rest period at this time may allow the oxygen saturation to drop further. Humidification will not be helpful unless the secretions can be mobilized. Positioning on the left side may cause a further decrease in oxygen saturation because perfusion will be directed more toward the more poorly ventilated lung

After being hospitalized for 3 days with a right femur fracture, a 32-year-old patient suddenly develops shortness of breath and tachypnea. The patient tells the nurse, "I feel like I am going to die!" Which action should the nurse take first? a. Stay with the patient and offer reassurance. b. Administer the prescribed PRN oxygen at 4 L/min. c. Check the patient's legs for swelling or tenderness. d. Notify the health care provider about the symptoms.

ANS: B The patient's clinical manifestations and history are consistent with a pulmonary embolus, and the nurse's first action should be to ensure adequate oxygenation. The nurse should offer reassurance to the patient, but meeting the physiologic need for oxygen is a higher priority. The health care provider should be notified after the oxygen is started and pulse oximetry and assessment for fat embolus or venous thromboembolism (VTE) are obtained

An 81-year-old patient who has been in the intensive care unit (ICU) for a week is now stable and transfer to the progressive care unit is planned. On rounds, the nurse notices that the patient has new onset confusion. The nurse will plan to a. give PRN lorazepam (Ativan) and cancel the transfer. b. inform the receiving nurse and then transfer the patient. c. notify the health care provider and postpone the transfer. d. obtain an order for restraints as needed and transfer the patient.

ANS: B The patient's history and symptoms most likely indicate delirium associated with the sleep deprivation and sensory overload in the ICU environment. Informing the receiving nurse and transferring the patient is appropriate. Postponing the transfer is likely to prolong the delirium. Benzodiazepines and restraints contribute to delirium and agitation

A patient with respiratory failure has a respiratory rate of 6 breaths/minute and an oxygen saturation (SpO2) of 88%. The patient is increasingly lethargic. Which intervention will the nurse anticipate? a. Administration of 100% oxygen by non-rebreather mask b. Endotracheal intubation and positive pressure ventilation c. Insertion of a mini-tracheostomy with frequent suctioning d. Initiation of continuous positive pressure ventilation (CPAP)

ANS: B The patient's lethargy, low respiratory rate, and SpO2 indicate the need for mechanical ventilation with ventilator-controlled respiratory rate. Administration of high flow oxygen will not be helpful because the patient's respiratory rate is so low. Insertion of a mini-tracheostomy will facilitate removal of secretions, but it will not improve the patient's respiratory rate or oxygenation. CPAP requires that the patient initiate an adequate respiratory rate to allow adequate gas exchange

A patient with terminal cancer-related pain and a history of opioid abuse complains of breakthrough pain 2 hours before the next dose of sustained-release morphine sulfate (MS Contin) is due. Which action should the nurse take first? a. Use distraction by talking about things the patient enjoys. b. Administer the prescribed PRN immediate-acting morphine. c. Suggest the use of alternative therapies such as heat or cold. d. Consult with the doctor about increasing the MS Contin dose.

ANS: B The patient's pain requires rapid treatment and the nurse should administer the immediate-acting morphine. Increasing the MS Contin dose and use of alternative therapies may also be needed, but the initial action should be to use the prescribed analgesic medications

A 42-year-old male patient complains of shoulder pain when the nurse moves his arm behind the back. Which question should the nurse ask? a. "Are you able to feed yourself without difficulty?" b. "Do you have difficulty when you are putting on a shirt?" c. "Are you able to sleep through the night without waking?" d. "Do you ever have trouble lowering yourself to the toilet?"

ANS: B The patient's pain will make it more difficult to accomplish tasks like putting on a shirt or jacket. This pain should not affect the patient's ability to feed himself or use the toilet because these tasks do not involve moving the arm behind the patient. The arm will not usually be positioned behind the patient during sleeping

A 52-year-old man tells the nurse that he decided to seek treatment for erectile dysfunction (ED) because his wife "is losing patience with the situation." The most appropriate nursing diagnosis for the patient is a. situational low self-esteem related to effects of ED. b. ineffective role performance related to effects of ED. c. anxiety related to inability to have sexual intercourse. d. ineffective sexuality patterns related to infrequent intercourse.

ANS: B The patient's statement indicates that the relationship with his wife is his primary concern. Although anxiety, low self-esteem, and ineffective sexuality patterns may also be concerns, the patient information suggests that addressing the role performance problem will lead to the best outcome for this patient

A 34-year-old woman who is discussing contraceptive options with the nurse says, "I want to have children, but not for a few years." Which response by the nurse is appropriate? a. "If you do not become pregnant within the next few years, you never will." b. "You may have more difficulty becoming pregnant after about age 35." c. "You have many years of fertility left, so there is no rush to have children." d. "You should plan to stop taking oral contraceptives several years before you want to become pregnant."

ANS: B The probability of successfully becoming pregnant decreases after age 35, although some patients may have no difficulty in becoming pregnant. Oral contraceptives do not need to be withdrawn for several years for a woman to become pregnant. Although the patient may be fertile for many years, it would be inaccurate to indicate that there is no concern about fertility as she becomes older. Although the risk for infertility increases after age 35, not all patients have difficulty in conceiving

The nurse cares for a patient with lung cancer in a home hospice program. Which action by the nurse is most appropriate? a. Discuss cancer risk factors and appropriate lifestyle modifications. b. Encourage the patient to discuss past life events and their meaning. c. Teach the patient about the purpose of chemotherapy and radiation. d. Accomplish a thorough head-to-toe assessment several times a week.

ANS: B The role of the hospice nurse includes assisting the patient with the important end-of-life task of finding meaning in the patient's life. Frequent head-to-toe assessments are not needed for hospice patients and may tire the patient unnecessarily. Patients admitted to hospice forego curative treatments such as chemotherapy and radiation for lung cancer. Discussion of cancer risk factors and therapies is not appropriate

A nurse is caring for a patient with ARDS who is being treated with mechanical ventilation and high levels of positive end-expiratory pressure (PEEP). Which assessment finding by the nurse indicates that the PEEP may need to be reduced? a. The patient's PaO2 is 50 mm Hg and the SaO2 is 88%. b. The patient has subcutaneous emphysema on the upper thorax. c. The patient has bronchial breath sounds in both the lung fields. d. The patient has a first-degree atrioventricular heart block with a rate of 58.

ANS: B The subcutaneous emphysema indicates barotrauma caused by positive pressure ventilation and PEEP. Bradycardia, hypoxemia, and bronchial breath sounds are all concerns and will need to be addressed, but they are not specific indications that PEEP should be reduced

A 32-year-old woman who is diagnosed with Chlamydia tells the nurse that she is very angry because her husband is her only sexual partner. Which response should the nurse make first? a. "You may need professional counseling to help resolve your anger." b. "It is understandable that you are angry with your husband right now." c. "Your feelings are justified and you should share them with your husband." d. "It is important that both you and your husband be treated for the infection."

ANS: B This response expresses the nurse's acceptance of the patient's feelings and encourages further discussion and problem solving. The patient may need professional counseling, but more assessment of the patient is needed before making this judgment. The nurse should also assess further before suggesting that the patient share her feelings with the husband because problems such as abuse might be present in the relationship. Although it is important that both partners be treated, the patient's current anger suggests that this is not the appropriate time to bring this up

When administering alendronate (Fosamax) to a patient with osteoporosis, the nurse will a. ask about any leg cramps or hot flashes. b. assist the patient to sit up at the bedside. c. be sure that the patient has recently eaten. d. administer the ordered calcium carbonate.

ANS: B To avoid esophageal erosions, the patient taking bisphosphonates should be upright for at least 30 minutes after taking the medication. Fosamax should be taken on an empty stomach, not after taking other medications or eating. Leg cramps and hot flashes are not side effects of bisphosphonates

When caring for a patient who is using Buck's traction after a hip fracture, which action can the nurse delegate to unlicensed assistive personnel (UAP)? a. Monitor the skin under the traction boot for redness. b. Ensure that the weight for the traction is off the floor. c. Check for intact sensation and movement in the affected leg. d. Offer reassurance that hip and leg pain are normal after hip fracture.

ANS: B UAP can be responsible for maintaining the integrity of the traction once it has been established. Assessment of skin integrity and circulation should be done by the registered nurse (RN). UAP should notify the RN if the patient experiences hip and leg pain because pain and effectiveness of pain relief measures should be assessed by the RN

When caring for a patient with continuous bladder irrigation after having transurethral resection of the prostate, which action could the nurse delegate to unlicensed assistive personnel (UAP)? a. Teach the patient how to perform Kegel exercises. b. Report any complaints of pain or spasms to the nurse. c. Monitor for increases in bleeding or presence of clots. d. Increase the flow rate of the irrigation if clots are noted.

ANS: B UAP education and role includes reporting patient concerns to supervising nurses. Patient teaching, assessments for complications, and actions such as bladder irrigation require more education and should be done by licensed nursing staff

A patient has a long-arm plaster cast applied for immobilization of a fractured left radius. Until the cast has completely dried, the nurse should a. keep the left arm in dependent position. b. avoid handling the cast using fingertips. c. place gauze around the cast edge to pad any roughness. d. cover the cast with a small blanket to absorb the dampness.

ANS: B Until a plaster cast has dried, using the palms rather than the fingertips to handle the cast helps prevent creating protrusions inside the cast that could place pressure on the skin. The left arm should be elevated to prevent swelling. The edges of the cast may be petaled once the cast is dry, but padding the edges before that may cause the cast to be misshapen. The cast should not be covered until it is dry because heat builds up during drying

A 32-year-old woman is scheduled for an induced abortion using instillation of hypertonic saline solution. Which information will the nurse plan to discuss with the patient before the procedure? a. The patient will require a general anesthetic. b. The expulsion of the fetus may take 1 to 2 days. c. There is a possibility that the patient may deliver a live fetus. d. The procedure may be unsuccessful in terminating the pregnancy.

ANS: B Uterine contractions take 12 to 36 hours to begin after the hypertonic saline is instilled. Because the saline is feticidal, the nurse does not need to discuss any possibility of a live delivery or that the pregnancy termination will not be successful. General anesthesia is not needed for this procedure

A patient with Hodgkin's lymphoma who is undergoing external radiation therapy tells the nurse, "I am so tired I can hardly get out of bed in the morning." Which intervention should the nurse add to the plan of care? a. Minimize activity until the treatment is completed. b. Establish time to take a short walk almost every day. c. Consult with a psychiatrist for treatment of depression. d. Arrange for delivery of a hospital bed to the patient's home.

ANS: B Walking programs are used to keep the patient active without excessive fatigue. Having a hospital bed does not necessarily address the fatigue. The better option is to stay as active as possible while combating fatigue. Fatigue is expected during treatment and is not an indication of depression. Minimizing activity may lead to weakness and other complications of immobility

After receiving 2 L of normal saline, the central venous pressure for a patient who has septic shock is 10 mm Hg, but the blood pressure is still 82/40 mm Hg. The nurse will anticipate an order for a. nitroglycerine (Tridil). b. norepinephrine (Levophed). c. sodium nitroprusside (Nipride). d. methylprednisolone (Solu-Medrol).

ANS: B When fluid resuscitation is unsuccessful, vasopressor drugs are administered to increase the systemic vascular resistance (SVR) and blood pressure, and improve tissue perfusion. Nitroglycerin would decrease the preload and further drop cardiac output and BP. Methylprednisolone (Solu-Medrol) is considered if blood pressure does not respond first to fluids and vasopressors. Nitroprusside is an arterial vasodilator and would further decrease SVR

The son of a dying patient tells the nurse, "Mother doesn't really respond any more when I visit. I don't think she knows that I am here." Which response by the nurse is appropriate? a. "You may need to cut back your visits for now to avoid overtiring your mother." b. "Withdrawal may sometimes be a normal response when preparing to leave life." c. "It will be important for you to stimulate your mother as she gets closer to dying." d. "Many patients don't really know what is going on around them at the end of life."

ANS: B Withdrawal is a normal psychosocial response to approaching death. Dying patients may maintain the ability to hear while not being able to respond. Stimulation will tire the patient and is not an appropriate response to withdrawal in this circumstance. Visitors are encouraged to be "present" with the patient, talking softly and making physical contact in a way that does not demand a response from the patient

The nurse teaches a patient with cancer of the liver about high-protein, high-calorie diet choices. Which snack choice by the patient indicates that the teaching has been effective? a. Lime sherbet b. Blueberry yogurt c. Cream cheese bagel d. Fresh strawberries and bananas

ANS: B Yogurt has high biologic value because of the protein and fat content. Fruit salad does not have high amounts of protein or fat. Lime sherbet is lower in fat and protein than yogurt. Cream cheese is low in protein

Which nursing actions can the nurse working in a women's health clinic delegate to unlicensed assistive personnel (UAP) (select all that apply)? a. Call a patient with the results of an endometrial biopsy. b. Assist the health care provider with performing a Pap test. c. Draw blood for CA-125 levels for a patient with ovarian cancer. d. Screen a patient for use of medications that may cause amenorrhea. e. Teach the parent of a 10-year-old about the human papilloma virus (HPV) vaccine (Gardasil).

ANS: B, C Assisting with a Pap test and drawing blood (if trained) are skills that require minimal critical thinking and judgment and can be safely delegated to UAP. Patient teaching, calling a patient who may have questions about results of diagnostic testing, and risk-factor screening all require more education and critical thinking and should be done by the registered nurse (RN).

According to the Center for Disease Control (CDC) guidelines, which personal protective equipment will the nurse put on when assessing a patient who is on contact precautions for diarrhea caused by Clostridium difficile (select all that apply)? a. Mask b. Gown c. Gloves d. Shoe covers e. Eye protection

ANS: B, C Because the nurse will have substantial contact with the patient and bedding when doing an assessment, gloves and gowns are needed. Eye protection and masks are needed for patients in contact precautions only when spraying or splashing is anticipated. Shoe covers are not recommended in the CDC guidelines

The health care provider orders a patient-controlled analgesia (PCA) machine to provide pain relief for a patient with acute surgical pain who has never received opioids in the past. Which nursing actions regarding opioid administration are appropriate at this time (select all that apply)? a. Assess for signs that the patient is becoming addicted to the opioid. b. Monitor for therapeutic and adverse effects of opioid administration. c. Emphasize that the risk of some opioid side effects increases over time. d. Teach the patient about how analgesics improve postoperative activity levels. e. Provide instructions on decreasing opioid doses by the second postoperative day.

ANS: B, D Monitoring for pain relief and teaching the patient about how opioid use will improve postoperative outcomes are appropriate actions when administering opioids for acute pain. Although postoperative patients usually need a decreasing amount of opioids by the second postoperative day, each patient's response is individual. Tolerance may occur, but addiction to opioids will not develop in the acute postoperative period. The patient should use the opioids to achieve adequate pain control, and so the nurse should not emphasize the adverse effects

Which topics will the nurse include when preparing to teach a patient with recurrent genital herpes simplex (select all that apply)? a. Infected areas should be kept moist to speed healing. b. Sitz baths may be used to relieve discomfort caused by the lesions. c. Genital herpes can be cured by consistent use of antiviral medications. d. Recurrent genital herpes episodes usually are shorter than the first episode. e. The virus can infect sexual partners even when you do not have symptoms of infection.

ANS: B, D, E Patients are taught that shedding of the virus and infection of sexual partners can occur even in asymptomatic periods, that recurrent episodes resolve more quickly, and that sitz baths can be used to relieve pain caused by the lesions. Antiviral medications decrease the number of outbreaks, but do not cure herpes simplex infections. Infected areas may be kept dry if this decreases pain and itching

An intraaortic balloon pump (IABP) is being used for a patient who is in cardiogenic shock. Which assessment data indicate to the nurse that the goals of treatment with the IABP are being met? a. Urine output of 25 mL/hr b. Heart rate of 110 beats/minute c. Cardiac output (CO) of 5 L/min d. Stroke volume (SV) of 40 mL/beat

ANS: C A CO of 5 L/min is normal and indicates that the IABP has been successful in treating the shock. The low SV signifies continued cardiogenic shock. The tachycardia and low urine output also suggest continued cardiogenic shock

The nurse teaches a patient who is scheduled for a prostate needle biopsy about the procedure. Which statement, if made by the patient, indicates that teaching was effective? a. "The biopsy will remove the cancer in my prostate gland." b. "The biopsy will determine how much longer I have to live." c. "The biopsy will help decide the treatment for my enlarged prostate." d. "The biopsy will indicate whether the cancer has spread to other organs."

ANS: C A biopsy is used to determine whether the prostate enlargement is benign or malignant, and determines the type of treatment that will be needed. A biopsy does not give information about metastasis, life expectancy, or the impact of cancer on the patient's life

When a 31-year-old male patient returns to the clinic for follow-up after treatment for gonococcal urethritis, a purulent urethral discharge is still present. When trying to determine the reason for the recurrent infection, which question is most appropriate for the nurse to ask the patient? a. "Did you take the prescribed antibiotic for a week?" b. "Did you drink at least 2 quarts of fluids every day?" c. "Were your sexual partners treated with antibiotics?" d. "Do you wash your hands after using the bathroom?"

ANS: C A common reason for recurrence of symptoms is reinfection because infected partners have not been simultaneously treated. Because gonorrhea is treated with one dose of antibiotic, antibiotic therapy for a week is not needed. An adequate fluid intake is important, but a low fluid intake is not a likely cause for failed treatment. Poor hygiene may cause complications such as ocular trachoma but will not cause a failure of treatment

A patient treated for human immunodeficiency virus (HIV) infection for 6 years has developed fat redistribution to the trunk, with wasting of the arms, legs, and face. What instructions will the nurse give to the patient? a. Review foods that are higher in protein. b. Teach about the benefits of daily exercise. c. Discuss a change in antiretroviral therapy. d. Talk about treatment with antifungal agents.

ANS: C A frequent first intervention for metabolic disorders is a change in antiretroviral therapy (ART). Treatment with antifungal agents would not be appropriate because there is no indication of fungal infection. Changes in diet or exercise have not proven helpful for this problem

Which action will the urgent care nurse take when caring for a patient who has a possible knee meniscus injury? a. Encourage bed rest for 24 to 48 hours. b. Avoid palpation or movement of the knee. c. Apply a knee immobilizer to the affected leg. d. Administer intravenous narcotics for pain relief.

ANS: C A knee immobilizer may be used for several days after a meniscus injury to stabilize the knee and minimize pain. Patients are encouraged to ambulate with crutches. The knee is assessed by flexing, internally rotating, and extending the knee (McMurray's test). The pain associated with a meniscus injury will not typically require IV opioid administration; nonsteroidal antiinflammatory drugs (NSAIDs) are usually recommended for pain relief

The emergency department (ED) nurse receives report that a patient involved in a motor vehicle crash is being transported to the facility with an estimated arrival in 1 minute. In preparation for the patient's arrival, the nurse will obtain a. hypothermia blanket. b. lactated Ringer's solution. c. two 14-gauge IV catheters. d. dopamine (Intropin) infusion.

ANS: C A patient with multiple trauma may require fluid resuscitation to prevent or treat hypovolemic shock, so the nurse will anticipate the need for 2 large bore IV lines to administer normal saline. Lactated Ringer's solution should be used cautiously and will not be ordered until the patient has been assessed for possible liver abnormalities. Vasopressor infusion is not used as the initial therapy for hypovolemic shock. Patients in shock need to be kept warm not cool.

A patient arrived at the emergency department after tripping over a rug and falling at home. Which finding is most important for the nurse to communicate to the health care provider? a. There is bruising at the shoulder area. b. The patient reports arm and shoulder pain. c. The right arm appears shorter than the left. d. There is decreased shoulder range of motion.

ANS: C A shorter limb after a fall indicates a possible dislocation, which is an orthopedic emergency. Bruising, pain, and decreased range of motion also should be reported, but these do not indicate that emergent treatment is needed to preserve

A woman is diagnosed with primary syphilis during her eighth week of pregnancy. The nurse will plan to teach the patient about the a. likelihood of a stillbirth. b. plans for cesarean section c. intramuscular injection of penicillin. d. antibiotic eye drops for the newborn.

ANS: C A single injection of penicillin is recommended to treat primary syphilis. This will treat the mother and prevent transmission of the disease to the fetus. Instillation of erythromycin into the eyes of the newborn is used to prevent gonorrheal eye infections. C-section is used to prevent the transmission of herpes to the newborn. Although stillbirth can occur if the fetus is infected with syphilis, treatment before the tenth week of gestation will eliminate in utero transmission to the fetus.

The nurse reviews the medication orders for an older patient with arthritis in both hips who is complaining of level 3 (0 to 10 scale) hip pain while ambulating. Which medication should the nurse use as initial therapy? a. Naproxen (Aleve) 200 mg orally b. Oxycodone (Roxicodone) 5 mg orally c. Acetaminophen (Tylenol) 650 mg orally d. Aspirin (acetylsalicylic acid, ASA) 650 mg orally

ANS: C Acetaminophen is the best first-choice medication. The principle of "start low, go slow" is used to guide therapy when treating older adults because the ability to metabolize medications is decreased and the likelihood of medication interactions is increased. Nonopioid analgesics are used first for mild to moderate pain, although opioids may be used later. Aspirin and the NSAIDs are associated with a high incidence of gastrointestinal bleeding in older patients

During change-of-shift report on a medical unit, the nurse learns that a patient with aspiration pneumonia who was admitted with respiratory distress has become increasingly agitated. Which action should the nurse take first? a. Give the prescribed PRN sedative drug. b. Offer reassurance and reorient the patient. c. Use pulse oximetry to check the oxygen saturation. d. Notify the health care provider about the patient's status.

ANS: C Agitation may be an early indicator of hypoxemia. The other actions may also be appropriate, depending on the findings about oxygen saturation

Which teaching should the nurse provide about intradermal skin testing to a patient with possible allergies? a. "Do not eat anything for about 6 hours before the testing." b. "Take an oral antihistamine about an hour before the testing." c. "Plan to wait in the clinic for 20 to 30 minutes after the testing." d. "Reaction to the testing will take about 48 to 72 hours to occur."

ANS: C Allergic reactions usually occur within minutes after injection of an allergen, and the patient will be monitored for at least 20 minutes for anaphylactic reactions after the testing. Medications that might modify the response, such as antihistamines, should be avoided before allergy testing. There is no reason to be NPO for skin testing. Results with intradermal testing occur within minutes

The health care provider prescribes the following interventions for a patient with acute prostatitis caused by E. coli. Which intervention should the nurse question? a. Instruct patient to avoid sexual intercourse until treatment is complete. b. Administer ibuprofen (Advil) 400 mg every 8 hours as needed for pain. c. Catheterize the patient as needed if symptoms of urinary retention develop. d. Give trimethoprim/sulfamethoxazole (Bactrim) DS 1 tablet daily for 28 days.

ANS: C Although acute urinary retention may occur, insertion of a catheter through an inflamed urethra is contraindicated and the nurse will anticipate that the health care provider will need to insert a suprapubic catheter. The other actions are appropriate

A 53-year-old man is scheduled for an annual physical exam. The nurse will plan to teach the patient about the purpose of a. urinalysis collection. b. uroflowmetry studies. c. prostate specific antigen (PSA) testing. d. transrectal ultrasound scanning (TRUS).

ANS: C An annual digital rectal exam (DRE) and PSA are usually recommended starting at age 50 for men who have an average risk for prostate cancer. Urinalysis and uroflowmetry studies are done if patients have symptoms of urinary tract infection or changes in the urinary stream. TRUS may be ordered if the DRE or PSA is abnormal

The health care provider asks the nurse whether a patient's angioedema has responded to prescribed therapies. Which assessment should the nurse perform? a. Ask the patient about any clear nasal discharge. b. Obtain the patient's blood pressure and heart rate. c. Check for swelling of the patient's lips and tongue. d. Assess the patient's extremities for wheal and flare lesions.

ANS: C Angioedema is characterized by swelling of the eyelids, lips, and tongue. Wheal and flare lesions, clear nasal drainage, and hypotension and tachycardia are characteristic of other allergic reactions.

Immediately after the nurse administers an intracutaneous injection of an allergen on the forearm, a patient complains of itching at the site and of weakness and dizziness. What action should the nurse take first? a. Remind the patient to remain calm. b. Administer subcutaneous epinephrine. c. Apply a tourniquet above the injection site. d. Rub a local antiinflammatory cream on the site.

ANS: C Application of a tourniquet will decrease systemic circulation of the allergen and should be the first reaction. A local antiinflammatory cream may be applied to the site of a cutaneous test if the itching persists. Epinephrine will be needed if the allergic reaction progresses to anaphylaxis. The nurse should assist the patient to remain calm, but this is not an adequate initial nursing action

A 19-year-old patient calls the school clinic and tells the nurse, "My menstrual period is very heavy this time. I have to change my tampon every 4 hours." Which action should the nurse take next? a. Tell the patient that her flow is not unusually heavy. b. Schedule the patient for an appointment later that day. c. Ask the patient how heavy her usual menstrual flow is. d. Have the patient call again if the heavy flow continues.

ANS: C Because a heavy menstrual flow is usually indicated by saturating a pad or tampon in 1 to 2 hours, the nurse should first assess how heavy the patient's usual flow is. There is no need to schedule the patient for an appointment that day. The patient may need to call again, but this is not the first action that the nurse should take. Telling the patient that she does not have a heavy flow implies that the patient's concern is not important

The nurse receives change-of-shift report on the oncology unit. Which patient should the nurse assess first? a. 35-year-old patient who has wet desquamation associated with abdominal radiation b. 42-year-old patient who is sobbing after receiving a new diagnosis of ovarian cancer c. 24-year-old patient who received neck radiation and has blood oozing from the neck d. 56-year-old patient who developed a new pericardial friction rub after chest radiation

ANS: C Because neck bleeding may indicate possible carotid artery rupture in a patient who is receiving radiation to the neck, this patient should be seen first. The diagnoses and clinical manifestations for the other patients are not immediately life threatening

A patient who collects honey to earn supplemental income has developed a hypersensitivity to bee stings. Which statement, if made by the patient, would indicate a need for additional teaching? a. "I need to find another way to earn extra money." b. "I will get a prescription for epinephrine and learn to self-inject it." c. "I will plan to take oral antihistamines daily before going to work." d. "I should wear a Medic-Alert bracelet indicating my allergy to bee stings."

ANS: C Because the patient is at risk for bee stings and the severity of allergic reactions tends to increase with added exposure to allergen, taking oral antihistamines will not adequately control the patient's hypersensitivity reaction. The other patient statements indicate a good understanding of management of the problem

The nurse notes thick, white secretions in the endotracheal tube (ET) of a patient who is receiving mechanical ventilation. Which intervention will be most effective in addressing this problem? a. Increase suctioning to every hour. b. Reposition the patient every 1 to 2 hours. c. Add additional water to the patient's enteral feedings. d. Instill 5 mL of sterile saline into the ET before suctioning.

ANS: C Because the patient's secretions are thick, better hydration is indicated. Suctioning every hour without any specific evidence for the need will increase the incidence of mucosal trauma and would not address the etiology of the ineffective airway clearance. Instillation of saline does not liquefy secretions and may decrease the SpO2. Repositioning the patient is appropriate but will not decrease the thickness of secretions

The nurse is providing teaching by telephone to a patient who is scheduled for a pelvic examination and Pap test next week. The nurse instructs the patient that she should a. shower, but not take a tub bath, before the examination. b. not have sexual intercourse the day before the Pap test. c. avoid douching for at least 24 hours before the examination. d. schedule to have the Pap test just after her menstrual period.

ANS: C Because the results of a Pap test may be affected by douching, the patient should not douche before the examination. The exam may be scheduled without regard to the menstrual period. The patient may shower or bathe before the examination. Sexual intercourse does not affect the results of the examination or Pap test

The nurse, who is reviewing a clinic patient's medical record, notes that the patient missed the previous appointment for weekly immunotherapy. Which action by the nurse is most appropriate? a. Schedule an additional dose that week. b. Administer the usual dosage of the allergen. c. Consult with the health care provider about giving a lower allergen dose. d. Re-evaluate the patient's sensitivity to the allergen with a repeat skin test.

ANS: C Because there is an increased risk for adverse reactions after a patient misses a scheduled dose of allergen, the nurse should check with the health care provider before administration of the injection. A skin test is used to identify the allergen and would not be used at this time. An additional dose for the week may increase the risk for a reaction.

Which information will the nurse include when teaching a patient who has developed a small vesicovaginal fistula 2 weeks into the postpartum period? a. Take stool softeners to prevent fecal contamination of the vagina. b. Limit oral fluid intake to minimize the quantity of urinary drainage. c. Change the perineal pad frequently to prevent perineal skin breakdown. d. Call the health care provider immediately if urine drains from the vagina.

ANS: C Because urine will leak from the bladder, the patient should plan to use perineal pads and change them frequently. A high fluid intake is recommended to decrease the risk for urinary tract infections. Drainage of urine from the vagina is expected with vesicovaginal fistulas. Fecal contamination is not a concern with vesicovaginal fistulas.

Which finding about a patient who is receiving vasopressin (Pitressin) to treat septic shock is most important for the nurse to communicate to the health care provider? a. The patient's urine output is 18 mL/hr. b. The patient's heart rate is 110 beats/minute. c. The patient is complaining of chest pain. d. The patient's peripheral pulses are weak.

ANS: C Because vasopressin is a potent vasoconstrictor, it may decrease coronary artery perfusion. The other information is consistent with the patient's diagnosis and should be reported to the health care provider but does not indicate a need for a change in therapy

A 25-year-old woman who is scheduled for a routine gynecologic examination tells the nurse that she has had intercourse during the last year with several men. The nurse will plan to teach about the reason for a. contraceptive use. b. antibiotic therapy. c. Chlamydia testing. d. pregnancy testing.

ANS: C Chlamydia testing is recommended annually for women with multiple sex partners. There is no indication that the patient needs teaching about contraceptives, pregnancy testing, or antibiotic therapy

Which finding from the nurse's physical assessment of a 42-year-old male patient should be reported to the health care provider? a. One testis hangs lower than the other. b. Genital hair distribution is diamond shaped. c. Clear discharge is present at the penile meatus. d. Inguinal lymph nodes are nonpalpable bilaterally.

ANS: C Clear penile discharge may be indicative of a sexually transmitted infection (STI). The other findings are normal and do not need to be reported

A new mother expresses concern about her baby developing allergies and asks what the health care provider meant by "passive immunity." Which example should the nurse use to explain this type of immunity? a. Early immunization b. Bone marrow donation c. Breastfeeding her infant d. Exposure to communicable diseases

ANS: C Colostrum provides passive immunity through antibodies from the mother. These antibodies protect the infant for a few months. However, memory cells are not retained, so the protection is not permanent. Active immunity is acquired by being immunized with vaccinations or having an infection. It requires that the infant has an immune response after exposure to an antigen. Cell-mediated immunity is acquired through T lymphocytes and is a form of active immunity

When teaching seniors at a community recreation center, which information will the nurse include about ways to prevent fractures? a. Tack down scatter rugs in the home. b. Most falls happen outside the home. c. Buy shoes that provide good support and are comfortable to wear. d. Range-of-motion exercises should be taught by a physical therapist.

ANS: C Comfortable shoes with good support will help decrease the risk for falls. Scatter rugs should be eliminated, not just tacked down. Activities of daily living provide range of motion exercise; these do not need to be taught by a physical therapist. Falls inside the home are responsible for many injuries

The nurse instructs a patient about application of corticosteroid cream to an area of contact dermatitis on the right leg. Which patient action indicates that further teaching is needed? a. The patient takes a tepid bath before applying the cream. b. The patient spreads the cream using a downward motion. c. The patient applies a thick layer of the cream to the affected skin. d. The patient covers the area with a dressing after applying the cream.

ANS: C Creams and ointments should be applied in a thin layer to avoid wasting the medication. The other actions by the patient indicate that the teaching has been successful

A 49-year-old woman is considering the use of combined estrogen-progesterone hormone replacement therapy (HT) during menopause. Which information will the nurse include during their discussion? a. Use of estrogen-containing vaginal creams provides most of the same benefits as oral HT. b. Increased incidence of colon cancer in women taking HT requires more frequent colonoscopy. c. HT decreases osteoporosis risk and increases the risk for cardiovascular disease and breast cancer. d. Use of HT for up to 10 years to prevent symptoms such as hot flashes is generally considered safe.

ANS: C Data from the Women's Health Initiative indicate an increased risk for cardiovascular disease and breast cancer in women taking combination HT but a decrease in hip fractures. Vaginal creams decrease symptoms related to vaginal atrophy and dryness, but they do not offer the other benefits of HT, such as decreased hot flashes. Most women who use HT are placed on short-term treatment and are not treated for up to 10 years. The incidence of colon cancer decreases in women taking HRT

A patient who has a positive test for human immunodeficiency virus (HIV) antibodies is admitted to the hospital with Pneumocystis jiroveci pneumonia (PCP) and a CD4+ T-cell count of less than 200 cells/mL. Based on diagnostic criteria established by the Centers for Disease Control and Prevention (CDC), which statement by the nurse is correct? a. "The patient meets the criteria for a diagnosis of an acute HIV infection." b. "The patient will be diagnosed with asymptomatic chronic HIV infection." c. "The patient has developed acquired immunodeficiency syndrome (AIDS)." d. "The patient will develop symptomatic chronic HIV infection in less than a year."

ANS: C Development of PCP meets the diagnostic criterion for AIDS. The other responses indicate earlier stages of HIV infection than is indicated by the PCP infection

A nurse who works on the orthopedic unit has just received the change-of-shift report. Which patient should the nurse assess first? a. Patient who reports foot pain after hammertoe surgery b. Patient with low back pain and a positive straight-leg-raise test c. Patient who has not voided 10 hours after having a laminectomy d. Patient with osteomyelitis who has a temperature of 100.5° F (38.1° C)

ANS: C Difficulty in voiding may indicate damage to the spinal nerves and should be assessed and reported to the surgeon immediately. The information about the other patients is consistent with their diagnoses. The nurse will need to assess them as quickly as possible, but the information about them does not indicate a need for immediate intervention

The nurse notes premature ventricular contractions (PVCs) while suctioning a patient's endotracheal tube. Which action by the nurse is a priority? a. Decrease the suction pressure to 80 mm Hg. b. Document the dysrhythmia in the patient's chart. c. Stop and ventilate the patient with 100% oxygen. d. Give antidysrhythmic medications per protocol.

ANS: C Dysrhythmias during suctioning may indicate hypoxemia or sympathetic nervous system stimulation. The nurse should stop suctioning and ventilate the patient with 100% oxygen. Lowering the suction pressure will decrease the effectiveness of suctioning without improving the hypoxemia. Because the PVCs occurred during suctioning, there is no need for antidysrhythmic medications (which may have adverse effects) unless they recur when the suctioning is stopped and patient is well oxygenated

A young adult female patient who is human immunodeficiency virus (HIV)-positive has a new prescription for efavirenz (Sustiva). Which information is most important to include in the medication teaching plan? a. Driving is allowed when starting this medication. b. Report any bizarre dreams to the health care provider. c. Continue to use contraception while on this medication. d. Take this medication in the morning on an empty stomach.

ANS: C Efavirenz can cause fetal anomalies and should not be used in patients who may be pregnant. The drug should not be used during pregnancy because large doses could cause fetal anomalies. Once-a-day doses should be taken at bedtime (at least initially) to help patients cope with the side effects that include dizziness and confusion. Patients should be cautioned about driving when starting this drug. Patients should be informed that many people who use the drug have reported vivid and sometimes bizarre dreams

Which information will the nurse teach a patient who has chronic prostatitis? a. Ibuprofen (Motrin) should provide good pain control. b. Prescribed antibiotics should be taken for 7 to 10 days. c. Intercourse or masturbation will help relieve symptoms. d. Cold packs used every 4 hours will decrease inflammation.

ANS: C Ejaculation helps drain the prostate and relieve pain. Warm baths are recommended to reduce pain. Nonsteroidal antiinflammatory drugs (NSAIDs) are frequently prescribed but usually do not offer adequate pain relief. Antibiotics for chronic prostatitis are taken for 4 to 12 weeks

Which question should the nurse ask when assessing a 60-year-old patient who has a history of benign prostatic hyperplasia (BPH)? a. "Have you noticed any unusual discharge from your penis?" b. "Has there been any change in your sex life in the last year?" c. "Has there been a decrease in the force of your urinary stream?" d. "Have you been experiencing any difficulty in achieving an erection?"

ANS: C Enlargement of the prostate blocks the urethra, leading to urinary changes such as a decrease in the force of the urinary stream. The other questions address possible problems with infection or sexual difficulties, but they would not be helpful in determining whether there were functional changes caused by BPH.

A 67-year-old patient is receiving IV antibiotics at home to treat chronic osteomyelitis of the left femur. The nurse chooses a nursing diagnosis of ineffective health maintenance when the nurse finds that the patient a. is frustrated with the length of treatment required. b. takes and records the oral temperature twice a day. c. is unable to plantar flex the foot on the affected side. d. uses crutches to avoid weight bearing on the affected leg.

ANS: C Foot drop is an indication that the foot is not being supported in a neutral position by a splint. Using crutches and monitoring the oral temperature are appropriate self-care activities. Frustration with the length of treatment is not an indicator of ineffective health maintenance of the osteomyelitis

A patient who has severe pain associated with terminal pancreatic cancer is being cared for at home by family members. Which finding by the nurse indicates that teaching regarding pain management has been effective? a. The patient uses the ordered opioid pain medication whenever the pain is greater than 5 (0 to 10 scale). b. The patient agrees to take the medications by the IV route in order to improve analgesic effectiveness. c. The patient takes opioids around the clock on a regular schedule and uses additional doses when breakthrough pain occurs. d. The patient states that nonopioid analgesics may be used when the maximal dose of the opioid is reached without adequate pain relief.

ANS: C For chronic cancer pain, analgesics should be taken on a scheduled basis, with additional doses as needed for breakthrough pain. Taking the medications only when pain reaches a certain level does not provide effective pain control. Although nonopioid analgesics also may be used, there is no maximum dose of opioid. Opioids are given until pain control is achieved. The IV route is not more effective than the oral route, and usually the oral route is preferred

Which medication information will the nurse identify as a concern for a patient's musculoskeletal status? a. The patient takes a daily multivitamin and calcium supplement. b. The patient takes hormone therapy (HT) to prevent "hot flashes." c. The patient has severe asthma and requires frequent therapy with oral corticosteroids. d. The patient has migraine headaches treated with nonsteroidal antiinflammatory drugs (NSAIDs).

ANS: C Frequent or chronic corticosteroid use may lead to skeletal problems such as avascular necrosis and osteoporosis. The use of HT and calcium supplements will help prevent osteoporosis. NSAID use does not increase the risk for musculoskeletal problems

Which patient should the nurse refer for hospice care? a. 60-year-old with lymphoma whose children are unable to discuss issues related to dying b. 72-year-old with chronic severe pain as a result of spinal arthritis and vertebral collapse c. 28-year-old with AIDS-related dementia who needs palliative care and pain management d. 56-year-old with advanced liver failure whose family members can no longer provide care in the home

ANS: C Hospice is designed to provide palliative care such as symptom management and pain control for patients at the end of life. Patients who require more care than the family can provide, whose families are unable to discuss important issues related to dying, or who have severe pain are candidates for other nursing services but are not appropriate hospice patients

Which statement by the patient indicates a good understanding of the nurse's teaching about a new short-arm plaster cast? a. "I can get the cast wet as long as I dry it right away with a hair dryer." b. "I should avoid moving my fingers and elbow until the cast is removed." c. "I will apply an ice pack to the cast over the fracture site off and on for 24 hours." d. "I can use a cotton-tipped applicator to rub lotion on any dry areas under the cast."

ANS: C Ice application for the first 24 hours after a fracture will help reduce swelling and can be placed over the cast. Plaster casts should not get wet. The patient should be encouraged to move the joints above and below the cast. Patients should not insert objects inside the cast

When giving home care instructions to a patient who has comminuted forearm fractures and a long-arm cast on the left arm, which information should the nurse include? a. Keep the left shoulder elevated on a pillow or cushion. b. Keep the hand immobile to prevent soft tissue swelling. c. Call the health care provider for increased swelling or numbness of the hand. d. Avoid nonsteroidal antiinflammatory drugs (NSAIDs) for 24 hours after the injury.

ANS: C Increased swelling or numbness may indicate increased pressure at the injury, and the health care provider should be notified immediately to avoid damage to nerves and other tissues. The patient should be encouraged to move the joints above and below the cast to avoid stiffness. There is no need to elevate the shoulder, although the forearm should be elevated to reduce swelling. NSAIDs are appropriate to treat pain after a fracture

The nurse is caring for a patient who has been diagnosed with stage I cancer of the colon. When assessing the need for psychologic support, which question by the nurse will provide the most information? a. "How long ago were you diagnosed with this cancer?" b. "Do you have any concerns about body image changes?" c. "Can you tell me what has been helpful to you in the past when coping with stressful events?" d. "Are you familiar with the stages of emotional adjustment to a diagnosis like cancer of the colon?"

ANS: C Information about how the patient has coped with past stressful situations helps the nurse determine usual coping mechanisms and their effectiveness. The length of time since the diagnosis will not provide much information about the patient's need for support. The patient's knowledge of typical stages in adjustment to a critical diagnosis does not provide insight into patient needs for assistance. Because surgical interventions for stage I cancer of the colon may not cause any body image changes, this question is not appropriate at this time

The family members of a patient who has just been admitted to the intensive care unit (ICU) with multiple traumatic injuries have just arrived in the ICU waiting room. Which action should the nurse take next? a. Explain ICU visitation policies and encourage family visits. b. Immediately take the family members to the patient's bedside. c. Describe the patient's injuries and the care that is being provided. d. Invite the family to participate in a multidisciplinary care conference.

ANS: C Lack of information is a major source of anxiety for family members and should be addressed first. Family members should be prepared for the patient's appearance and the ICU environment before visiting the patient for the first time. ICU visiting should be individualized to each patient and family rather than being dictated by rigid visitation policies. Inviting the family to participate in a multidisciplinary conference is appropriate but should not be the initial action by the nurse

An older patient with cardiogenic shock is cool and clammy and hemodynamic monitoring indicates a high systemic vascular resistance (SVR). Which intervention should the nurse anticipate doing next? a. Increase the rate for the dopamine (Intropin) infusion. b. Decrease the rate for the nitroglycerin (Tridil) infusion. c. Increase the rate for the sodium nitroprusside (Nipride) infusion. d. Decrease the rate for the 5% dextrose in normal saline (D5/.9 NS) infusion.

ANS: C Nitroprusside is an arterial vasodilator and will decrease the SVR and afterload, which will improve cardiac output. Changes in the D5/.9 NS and nitroglycerin infusions will not directly decrease SVR. Increasing the dopamine will tend to increase SVR.

The nurse teaches a student nurse about the action of ibuprofen. Which statement, if made by the student, indicates that teaching was effective? a. "The drug decreases pain impulses in the spinal cord." b. "The drug decreases sensitivity of the brain to painful stimuli." c. "The drug decreases production of pain-sensitizing chemicals." d. "The drug decreases the modulating effect of descending nerves."

ANS: C Nonsteroidal antiinflammatory drugs (NSAIDs) provide analgesic effects by decreasing the production of pain-sensitizing chemicals such as prostaglandins at the site of injury. Transmission of impulses through the spinal cord, brain sensitivity to pain, and the descending nerve pathways are not affected by NSAIDs

A 27-year-old patient tells the nurse that she would like a prescription for oral contraceptives to control her premenstrual dysphoric disorder (PMD-D) symptoms. Which patient information is most important to communicate to the health care provider? a. Bilateral breast tenderness b. Frequent abdominal bloating c. History of migraine headaches d. Previous spontaneous abortion

ANS: C Oral contraceptives are contraindicated in patients with a history of migraine headaches. The other patient information would not prevent the patient from receiving oral contraceptives

An 18-year-old requests a prescription for birth control pills to control severe abdominal cramping and headaches during her menstrual periods. Which should the nurse take first? a. Determine whether the patient is sexually active. b. Teach about the side effects of oral contraceptives. c. Take a personal and family health history from the patient. d. Suggest nonsteroidal antiinflammatory drugs (NSAIDs) for relief.

ANS: C Oral contraceptives may be appropriate to control this patient's symptoms, but the patient's health history may indicate contraindications to oral contraceptive use. Because the patient is requesting contraceptives for management of dysmenorrhea, whether she is sexually active is irrelevant. Because the patient is asking for birth control pills, responding that she should try NSAIDs is nontherapeutic. The patient does not need teaching about oral contraceptive side effects at this time.

When caring for a patient with pulmonary hypertension, which parameter is most appropriate for the nurse to monitor to evaluate the effectiveness of the treatment? a. Central venous pressure (CVP) b. Systemic vascular resistance (SVR) c. Pulmonary vascular resistance (PVR) d. Pulmonary artery wedge pressure (PAWP)

ANS: C PVR is a major contributor to pulmonary hypertension, and a decrease would indicate that pulmonary hypertension was improving. The other parameters also may be monitored but do not directly assess for pulmonary hypertension

Which action by the unlicensed assistive personnel (UAP) who are assisting with the care of patients with male reproductive problems indicates that the nurse should provide more teaching? a. The UAP apply a cold pack to the scrotum for a patient with mumps orchitis. b. The UAP help a patient who has had a prostatectomy to put on antiembolism hose. c. The UAP leave the foreskin pulled back after cleaning the glans of a patient who has a retention catheter. d. The UAP encourage a high oral fluid intake for patient who had transurethral resection of the prostate yesterday.

ANS: C Paraphimosis can be caused by failing to replace the foreskin back over the glans after cleaning. The other actions by UAP are appropriate

When a patient arrives in the emergency department with a facial fracture, which action will the nurse take first? a. Assess for nasal bleeding and pain. b. Apply ice to the face to reduce swelling. c. Use a cervical collar to stabilize the spine. d. Check the patient's alertness and orientation.

ANS: C Patients who have facial fractures are at risk for cervical spine injury and should be treated as if they have a cervical spine injury until this is ruled out. The other actions are also necessary, but the most important action is to prevent cervical spine injury.

Which patient in the women's health clinic will the nurse expect to teach about an endometrial biopsy? a. The 55-year-old patient who has 3 to 4 alcoholic drinks each day b. The 35-year-old patient who has used oral contraceptives for 15 years c. The 25-year-old patient who has a family history of hereditary nonpolyposis colorectal cancer d. The 45-year-old patient who has had 6 full-term pregnancies and 2 spontaneous abortions

ANS: C Patients with a personal or familial history of hereditary nonpolyposis colorectal cancer are at increased risk for endometrial cancer. Alcohol addiction does not increase this risk. Multiple pregnancies and oral contraceptive use offer protection from endometrial cancer

Which action should the nurse take when a 35-year-old patient has a result of minor cellular changes on her Pap test? a. Teach the patient about colposcopy. b. Teach the patient about punch biopsy. c. Schedule another Pap test in 4 months. d. Administer the human papillomavirus (HPV) vaccine.

ANS: C Patients with minor changes on the Pap test can be followed with Pap tests every 4 to 6 months because these changes may revert to normal. Punch biopsy or colposcopy may be used if the Pap test shows more prominent changes. The HPV vaccine may reduce the risk for cervical cancer, but it is recommended only for ages 9 through 26.

The nurse palpates enlarged cervical lymph nodes on a patient diagnosed with acute human immunodeficiency virus (HIV) infection. Which action would be most appropriate for the nurse to take? a. Instruct the patient to apply ice to the neck. b. Advise the patient that this is probably the flu. c. Explain to the patient that this is an expected finding. d. Request that an antibiotic be prescribed for the patient.

ANS: C Persistent generalized lymphadenopathy is common in the early stages of HIV infection. No antibiotic is needed because the enlarged nodes are probably not caused by bacteria. Applying ice to the neck may provide comfort, but the initial action is to reassure the patient this is an expected finding. Lymphadenopathy is common with acute HIV infection and is therefore not likely the flu

A nurse has obtained donor tissue typing information about a patient who is waiting for a kidney transplant. Which results should be reported to the transplant surgeon? a. Patient is Rh positive and donor is Rh negative b. Six antigen matches are present in HLA typing c. Results of patient-donor cross matching are positive d. Panel of reactive antibodies (PRA) percentage is low

ANS: C Positive crossmatching is an absolute contraindication to kidney transplantation, since a hyperacute rejection will occur after the transplant. The other information indicates that the tissue match between the patient and potential donor is acceptable

A 42-year-old patient is admitted to the emergency department with a left femur fracture. Which information obtained by the nurse is most important to report to the health care provider? a. Ecchymosis of the left thigh b. Complaints of severe thigh pain c. Slow capillary refill of the left foot d. Outward pointing toes on the left foot

ANS: C Prolonged capillary refill may indicate complications such as arterial damage or compartment syndrome. The other findings are typical with a left femur fracture

A patient develops increasing dyspnea and hypoxemia 2 days after heart surgery. To determine whether the patient has acute respiratory distress syndrome (ARDS) or pulmonary edema caused by heart failure, the nurse will plan to assist with a. obtaining a ventilation-perfusion scan. b. drawing blood for arterial blood gases. c. insertion of a pulmonary artery catheter. d. positioning the patient for a chest x-ray.

ANS: C Pulmonary artery wedge pressures are normal in the patient with ARDS because the fluid in the alveoli is caused by increased permeability of the alveolar-capillary membrane rather than by the backup of fluid from the lungs (as occurs in cardiogenic pulmonary edema). The other tests will not help in differentiating cardiogenic from noncardiogenic pulmonary edema

External-beam radiation is planned for a patient with cervical cancer. What instructions should the nurse give to the patient to prevent complications from the effects of the radiation? a. Test all stools for the presence of blood. b. Maintain a high-residue, high-fiber diet. c. Clean the perianal area carefully after every bowel movement. d. Inspect the mouth and throat daily for the appearance of thrush.

ANS: C Radiation to the abdomen will affect organs in the radiation path, such as the bowel, and cause frequent diarrhea. Careful cleaning of this area will help decrease the risk for skin breakdown and infection. Stools are likely to have occult blood from the inflammation associated with radiation, so routine testing of stools for blood is not indicated. Radiation to the abdomen will not cause stomatitis. A low-residue diet is recommended to avoid irritation of the bowel when patients receive abdominal radiation.

Which nursing action included in the care of a patient after laminectomy can the nurse delegate to experienced unlicensed assistive personnel (UAP)? a. Check ability to plantar and dorsiflex the foot. b. Determine the patient's readiness to ambulate. c. Log roll the patient from side to side every 2 hours. d. Ask about pain control with the patient-controlled analgesia (PCA).

ANS: C Repositioning a patient is included in the education and scope of practice of UAP, and experienced UAP will be familiar with how to maintain alignment in the postoperative patient. Evaluation of the effectiveness of pain medications, assessment of neurologic function, and evaluation of a patient's readiness to ambulate after surgery require higher level nursing education and scope of practice

An appropriate nursing intervention for a patient who has acute low back pain and muscle spasms is to teach the patient to a. keep both feet flat on the floor when prolonged standing is required. b. twist gently from side to side to maintain range of motion in the spine. c. keep the head elevated slightly and flex the knees when resting in bed. d. avoid the use of cold packs because they will exacerbate the muscle spasms.

ANS: C Resting with the head elevated and knees flexed will reduce the strain on the back and decrease muscle spasms. Twisting from side to side will increase tension on the lumbar area. A pillow placed under the upper back will cause strain on the lumbar spine. Alternate application of cold and heat should be used to decrease pain

The nurse designs a program to decrease the incidence of human immunodeficiency virus (HIV) infection in the adolescent and young adult populations. Which information should the nurse assign as the highest priority? a. Methods to prevent perinatal HIV transmission b. Ways to sterilize needles used by injectable drug users c. Prevention of HIV transmission between sexual partners d. Means to prevent transmission through blood transfusions

ANS: C Sexual transmission is the most common way that HIV is transmitted. The nurse should also provide teaching about perinatal transmission, needle sterilization, and blood transfusion, but the rate of HIV infection associated with these situations is lower

Which patient should the nurse assess first? a. Patient with urticaria after receiving an IV antibiotic b. Patient who has graft-versus-host disease and severe diarrhea c. Patient who is sneezing after having subcutaneous immunotherapy d. Patient with multiple chemical sensitivities who has muscle stiffness

ANS: C Sneezing after subcutaneous immunotherapy may indicate impending anaphylaxis and assessment and emergency measures should be initiated. The other patients also have findings that need assessment and intervention by the nurse, but do not have evidence of life-threatening complications

A 49-year-old man who has type 2 diabetes, high blood pressure, hyperlipidemia, and gastroesophageal reflux tells the nurse that he has had recent difficulty in achieving an erection. Which of the following drugs from his current medications list may cause erectile dysfunction (ED)? a. Ranitidine (Zantac) b. Atorvastatin (Lipitor) c. Propranolol (Inderal) d. Metformin (Glucophage)

ANS: C Some antihypertensives may cause erectile dysfunction, and the nurse should anticipate a change in antihypertensive therapy. The other medications will not affect erectile function

Which statement by a patient would alert the nurse to a possible immunodeficiency disorder? a. "I take one baby aspirin every day to prevent stroke." b. "I usually eat eggs or meat for at least 2 meals a day." c. "I had my spleen removed many years ago after a car accident." d. "I had a chest x-ray 6 months ago when I had walking pneumonia."

ANS: C Splenectomy increases the risk for septicemia from bacterial infections. The patient's protein intake is good and should improve immune function. Daily aspirin use does not affect immune function. A chest x-ray does not have enough radiation to suppress immune function

A nurse is weaning a 68-kg male patient who has chronic obstructive pulmonary disease (COPD) from mechanical ventilation. Which patient assessment finding indicates that the weaning protocol should be stopped? a. The patient's heart rate is 97 beats/min. b. The patient's oxygen saturation is 93%. c. The patient respiratory rate is 32 breaths/min. d. The patient's spontaneous tidal volume is 450 mL.

ANS: C Tachypnea is a sign that the patient's work of breathing is too high to allow weaning to proceed. The patient's heart rate is within normal limits, although the nurse should continue to monitor it. An oxygen saturation of 93% is acceptable for a patient with COPD. A spontaneous tidal volume of 450 mL is within the acceptable range

The nurse is caring for a patient who smokes 2 packs/day. To reduce the patient's risk of lung cancer, which action by the nurse is best? a. Teach the patient about the seven warning signs of cancer. b. Plan to monitor the patient's carcinoembryonic antigen (CEA) level. c. Discuss the risks associated with cigarettes during every patient encounter. d. Teach the patient about the use of annual chest x-rays for lung cancer screening.

ANS: C Teaching about the risks associated with cigarette smoking is recommended at every patient encounter because cigarette smoking is associated with multiple health problems. A tumor must be at least 0.5 cm large before it is detectable by current screening methods and may already have metastasized by that time. Oncofetal antigens such as CEA may be used to monitor therapy or detect tumor reoccurrence, but are not helpful in screening for cancer. The seven warning signs of cancer are actually associated with fairly advanced disease

After change-of-shift report on the oncology unit, which patient should the nurse assess first? a. Patient who has a platelet count of 82,000/µL after chemotherapy b. Patient who has xerostomia after receiving head and neck radiation c. Patient who is neutropenic and has a temperature of 100.5° F (38.1° C) d. Patient who is worried about getting the prescribed long-acting opioid on time

ANS: C Temperature elevation is an emergency in neutropenic patients because of the risk for rapid progression to severe infections and sepsis. The other patients also require assessments or interventions, but do not need to be assessed as urgently. Patients with thrombocytopenia do not have spontaneous bleeding until the platelets are 20,000/µL. Xerostomia does not require immediate intervention. Although breakthrough pain needs to be addressed rapidly, the patient does not appear to have breakthrough pain

The nurse notes the presence of white lesions that resemble milk curds in the back of a patient's throat. Which question by the nurse is appropriate at this time? a. "Do you have a productive cough?" b. "How often do you brush your teeth?" c. "Are you taking any medications at present?" d. "Have you ever had an oral herpes infection?"

ANS: C The appearance of the lesions is consistent with an oral candidiasis (thrush) infection, which can occur in patients who are taking medications such as immunosuppressants or antibiotics. Candidiasis is not associated with poor oral hygiene or lower respiratory infections. The lesions do not look like an oral herpes infection

A patient who slipped and fell in the shower at home has a proximal humerus fracture immobilized with a left-sided long-arm cast and a sling. Which nursing intervention will be included in the plan of care? a. Use surgical net dressing to hang the arm from an IV pole. b. Immobilize the fingers of the left hand with gauze dressings. c. Assess the left axilla and change absorbent dressings as needed. d. Assist the patient in passive range of motion (ROM) for the right arm.

ANS: C The axilla can become excoriated when a sling is used to support the arm, and the nurse should check the axilla and apply absorbent dressings to prevent this. A patient with a sling would not have traction applied by hanging. The patient will be encouraged to move the fingers on the injured arm to maintain function and to help decrease swelling. The patient will do active ROM on the uninjured side

The patient with neurogenic shock is receiving a phenylephrine (Neo-Synephrine) infusion through a right forearm IV. Which assessment finding obtained by the nurse indicates a need for immediate action? a. The patient's heart rate is 58 beats/minute. b. The patient's extremities are warm and dry. c. The patient's IV infusion site is cool and pale. d. The patient's urine output is 28 mL over the last hour.

ANS: C The coldness and pallor at the infusion site suggest extravasation of the phenylephrine. The nurse should discontinue the IV and, if possible, infuse the medication into a central line. An apical pulse of 58 is typical for neurogenic shock but does not indicate an immediate need for nursing intervention. A 28-mL urinary output over 1 hour would require the nurse to monitor the output over the next hour, but an immediate change in therapy is not indicated. Warm, dry skin is consistent with early neurogenic shock, but it does not indicate a need for a change in therapy or immediate action

The nurse cares for an adolescent patient who is dying. The patient's parents are interested in organ donation and ask the nurse how the decision about brain death is made. Which response by the nurse is most appropriate? a. "Brain death occurs if a person is flaccid and unresponsive." b. "If CPR is ineffective in restoring a heartbeat, the brain cannot function." c. "Brain death has occurred if there is no breathing and certain reflexes are absent." d. "If respiratory efforts cease and no apical pulse is audible, brain death is present."

ANS: C The diagnosis of brain death is based on irreversible loss of all brain functions, including brainstem functions that control respirations and brainstem reflexes. The other descriptions describe other clinical manifestations associated with death but are insufficient to declare a patient brain dead

A patient is undergoing psoralen plus ultraviolet A light (PUVA) therapy for treatment of psoriasis. What action should the nurse take to prevent adverse effects from this procedure? a. Cleanse the skin carefully with an antiseptic soap. b. Shield any unaffected areas with lead-lined drapes. c. Have the patient use protective eyewear while receiving PUVA. d. Apply petroleum jelly to the areas surrounding the psoriatic lesions.

ANS: C The eyes should be shielded from UV light (UVL) during and after PUVA therapy to prevent the development of cataracts. The patient should be taught about the effects of UVL on unaffected skin, but lead-lined drapes, use of antiseptic soap, and petroleum jelly are not used to prevent skin damage

A patient who arrives at the emergency department experiencing severe left knee pain is diagnosed with a patellar dislocation. The initial patient teaching by the nurse will focus on the need for a. a knee immobilizer. b. gentle knee flexion. c. monitored anesthesia care. d. physical activity restrictions.

ANS: C The first goal of collaborative management is realignment of the knee to its original anatomic position, which will require anesthesia or monitored anesthesia care (MAC), formerly called conscious sedation. Immobilization, gentle range-of-motion (ROM) exercises, and discussion about activity restrictions will be implemented after the knee is realigned

A high school teacher with ulnar drift caused by rheumatoid arthritis (RA) is scheduled for a left hand arthroplasty. Which patient statement to the nurse indicates a realistic expectation for the surgery? a. "This procedure will correct the deformities in my fingers." b. "I will not have to do as many hand exercises after the surgery." c. "I will be able to use my fingers with more flexibility to grasp things." d. "My fingers will appear more normal in size and shape after this surgery."

ANS: C The goal of hand surgery in RA is to restore function, not to correct for cosmetic deformity or treat the underlying process. Hand exercises will be prescribed after the surgery.

When visiting a hospice patient, the nurse assesses that the patient has a respiratory rate of 11 breaths/minute and complains of severe pain. Which action is best for the nurse to take? a. Inform the patient that increasing the morphine will cause the respiratory drive to fail. b. Tell the patient that additional morphine can be administered when the respirations are 12. c. Titrate the prescribed morphine dose upward until the patient indicates adequate pain relief. d. Administer a nonopioid analgesic, such as a nonsteroidal antiinflammatory drug (NSAID), to improve patient pain control.

ANS: C The goal of opioid use in terminally ill patients is effective pain relief regardless of adverse effects such as respiratory depression. A nonopioid analgesic like ibuprofen would not provide adequate analgesia or be absorbed quickly. The rule of double effect provides ethical justification for administering an increased morphine dose to provide effective pain control even though the morphine may further decrease the patient's respiratory rate.

The nurse is caring for a 20-year-old patient with pelvic inflammatory disease (PID) requiring hospitalization. Which nursing intervention will be included in the plan of care? a. Monitor liver function tests. b. Use cold packs PRN for pelvic pain. c. Elevate the head of the bed to at least 30 degrees. d. Teach the patient how to perform Kegel exercises.

ANS: C The head of the bed should be elevated to at least 30 degrees to promote drainage of the pelvic cavity and prevent abscess formation higher in the abdomen. Although a possible complication of PID is acute perihepatitis, liver function tests will remain normal. There is no indication for increased fluid intake. Application of heat is used to reduce pain. Kegel exercises are not helpful in PID

Which assessment finding obtained by the nurse when caring for a patient receiving mechanical ventilation indicates the need for suctioning? a. The patient's oxygen saturation is 93%. b. The patient was last suctioned 6 hours ago. c. The patient's respiratory rate is 32 breaths/minute. d. The patient has occasional audible expiratory wheezes.

ANS: C The increase in respiratory rate indicates that the patient may have decreased airway clearance and requires suctioning. Suctioning is done when patient assessment data indicate that it is needed, not on a scheduled basis. Occasional expiratory wheezes do not indicate poor airway clearance, and suctioning the patient may induce bronchospasm and increase wheezing. An oxygen saturation of 93% is acceptable and does not suggest that immediate suctioning is needed

Following a motorcycle accident, a 58-year-old patient arrives in the emergency department with massive left lower leg swelling. Which action will the nurse take first? a. Elevate the leg on 2 pillows. b. Apply a compression bandage. c. Check leg pulses and sensation. d. Place ice packs on the lower leg.

ANS: C The initial action by the nurse will be to assess the circulation to the leg and to observe for any evidence of injury such as fractures or dislocations. After the initial assessment, the other actions may be appropriate, based on what is observed during the assessment

A pedestrian who was hit by a car is admitted to the emergency department with possible right lower leg fractures. The initial action by the nurse should be to a. elevate the right leg. b. splint the lower leg. c. check the pedal pulses. d. verify tetanus immunizations.

ANS: C The initial nursing action should be assessment of the neurovascular status of the injured leg. After assessment, the nurse may need to splint and elevate the leg, based on the assessment data. Information about tetanus immunizations should be done if there is an open wound

After the health care provider has recommended amputation for a patient who has nonhealing ischemic foot ulcers, the patient tells the nurse that he would rather die than have an amputation. Which response by the nurse is best? a. "You are upset, but you may lose the foot anyway." b. "Many people are able to function with a foot prosthesis." c. "Tell me what you know about your options for treatment." d. "If you do not want an amputation, you do not have to have it."

ANS: C The initial nursing action should be to assess the patient's knowledge level and feelings about the options available. Discussion about the patient's option to not have the procedure, the seriousness of the condition, or rehabilitation after the procedure may be appropriate after the nurse knows more about the patient's current level of knowledge and emotional state

A 47-year-old woman asks whether she is going into menopause if she has not had a menstrual period for 3 months. The best response by the nurse is which of the following? a. "Have you thought about using hormone replacement therapy?" b. "Most women feel a little depressed about entering menopause." c. "What was your menstrual pattern before your periods stopped?" d. "Since you are in your mid-40s, it is likely that you are menopausal."

ANS: C The initial response by the nurse should be to assess the patient's baseline menstrual pattern. Although many women do enter menopause in the mid-40s, more information about this patient is needed before telling her that it is likely she is menopausal. Although hormone therapy (HT) may be prescribed, further assessment of the patient is needed before discussing therapies for menopause. Because the response to menopause is very individual, the nurse should not assume that the patient is experiencing any adverse emotional reactions.

Which action is a priority for the nurse to take when the low pressure alarm sounds for a patient who has an arterial line in the left radial artery? a. Fast flush the arterial line. b. Check the left hand for pallor. c. Assess for cardiac dysrhythmias. d. Rezero the monitoring equipment.

ANS: C The low pressure alarm indicates a drop in the patient's blood pressure, which may be caused by cardiac dysrhythmias. There is no indication to rezero the equipment. Pallor of the left hand would be caused by occlusion of the radial artery by the arterial catheter, not by low pressure. There is no indication of a need for flushing the line

A patient who is scheduled for a right breast biopsy asks the nurse the difference between a benign tumor and a malignant tumor. Which answer by the nurse is correct? a. "Benign tumors do not cause damage to other tissues." b. "Benign tumors are likely to recur in the same location." c. "Malignant tumors may spread to other tissues or organs." d. "Malignant cells reproduce more rapidly than normal cells."

ANS: C The major difference between benign and malignant tumors is that malignant tumors invade adjacent tissues and spread to distant tissues and benign tumors never metastasize. The other statements are inaccurate. Both types of tumors may cause damage to adjacent tissues. Malignant cells do not reproduce more rapidly than normal cells. Benign tumors do not usually recur

Which of these patients being seen at the human immunodeficiency virus (HIV) clinic should the nurse assess first? a. Patient whose latest CD4+ count is 250/µL b. Patient whose rapid HIV-antibody test is positive c. Patient who has had 10 liquid stools in the last 24 hours d. Patient who has nausea from prescribed antiretroviral drugs

ANS: C The nurse should assess the patient for dehydration and hypovolemia. The other patients also will require assessment and possible interventions, but do not require immediate action to prevent complications such as hypovolemia and shock

During a routine health examination, a 40-year-old patient tells the nurse about a family history of colon cancer. Which action should the nurse take next? a. Teach the patient about the need for a colonoscopy at age 50. b. Teach the patient how to do home testing for fecal occult blood. c. Obtain more information from the patient about the family history. d. Schedule a sigmoidoscopy to provide baseline data about the patient.

ANS: C The patient may be at increased risk for colon cancer, but the nurse's first action should be further assessment. The other actions may be appropriate, depending on the information that is obtained from the patient with further questioning.

During the teaching session for a patient who has a new diagnosis of acute leukemia the patient is restless and is looking away, never making eye contact. After teaching about the complications associated with chemotherapy, the patient asks the nurse to repeat all of the information. Based on this assessment, which nursing diagnosis is most appropriate for the patient? a. Risk for ineffective adherence to treatment related to denial of need for chemotherapy b. Acute confusion related to infiltration of leukemia cells into the central nervous system c. Risk for ineffective health maintenance related to anxiety about new leukemia diagnosis d. Deficient knowledge: chemotherapy related to a lack of interest in learning about treatment

ANS: C The patient who has a new cancer diagnosis is likely to have high anxiety, which may impact learning and require that the nurse repeat and reinforce information. The patient's history of a recent diagnosis suggests that infiltration of the leukemia is not a likely cause of the confusion. The patient asks for the information to be repeated, indicating that lack of interest in learning and denial are not etiologic factors

Which information regarding a patient's sleep is most important for the nurse to communicate to the health care provider? a. 64-year-old nurse who works the night shift reports drinking hot chocolate before going to bed in the morning b. 21-year-old student who takes melatonin to assist in sleeping when traveling from the United States to Europe c. 41-year-old librarian who has a body mass index (BMI) of 42 kg/m2 says that the spouse complains about snoring d. 32-year-old accountant who is experiencing a stressful week uses diphenhydramine (Benadryl) for several nights

ANS: C The patient's BMI and snoring suggest possible sleep apnea, which can cause complications such as cardiac dysrhythmias, hypertension, and right-sided heart failure. Melatonin is safe to use as a therapy for jet lag. Short-term use of diphenhydramine in young adults is not a concern. Hot chocolate contains only 5 mg of caffeine and is unlikely to affect this patient's sleep quality

While obtaining a health history from a patient, the nurse learns that the patient has a history of allergic rhinitis and multiple food allergies. Which action by the nurse is most appropriate? a. Encourage the patient to carry an epinephrine kit in case a type IV allergic reaction to latex develops. b. Advise the patient to use oil-based hand creams to decrease contact with natural proteins in latex gloves. c. Document the patient's allergy history and be alert for any clinical manifestations of a type I latex allergy. d. Recommend that the patient use vinyl gloves instead of latex gloves in preventing blood-borne pathogen contact.

ANS: C The patient's allergy history and occupation indicate a risk of developing a latex allergy. The nurse should be prepared to manage any symptoms that may occur. Epinephrine is not an appropriate treatment for contact dermatitis that is caused by a type IV allergic reaction to latex. Oil-based creams will increase the exposure to latex from latex gloves. Vinyl gloves are appropriate to use when exposure to body fluids is unlikely

The following patients call the outpatient clinic. Which phone call should the nurse return first? a. A 44-year-old patient who has bloody discharge after a hysteroscopy earlier today b. A 64-year-old patient who is experiencing shoulder pain after a laparoscopy yesterday c. A 34-year-old patient who is short of breath after pelvic computed tomography (CT) with contrast d. A 54-year-old patient who has severe breast tenderness following a needle aspiration breast biopsy

ANS: C The patient's dyspnea suggests a delayed reaction to the iodine dye used for the CT scan. The other patient's symptoms are not unusual after the procedures they had done.

The nurse documents the vital signs for a patient admitted 2 days ago with gram-negative sepsis: temperature 101.2° F, blood pressure 90/56 mm Hg, pulse 92, respirations 34. Which action should the nurse take next? a. Give the scheduled IV antibiotic. b. Give the PRN acetaminophen (Tylenol). c. Obtain oxygen saturation using pulse oximetry. d. Notify the health care provider of the patient's vital signs.

ANS: C The patient's increased respiratory rate in combination with the admission diagnosis of gram-negative sepsis indicates that acute respiratory distress syndrome (ARDS) may be developing. The nurse should check for hypoxemia, a hallmark of ARDS. The health care provider should be notified after further assessment of the patient. Giving the scheduled antibiotic and the PRN acetaminophen will also be done, but they are not the highest priority for a patient who may be developing ARDS

The nurse reviews the medication administration record in order to choose the most appropriate pain medication for a patient with cancer who describes the pain as "deep, aching and at a level 8 on a 0 to 10 scale". Which medication should the nurse administer? a. Fentanyl (Duragesic) patch b. Ketorolac (Toradol) tablets c. Hydromorphone (Dilaudid) IV d. Acetaminophen (Tylenol) suppository

ANS: C The patient's pain level indicates that a rapidly acting medication such as an IV opioid is needed. The other medications also may be appropriate to use, but will not work as rapidly or as effectively as the IV hydromorphone

A patient who has been diagnosed with inoperable lung cancer and has a poor prognosis plans a trip across the country "to settle some issues with sisters and brothers." The nurse recognizes that the patient is manifesting which psychosocial response to death? a. Restlessness b. Yearning and protest c. Anxiety about unfinished business d. Fear of the meaninglessness of one's life

ANS: C The patient's statement indicates that there is some unfinished family business that the patient would like to address before dying. Restlessness is frequently a behavior associated with an inability to express emotional or physical distress, but this patient does not express distress and is able to communicate clearly. There is no indication that the patient is protesting the prognosis, or that there is any fear that the patient's life has been meaningless

A female patient tells the nurse that she has been having nightmares and acute anxiety around men since being sexually assaulted 3 months ago. The most appropriate nursing diagnosis for the patient is a. anxiety related to effects of being raped. b. sleep deprivation related to frightening dreams. c. rape-trauma syndrome related to rape experience. d. ineffective coping related to inability to resolve incident.

ANS: C The patient's symptoms are most consistent with the nursing diagnosis of rape-trauma syndrome. The nursing diagnoses of sleep deprivation, ineffective coping, and anxiety address some aspects of the patient's symptoms but do not address the problem as completely as the rape-trauma syndrome diagnosis.

A patient returning from surgery for a perineal radical prostatectomy will have a nursing diagnosis of risk for infection related to a. urinary incontinence. b. prolonged urinary stasis. c. possible fecal wound contamination. d. placement of a suprapubic bladder catheter.

ANS: C The perineal approach increases the risk for infection because the incision is located close to the anus and contamination with feces is possible. Urinary stasis and incontinence do not occur because the patient has a retention catheter in place for 1 to 2 weeks. A urethral catheter is used after the surgery

The nurse should reposition the patient who has just had a laminectomy and diskectomy by a. instructing the patient to move the legs before turning the rest of the body. b. having the patient turn by grasping the side rails and pulling the shoulders over. c. placing a pillow between the patient's legs and turning the entire body as a unit. d. turning the patient's head and shoulders first, followed by the hips, legs, and feet.

ANS: C The spine should be kept in correct alignment after laminectomy. The other positions will create misalignment of the spine

The nurse in the infertility clinic is explaining in vitro fertilization (IVF) to a couple. The woman tells the nurse that they cannot afford IVF on her husband's salary. The man replies that if his wife worked outside the home, they would have enough money. Which nursing diagnosis is appropriate? a. Decisional conflict related to inadequate financial resources b. Ineffective sexuality patterns related to psychological stress c. Defensive coping related to anxiety about lack of conception d. Ineffective denial related to frustration about continued infertility

ANS: C The statements made by the couple are consistent with the diagnosis of defensive coping. No data indicate that ineffective sexuality and ineffective denial are problems. Although the couple is quarreling about finances, the data do not provide information indicating that the finances are inadequate

Which statement by a 24-year-old patient indicates that the nurse's teaching about management of primary genital herpes has been effective? a. "I will use acyclovir ointment on the area to relieve the pain." b. "I will use condoms for intercourse until the medication is all gone." c. "I will take the acyclovir (Zovirax) every 8 hours for the next week." d. "I will need to take all of the medication to be sure the infection is cured."

ANS: C The treatment regimen for primary genital herpes infections includes acyclovir 400 mg 3 times daily for 7 to 10 days. The patient is taught to abstain from intercourse until the lesions are gone. (Condoms should be used even when the patient is asymptomatic.) Acyclovir ointment is not effective in treating lesions or reducing pain. Herpes infection is chronic and recurrent

The charge nurse is assigning rooms for new admissions. Which patient would be the most appropriate roommate for a patient who has acute rejection of an organ transplant? a. A patient who has viral pneumonia b. A patient with second-degree burns c. A patient who is recovering from an anaphylactic reaction to a bee sting d. A patient with graft-versus-host disease after a recent bone marrow transplant

ANS: C Treatment for a patient with acute rejection includes administration of additional immunosuppressants, and the patient should not be exposed to increased risk for infection as would occur from patients with viral pneumonia, graft-versus-host disease, and burns. There is no increased exposure to infection from a patient who had an anaphylactic reaction

A patient with metastatic cancer of the colon experiences severe vomiting following each administration of chemotherapy. Which action, if taken by the nurse, is most appropriate? a. Have the patient eat large meals when nausea is not present. b. Offer dry crackers and carbonated fluids during chemotherapy. c. Administer prescribed antiemetics 1 hour before the treatments. d. Give the patient two ounces of a citrus fruit beverage during treatments.

ANS: C Treatment with antiemetics before chemotherapy may help prevent nausea. The patient should eat small, frequent meals. Offering food and beverages during chemotherapy is likely to cause nausea. The acidity of citrus fruits may be further irritating to the stomach

The nurse assesses that a patient receiving epidural morphine has not voided for over 10 hours. What action should the nurse take initially? a. Monitor for withdrawal symptoms. b. Place an indwelling urinary catheter. c. Ask if the patient feels the need to void. d. Document this allergic reaction in the patient's chart.

ANS: C Urinary retention is a common side effect of epidural opioids. Assess whether the patient feels the need to void. Since urinary retention is a possible side effect, there is no reason for concern of withdrawal symptoms. Placing an indwelling catheter requires an order from the health care provider. Usually an in and out catheter is performed to empty the bladder if the patient is unable to void because of the risk of infection with an indwelling catheter. Urinary retention does not indicate that this reaction is an allergic reaction

The nurse is caring for a patient receiving a continuous norepinephrine (Levophed) IV infusion. Which patient assessment finding indicates that the infusion rate may need to be adjusted? a. Heart rate is 58 beats/minute. b. Mean arterial pressure (MAP) is 56 mm Hg. c. Systemic vascular resistance (SVR) is elevated. d. Pulmonary artery wedge pressure (PAWP) is low.

ANS: C Vasoconstrictors such as norepinephrine (Levophed) will increase SVR, and this will increase the work of the heart and decrease peripheral perfusion. The infusion rate may need to be decreased. Bradycardia, hypotension (MAP of 56 mm Hg), and low PAWP are not associated with norepinephrine infusion

Which information will the nurse include when teaching an older patient about skin care? a. Dry the skin thoroughly before applying lotions. b. Bathe and wash hair daily with soap and shampoo. c. Use warm water and a moisturizing soap when bathing. d. Use antibacterial soaps when bathing to avoid infection.

ANS: C Warm water and moisturizing soap will avoid overdrying the skin. Because older patients have dryer skin, daily bathing and shampooing are not necessary and may dry the skin unnecessarily. Antibacterial soaps are not necessary. Lotions should be applied while the skin is still damp to seal moisture in

A nurse is assessing a patient who is receiving a nitroprusside (Nipride) infusion to treat cardiogenic shock. Which finding indicates that the medication is effective? a. No new heart murmurs b. Decreased troponin level c. Warm, pink, and dry skin d. Blood pressure 92/40 mm Hg

ANS: C Warm, pink, and dry skin indicates that perfusion to tissues is improved. Since nitroprusside is a vasodilator, the blood pressure may be low even if the medication is effective. Absence of a heart murmur and a decrease in troponin level are not indicators of improvement in shock.

Which information will the nurse include when teaching a patient with acute low back pain (select all that apply)? a. Sleep in a prone position with the legs extended. b. Keep the knees straight when leaning forward to pick something up. c. Avoid activities that require twisting of the back or prolonged sitting. d. Symptoms of acute low back pain frequently improve in a few weeks. e. Ibuprofen (Motrin, Advil) or acetaminophen (Tylenol) can be used to relieve pain.

ANS: C, D, E Acute back pain usually starts to improve within 2 weeks. In the meantime, the patient should use medications such as nonsteroidal antiinflammatory drugs (NSAIDs) or acetaminophen to manage pain and avoid activities that stress the back. Sleeping in a prone position and keeping the knees straight when leaning forward will place stress on the back, and should be avoided

Which information obtained by the nurse about an older adult who complains of occasional insomnia indicates a need for patient teaching (select all that apply)? a. Drinks a cup of coffee every morning with breakfast b. Has a snack every evening 1 hour before going to bed c. Likes to read or watch television in bed on most evenings d. Usually takes a warm bath just before bedtime every night e. Occasionally uses diphenhydramine (Benadryl) as a sleep aid

ANS: C, E Reading and watching television in bed may contribute to insomnia. Older adults should avoid the use of medications that have anticholinergic effects, such as diphenhydramine. Having a snack 1 hour before bedtime and/or coffee early in the day should not affect sleep quality. Rituals such as a warm bath before bedtime can enhance sleep quality

A patient with a large stomach tumor that is attached to the liver is scheduled to have a debulking procedure. Which information should the nurse teach the patient about the outcome of this procedure? a. Pain will be relieved by cutting sensory nerves in the stomach. b. Relief of pressure in the stomach will promote better nutrition. c. Tumor growth will be controlled by the removal of malignant tissue. d. Tumor size will decrease and this will improve the effects of other therapy.

ANS: D A debulking surgery reduces the size of the tumor and makes radiation and chemotherapy more effective. Debulking surgeries do not control tumor growth. The tumor is debulked because it is attached to the liver, a vital organ (not to relieve pressure on the stomach). Debulking does not sever the sensory nerves, although pain may be lessened by the reduction in pressure on the abdominal organs

A patient with chronic obstructive pulmonary disease (COPD) arrives in the emergency department complaining of shortness of breath and dyspnea on minimal exertion. Which assessment finding by the nurse is most important to report to the health care provider? a. The patient has bibasilar lung crackles. b. The patient is sitting in the tripod position. c. The patient's respirations have decreased from 30 to 10 breaths/minute. d. The patient's pulse oximetry indicates an O2 saturation of 91%.

ANS: D A decrease in respiratory rate in a patient with respiratory distress suggests the onset of fatigue and a high risk for respiratory arrest. Therefore immediate action such as positive pressure ventilation is needed. Patients who are experiencing respiratory distress frequently sit in the tripod position because it decreases the work of breathing. Crackles in the lung bases may be the baseline for a patient with COPD. An oxygen saturation of 91% is common in patients with COPD and will provide adequate gas exchange and tissue oxygenation

Which finding is of highest priority when the nurse is planning care for a 77-year-old patient seen in the outpatient clinic? a. Symmetric joint swelling of fingers b. Decreased right knee range of motion c. Report of left hip aching when jogging d. History of recent loss of balance and fall

ANS: D A history of falls requires further assessment and development of fall prevention strategies. The other changes are more typical of bone and joint changes associated with normal aging.

The nurse finds that a patient can flex the arms when no resistance is applied but is unable to flex when the nurse applies light resistance. The nurse should document the patient's muscle strength as level a. 0. b. 1. c. 2. d. 3.

ANS: D A level 3 indicates that the patient is unable to move against resistance but can move against gravity. Level 1 indicates minimal muscle contraction, level 2 indicates that the arm can move when gravity is eliminated, and level 4 indicates active movement with some resistance

A patient who is receiving immunotherapy has just received an allergen injection. Which assessment finding is most important to communicate to the health care provider? a. The patient's IgG level is increased. b. The injection site is red and swollen. c. The patient's allergy symptoms have not improved. d. There is a 2-cm wheal at the site of the allergen injection.

ANS: D A local reaction larger than quarter size may indicate that a decrease in the allergen dose is needed. An increase in IgG indicates that the therapy is effective. Redness and swelling at the site are not unusual. Because immunotherapy usually takes 1 to 2 years to achieve an effect, an improvement in the patient's symptoms is not expected after a few months

The day after a 60-year-old patient has an open reduction and internal fixation (ORIF) for an open, displaced tibial fracture, the priority nursing diagnosis is a. activity intolerance related to deconditioning. b. risk for constipation related to prolonged bed rest. c. risk for impaired skin integrity related to immobility. d. risk for infection related to disruption of skin integrity.

ANS: D A patient having an ORIF is at risk for problems such as wound infection and osteomyelitis. After an ORIF, patients typically are mobilized starting the first postoperative day, so problems caused by immobility are not as likely.

A patient with chronic insomnia asks the nurse about ways to improve sleep quality. What is the nurse's best response? a. Avoid aerobic exercise during the day. b. Read in bed for a few minutes each night. c. Keep the bedroom temperature slightly warm. d. Try to go to bed at the same time every evening.

ANS: D A regular evening schedule is recommended to improve sleep time and quality. Aerobic exercise may improve sleep quality but should occur at least 6 hours before bedtime. Reading in bed is discouraged for patients with insomnia. The bedroom temperature should be slightly cool.

Which action will the nurse include in the plan of care for a patient who has had a total right knee arthroplasty? a. Avoid extension of the right knee beyond 120 degrees. b. Use a compression bandage to keep the right knee flexed. c. Teach about the need to avoid weight bearing for 4 weeks. d. Start progressive knee exercises to obtain 90-degree flexion.

ANS: D After knee arthroplasty, active or passive flexion exercises are used to obtain a 90-degree flexion of the knee. The goal for extension of the knee will be 180 degrees. A compression bandage is used to hold the knee in an extended position after surgery. Full weight bearing is expected before discharge.

Which assessment finding for a patient who has had a surgical reduction of an open fracture of the right radius is most important to report to the health care provider? a. Serous wound drainage b. Right arm muscle spasms c. Right arm pain with movement d. Temperature 101.4° F (38.6° C)

ANS: D An elevated temperature is suggestive of possible osteomyelitis. The other clinical manifestations are typical after a repair of an open fracture

The nurse obtains information about a hospitalized patient who is receiving chemotherapy for colorectal cancer. Which information about the patient alerts the nurse to discuss a possible change in therapy with the health care provider? a. Poor oral intake b. Frequent loose stools c. Complaints of nausea and vomiting d. Increase in carcinoembryonic antigen (CEA)

ANS: D An increase in CEA indicates that the chemotherapy is not effective for the patient's cancer and may need to be modified. The other patient findings are common adverse effects of chemotherapy. The nurse may need to address these, but they would not necessarily indicate a need for a change in therapy

An older adult patient with a squamous cell carcinoma (SCC) on the lower arm has a Mohs procedure in the dermatology clinic. Which nursing action will be included in the postoperative plan of care? a. Describe the use of topical fluorouracil on the incision. b. Teach how to use sterile technique to clean the suture line. c. Schedule daily appointments for wet-to-dry dressing changes. d. Teach about the use of cold packs to reduce bruising and swelling.

ANS: D Application of cold packs to the incision after the surgery will help decrease bruising and swelling at the site. Since the Mohs procedure results in complete excision of the lesion, topical fluorouracil is not needed after surgery. After the Mohs procedure the edges of the wound can be left open to heal or the edges can be approximated and sutured together. The suture line can be cleaned with tap water. No debridement with wet-to-dry dressings is indicated

When caring for patients with sleep disorders, which activity can the nurse appropriately delegate to unlicensed assistive personnel (UAP)? a. Interview a new patient about risk factors for obstructive sleep disorders. b. Discuss the benefits of oral appliances in decreasing obstructive sleep apnea. c. Help a patient choose an appropriate continuous positive airway pressure (CPAP) mask. d. Assist a patient to place the CPAP device correctly over the nose and mouth at bedtime.

ANS: D Because CPAP mask placement is consistently done in the same way, this is appropriate to delegate to UAP. The other actions require critical thinking and nursing judgment, and should be done by the RN.

A patient with acute respiratory distress syndrome (ARDS) who is intubated and receiving mechanical ventilation develops a right pneumothorax. Which action will the nurse anticipate taking next? a. Increase the tidal volume and respiratory rate. b. Increase the fraction of inspired oxygen (FIO2). c. Perform endotracheal suctioning more frequently. d. Lower the positive end-expiratory pressure (PEEP).

ANS: D Because barotrauma is associated with high airway pressures, the level of PEEP should be decreased. The other actions will not decrease the risk for pneumothorax

A 44-year-old patient in the sexually transmitted infection clinic has a positive Venereal Disease Research Laboratory (VDRL) test, but no chancre is visible on assessment. The nurse will plan to send specimens for a. gram stain. b. cytologic studies. c. rapid plasma reagin (RPR) agglutination. d. fluorescent treponemal antibody absorption (FTA-Abs).

ANS: D Because false positives are common with VDRL and RPR testing, FTA-Abs testing is recommended to confirm a diagnosis of syphilis. Gram staining is used for other sexually transmitted infections (STIs) such as gonorrhea and Chlamydia and cytologic studies are used to detect abnormal cells (such as neoplastic cells).

The nurse cares for a patient who is human immunodeficiency virus (HIV) positive and taking antiretroviral therapy (ART). Which information is most important for the nurse to address when planning care? a. The patient's blood glucose level is 142 mg/dL. b. The patient complains of feeling "constantly tired." c. The patient is unable to state the side effects of the medications. d. The patient states, "Sometimes I miss a dose of zidovudine (AZT)."

ANS: D Because missing doses of ART can lead to drug resistance, this patient statement indicates the need for interventions such as teaching or changes in the drug scheduling. Elevated blood glucose and fatigue are common side effects of ART. The nurse should discuss medication side effects with the patient, but this is not as important as addressing the skipped doses of AZT

Which assessment information is most important for the nurse to obtain to evaluate whether treatment of a patient with anaphylactic shock has been effective? a. Heart rate b. Orientation c. Blood pressure d. Oxygen saturation

ANS: D Because the airway edema that is associated with anaphylaxis can affect airway and breathing, the oxygen saturation is the most critical assessment. Improvements in the other assessments will also be expected with effective treatment of anaphylactic shock

A patient has been assigned the nursing diagnosis of imbalanced nutrition: less than body requirements related to painful oral ulcers. Which nursing action will be most effective in improving oral intake? a. Offer the patient frequent small snacks between meals. b. Assist the patient to choose favorite foods from the menu. c. Provide teaching about the importance of nutritional intake. d. Apply the ordered anesthetic gel to oral lesions before meals.

ANS: D Because the etiology of the patient's poor nutrition is the painful oral ulcers, the best intervention is to apply anesthetic gel to the lesions before the patient eats. The other actions might be helpful for other patients with impaired nutrition, but would not be as helpful for this patient

A patient has the following risk factors for melanoma. Which risk factor should the nurse assign as the priority focus of patient teaching? a. The patient has multiple dysplastic nevi. b. The patient is fair-skinned and has blue eyes. c. The patient's mother died of a malignant melanoma. d. The patient uses a tanning booth throughout the winter.

ANS: D Because the only risk factor that the patient can change is the use of a tanning booth, the nurse should focus teaching about melanoma prevention on this factor. The other factors also will contribute to increased risk for melanoma

The nurse notes that a patient who has a large cystocele, admitted 10 hours ago, has not yet voided. Which action should the nurse take first? a. Insert a straight catheter per the PRN order. b. Encourage the patient to increase oral fluids. c. Notify the health care provider of the inability to void. d. Use an ultrasound scanner to check for urinary retention.

ANS: D Because urinary retention is common with a large cystocele, the nurse's first action should be to use an ultrasound bladder scanner to check for the presence of urine in the bladder. The other actions may be appropriate, depending on the findings with the bladder scanner

The nurse will most likely prepare a medication teaching plan about antiretroviral therapy (ART) for which patient? a. Patient who is currently HIV negative but has unprotected sex with multiple partners b. Patient who was infected with HIV 15 years ago and now has a CD4+ count of 840/µL c. HIV-positive patient with a CD4+ count of 160/µL who drinks a fifth of whiskey daily d. Patient who tested positive for HIV 2 years ago and now has cytomegalovirus (CMV) retinitis

ANS: D CMV retinitis is an acquired immunodeficiency syndrome (AIDS)-defining illness and indicates that the patient is appropriate for ART even though the HIV infection period is relatively short. An HIV-negative patient would not be offered ART. A patient with a CD4+ count in the normal range would not typically be started on ART. A patient who drinks alcohol heavily would be unlikely to be able to manage the complex drug regimen and would not be appropriate for ART despite the low CD4+ count

The nurse assesses a patient who is receiving interleukin-2. Which finding should the nurse report immediately to the health care provider? a. Generalized muscle aches b. Complaints of nausea and anorexia c. Oral temperature of 100.6° F (38.1° C) d. Crackles heard at the lower scapular border

ANS: D Capillary leak syndrome and acute pulmonary edema are possible toxic effects of interleukin-2. The patient may need oxygen and the nurse should rapidly notify the health care provider. The other findings are common side effects of interleukin-2.

The nurse cares for a terminally ill patient who has 20-second periods of apnea followed by periods of deep and rapid breathing. Which action by the nurse would be most appropriate? a. Suction the patient. b. Administer oxygen via face mask. c. Place the patient in high Fowler's position. d. Document the respirations as Cheyne-Stokes.

ANS: D Cheyne-Stokes respirations are characterized by periods of apnea alternating with deep and rapid breaths. Cheyne-Stokes respirations are expected in the last days of life. There is also no need for supplemental oxygen by face mask or suctioning the patient. Raising the head of the bed slightly and/or turning the patient on the side may promote comfort. There is no need to place the patient in high Fowler's position

A 31-year-old patient has just been instructed in the treatment for a Chlamydia trachomatis vaginal infection. Which patient statement indicates that the nurse's teaching has been effective? a. "I can purchase an over-the-counter medication to treat this infection." b. "The symptoms are due to the overgrowth of normal vaginal bacteria." c. "The medication will need to be inserted once daily with an applicator." d. "Both my partner and I will need to take the medication for a full week."

ANS: D Chlamydia is a sexually transmitted bacterial infection that requires treatment of both partners with antibiotics for 7 days. The other statements are true for the treatment of Candida albicans infection

The nurse is caring for a patient diagnosed with furunculosis. Which nursing action could the nurse delegate to unlicensed assistive personnel (UAP)? a. Applying antibiotic cream to the groin. b. Obtaining cultures from ruptured lesions. c. Evaluating the patient's personal hygiene. d. Cleaning the skin with antimicrobial soap.

ANS: D Cleaning the skin is within the education and scope of practice for UAP. Administration of medication, obtaining cultures, and evaluation are higher-level skills that require the education and scope of practice of licensed nursing personnel

The nurse explains to a 37-year-old patient being prepared for colposcopy with a cervical biopsy that the procedure a. involves dilation of the cervix and biopsy of the tissue lining the uterus. b. will take place in a same-day surgery center so that local anesthesia can be used. c. requires that the patient have nothing to eat or drink for 6 hours before the procedure. d. is similar to a speculum examination of the cervix and should result in little discomfort.

ANS: D Colposcopy involves visualization of the cervix with a binocular microscope and is similar to a speculum examination. Anesthesia is not required and fasting is not necessary. A cervical biopsy may cause a minimal amount of pain

A patient who is human immunodeficiency virus (HIV)-infected has a CD4+ cell count of 400/µL. Which factor is most important for the nurse to determine before the initiation of antiretroviral therapy (ART) for this patient? a. HIV genotype and phenotype b. Patient's social support system c. Potential medication side effects d. Patient's ability to comply with ART schedule

ANS: D Drug resistance develops quickly unless the patient takes ART medications on a strict, regular schedule. In addition, drug resistance endangers both the patient and the community. The other information is also important to consider, but patients who are unable to manage and follow a complex drug treatment regimen should not be considered for ART

Which action will the nurse need to do when preparing to assist with the insertion of a pulmonary artery catheter? a. Determine if the cardiac troponin level is elevated. b. Auscultate heart and breath sounds during insertion. c. Place the patient on NPO status before the procedure. d. Attach cardiac monitoring leads before the procedure.

ANS: D Dysrhythmias can occur as the catheter is floated through the right atrium and ventricle, and it is important for the nurse to monitor for these during insertion. Pulmonary artery catheter insertion does not require anesthesia, and the patient will not need to be NPO. Changes in cardiac troponin or heart and breath sounds are not expected during pulmonary artery catheter insertion

To verify the correct placement of an oral endotracheal tube (ET) after insertion, the best initial action by the nurse is to a. auscultate for the presence of bilateral breath sounds. b. obtain a portable chest x-ray to check tube placement. c. observe the chest for symmetric chest movement with ventilation. d. use an end-tidal CO2 monitor to check for placement in the trachea.

ANS: D End-tidal CO2 monitors are currently recommended for rapid verification of ET placement. Auscultation for bilateral breath sounds and checking chest expansion are also used, but they are not as accurate as end-tidal CO2 monitoring. A chest x-ray confirms the placement but is done after the tube is secured

A patient who has fibromyalgia tells the nurse, "I feel depressed because I ache too much to play golf." The patient says the pain is usually at a level 7 (0 to 10 scale). Which patient goal has the highest priority when the nurse is developing the treatment plan? a. The patient will exhibit fewer signs of depression. b. The patient will say that the aching has decreased. c. The patient will state that pain is at a level 2 of 10. d. The patient will be able to play 1 to 2 rounds of golf.

ANS: D For chronic pain, patients are encouraged to set functional goals such as being able to perform daily activities and hobbies. The patient has identified playing golf as the desired activity, so a pain level of 2 of 10 or a decrease in aching would be less useful in evaluating successful treatment. The nurse should also assess for depression, but the patient has identified the depression as being due to the inability to play golf, so the goal of being able to play 1 or 2 rounds of golf is the most appropriate

An older adult patient has a prescription for cyclosporine following a kidney transplant. Which information in the patient's health history has the most implications for planning patient teaching about the medication at this time? a. The patient restricts salt to treat prehypertension. b. The patient drinks 3 to 4 quarts of fluids every day. c. The patient has many concerns about the effects of cyclosporine. d. The patient has a glass of grapefruit juice every day for breakfast.

ANS: D Grapefruit juice can increase the toxicity of cyclosporine. The patient should be taught to avoid grapefruit juice. High fluid intake will not affect cyclosporine levels or renal function. Cyclosporine may cause hypertension, and the patient's many concerns should be addressed, but these are not potentially life-threatening problems

A nurse assesses a patient with chronic cancer pain who is receiving imipramine (Tofranil) in addition to long-acting morphine. Which statement, if made by the patient, indicates to the nurse that the patient is receiving adequate pain control? a. "I'm not anxious at all." b. "I sleep 8 hours every night." c. "I feel much less depressed since I've been taking the Tofranil." d. "The pain is manageable and I can accomplish my desired activities.

ANS: D Imipramine is being used in this patient to manage chronic pain and improve functional ability. Although the medication is also prescribed for patients with depression, insomnia, and anxiety, the evaluation for this patient is based on improved pain control and activity level

The nurse will anticipate that a 61-year-old patient who has an enlarged prostate detected by digital rectal examination (DRE) and an elevated prostate specific antigen (PSA) level will need teaching about a. cystourethroscopy. b. uroflowmetry studies. c. magnetic resonance imaging (MRI). d. transrectal ultrasonography (TRUS).

ANS: D In a patient with an abnormal DRE and elevated PSA, transrectal ultrasound is used to visualize the prostate for biopsy. Uroflowmetry studies help determine the extent of urine blockage and treatment, but there is no indication that this is a problem for this patient. Cystoscopy may be used before prostatectomy but will not be done until after the TRUS and biopsy. MRI is used to determine whether prostatic cancer has metastasized but would not be ordered at this stage of the diagnostic process

A 49-year-old woman tells the nurse that she is postmenopausal but has occasional spotting. Which initial response by the nurse is most appropriate? a. "A frequent cause of spotting is endometrial cancer." b. "How long has it been since your last menstrual period?" c. "Breakthrough bleeding is not unusual in women your age." d. "Are you using prescription hormone replacement therapy?"

ANS: D In postmenopausal women, a common cause of spotting is hormone therapy (HT). Because breakthrough bleeding may be a sign of problems such as cancer or infection, the nurse would not imply that this is normal. The length of time since the last menstrual period is not relevant to the patient's symptoms. Although endometrial cancer may cause spotting, this information is not appropriate as an initial response

A 32-year-old man who has a profuse, purulent urethral discharge with painful urination is seen at the clinic. Which information will be most important for the nurse to obtain? a. Contraceptive use b. Sexual orientation c. Immunization history d. Recent sexual contacts

ANS: D Information about sexual contacts is needed to help establish whether the patient has been exposed to a sexually transmitted infection (STI) and because sexual contacts also will need treatment. The other information also may be gathered but is not as important in determining the plan of care for the patient's current symptoms

Which nursing interventions included in the care of a mechanically ventilated patient with acute respiratory failure can the registered nurse (RN) delegate to an experienced licensed practical/vocational nurse (LPN/LVN) working in the intensive care unit? a. Assess breath sounds every hour. b. Monitor central venous pressures. c. Place patient in the prone position. d. Insert an indwelling urinary catheter.

ANS: D Insertion of indwelling urinary catheters is included in LPN/LVN education and scope of practice and can be safely delegated to an LPN/LVN who is experienced in caring for critically ill patients. Placing a patient who is on a ventilator in the prone position requires multiple staff, and should be supervised by an RN. Assessment of breath sounds and obtaining central venous pressures require advanced assessment skills and should be done by the RN caring for a critically ill patient

A clinic patient is experiencing an allergic reaction to an unknown allergen. Which action is most appropriate for the registered nurse (RN) to delegate to a licensed practical/vocational nurse (LPN/LVN)? a. Perform a focused physical assessment. b. Obtain the health history from the patient. c. Teach the patient about the various diagnostic studies. d. Administer skin testing by the cutaneous scratch method.

ANS: D LPN/LVNs are educated and licensed to administer medications under the supervision of an RN. RN-level education and the scope of practice include assessment of health history, focused physical assessment, and patient teaching

The nurse is completing the medication reconciliation form for a patient admitted with chronic cancer pain. Which medication is of most concern to the nurse? a. Amitriptyline (Elavil) 50 mg at bedtime b. Ibuprofen (Advil) 800 mg 3 times daily c. Oxycodone (OxyContin) 80 mg twice daily d. Meperidine (Demerol) 25 mg every 4 hours

ANS: D Meperidine is contraindicated for chronic pain because it forms a metabolite that is neurotoxic and can cause seizures when used for prolonged periods. The ibuprofen, amitriptyline, and oxycodone are all appropriate medications for long-term pain management

A patient who is diagnosed with acquired immunodeficiency syndrome (AIDS) tells the nurse, "I feel obsessed with thoughts about dying. Do you think I am just being morbid?" Which response by the nurse is best? a. "Thinking about dying will not improve the course of AIDS." b. "It is important to focus on the good things about your life now." c. "Do you think that taking an antidepressant might be helpful to you?" d. "Can you tell me more about the kind of thoughts that you are having?"

ANS: D More assessment of the patient's psychosocial status is needed before taking any other action. The statements, "Thinking about dying will not improve the course of AIDS" and "It is important to focus on the good things in life" discourage the patient from sharing any further information with the nurse and decrease the nurse's ability to develop a trusting relationship with the patient. Although antidepressants may be helpful, the initial action should be further assessment of the patient's feelings

A 55-year-old woman in the sexually transmitted infection (STI) clinic tells the nurse that she is concerned she may have been exposed to gonorrhea by her partner. To determine whether the patient has gonorrhea, the nurse will plan to a. interview the patient about symptoms of gonorrhea. b. take a sample of cervical discharge for Gram staining. c. draw a blood specimen or rapid plasma reagin (RPR) testing. d. obtain secretions for a nucleic acid amplification test (NAAT).

ANS: D NAAT has a high sensitivity (similar to a culture) for gonorrhea. Because women have few symptoms of gonorrhea, asking the patient about symptoms may not be helpful in making a diagnosis. Smears and Gram staining are not useful because the female genitourinary tract has many normal flora that resemble N. gonorrhoeae. RPR testing is used to detect syphilis

A 19-year-old has been diagnosed with primary dysmenorrhea. How will the nurse suggest that the patient prevent discomfort? a. Avoid aerobic exercise during her menstrual period. b. Use cold packs on the abdomen and back for pain relief. c. Talk with her health care provider about beginning antidepressant therapy. d. Take nonsteroidal antiinflammatory drugs (NSAIDs) when her period starts.

ANS: D NSAIDs should be started as soon as the menstrual period begins and taken at regular intervals during the usual time frame in which pain occurs. Aerobic exercise may help reduce symptoms. Heat therapy, such as warm packs, is recommended for relief of pain. Antidepressant therapy is not a typical treatment for dysmenorrhea

19. A 28-year-old patient was recently diagnosed with polycystic ovary syndrome. It is most important for the nurse to teach the patient a. reasons for a total hysterectomy. b. how to decrease facial hair growth. c. ways to reduce the occurrence of acne. d. methods to maintain appropriate weight.

ANS: D Obesity exacerbates the problems associated with polycystic ovary syndrome, such as insulin resistance and type 2 diabetes. The nurse should also address the problems of acne and hirsutism, but these symptoms are lower priority because they do not have long-term health consequences. Although some patients do require total hysterectomy, this is usually performed only after other therapies have been unsuccessful

When monitoring for the effectiveness of treatment for a patient with a large anterior wall myocardial infarction, the most important information for the nurse to obtain is a. central venous pressure (CVP). b. systemic vascular resistance (SVR). c. pulmonary vascular resistance (PVR). d. pulmonary artery wedge pressure (PAWP).

ANS: D PAWP reflects left ventricular end diastolic pressure (or left ventricular preload) and is a sensitive indicator of cardiac function. Because the patient is high risk for left ventricular failure, the PAWP must be monitored. An increase will indicate left ventricular failure. The other values would also provide useful information, but the most definitive measurement of changes in cardiac function is the PAWP.

The nurse will plan to teach a 51-year-old man who is scheduled for an annual physical exam about a(n) a. increased risk for testicular cancer. b. possible changes in erectile function. c. normal decreases in testosterone level. d. prostate specific antigen (PSA) testing.

ANS: D PSA testing may be recommended annually for men, starting at age 50. There is no indication that the other patient teaching topics are appropriate for this patient

A patient who uses a fentanyl (Duragesic) patch for chronic cancer pain suddenly complains of rapid onset pain at a level 9 (0 to 10 scale) and requests "something for pain that will work now." How will the nurse document the type of pain reported by this patient? a. Somatic pain b. Referred pain c. Neuropathic pain d. Breakthrough pain

ANS: D Pain that occurs beyond the chronic pain already being treated by appropriate analgesics is termed breakthrough pain. Neuropathic pain is caused by damage to peripheral nerves or the central nervous system (CNS). Somatic pain is localized and arises from bone, joint, muscle, skin, or connective tissue. Referred pain is pain that is localized in uninjured tissue

The nurse assesses a patient who has just arrived in the postanesthesia recovery area (PACU) after a blepharoplasty. Which assessment data should be reported to the surgeon immediately? a. The patient complains of incisional pain. b. The patient's heart rate is 110 beats/minute. c. The patient is unable to detect when the eyelids are touched. d. The skin around the incision is pale and cold when palpated.

ANS: D Pale, cool skin indicates a possible decrease in circulation, so the surgeon should be notified immediately. The other assessment data indicate a need for ongoing assessment or nursing action. A heart rate of 110 beats/minute may be related to the stress associated with surgery. Assessment of other vital signs and continued monitoring are appropriate. Because local anesthesia would be used for the procedure, numbness of the incisional area is expected immediately after surgery. The nurse should monitor for return of feeling

A 76-year-old patient who has been diagnosed with stage 2 prostate cancer chooses the option of active surveillance. The nurse will plan to a. vaccinate the patient with sipuleucel-T ( Provenge). b. provide the patient with information about cryotherapy. c. teach the patient about placement of intraurethral stents. d. schedule the patient for annual prostate-specific antigen testing.

ANS: D Patients who opt for active surveillance need to have annual digital rectal exams and prostate-specific antigen testing. Vaccination with sipuleucel-T, cryotherapy, and stent placement are options for patients who choose to have active treatment for prostate cancer

When the nurse educator is evaluating the skills of a new registered nurse (RN) caring for patients experiencing shock, which action by the new RN indicates a need for more education? a. Placing the pulse oximeter on the ear for a patient with septic shock b. Keeping the head of the bed flat for a patient with hypovolemic shock c. Increasing the nitroprusside (Nipride) infusion rate for a patient with a high SVR d. Maintaining the room temperature at 66° to 68° F for a patient with neurogenic shock

ANS: D Patients with neurogenic shock may have poikilothermia. The room temperature should be kept warm to avoid hypothermia. The other actions by the new RN are appropriate

After a 26-year-old patient has been treated for pelvic inflammatory disease, the nurse will plan to teach about a. use of hormone therapy (HT). b. irregularities in the menstrual cycle. c. changes in secondary sex characteristics. d. possible difficulty with becoming pregnant.

ANS: D Pelvic inflammatory disease may cause scarring of the fallopian tubes and result in difficulty in fertilization or implantation of the fertilized egg. Because ovarian function is not affected, the patient will not require HT, have irregular menstrual cycles, or experience changes in secondary sex characteristics.

To prevent pregnancy in a patient who has been sexually assaulted, the nurse in the emergency department will plan to teach the patient about the use of a. mifepristone (RU-486). b. dilation and evacuation. c. methotrexate with misoprostol. d. levonorgestrel (Plan-B One-Step).

ANS: D Plan B One-Step reduces the risk of pregnancy when taken within 72 hours of intercourse. The other methods are used for therapeutic abortion, but not for pregnancy prevention after unprotected intercourse

A 54-year-old woman who recently reached menopause and has a family history of osteoporosis is diagnosed with osteopenia following densitometry testing. In teaching the woman about her osteoporosis, the nurse explains that a. estrogen replacement therapy must be started to prevent rapid progression to osteoporosis. b. continuous, low-dose corticosteroid treatment is effective in stopping the course of osteoporosis. c. with a family history of osteoporosis, there is no way to prevent or slow gradual bone resorption. d. calcium loss from bones can be slowed by increasing calcium intake and weight-bearing exercise.

ANS: D Progression of osteoporosis can be slowed by increasing calcium intake and weight-bearing exercise. Estrogen replacement therapy does help prevent osteoporosis, but it is not the only treatment and is not appropriate for some patients. Corticosteroid therapy increases the risk for osteoporosis

To evaluate the effectiveness of the pantoprazole (Protonix) ordered for a patient with systemic inflammatory response syndrome (SIRS), which assessment will the nurse perform? a. Auscultate bowel sounds. b. Palpate for abdominal pain. c. Ask the patient about nausea. d. Check stools for occult blood.

ANS: D Proton pump inhibitors are given to decrease the risk for stress ulcers in critically ill patients. The other assessments also will be done, but these will not help in determining the effectiveness of the pantoprazole administration

An older adult patient who has colorectal cancer is receiving IV fluids at 175 mL/hour in conjunction with the prescribed chemotherapy. Which finding by the nurse is most important to report to the health care provider? a. Patient complains of severe fatigue. b. Patient needs to void every hour during the day. c. Patient takes only 50% of meals and refuses snacks. d. Patient has audible crackles to the midline posterior chest.

ANS: D Rapid fluid infusions may cause heart failure, especially in older patients. The other findings are common in patients who have cancer and/or are receiving chemotherapy

Which menu choice by a patient with osteoporosis indicates that the nurse's teaching about appropriate diet has been effective? a. Pancakes with syrup and bacon b. Whole wheat toast and fresh fruit c. Egg-white omelet and a half grapefruit d. Oatmeal with skim milk and fruit yogurt

ANS: D Skim milk and yogurt are high in calcium. The other choices do not contain any high-calcium foods

After change-of-shift report, which patient should the nurse assess first? a. Patient with a Colles' fracture who has right wrist swelling and deformity b. Patient with a intracapsular left hip fracture whose leg is externally rotated c. Patient with a repaired mandibular fracture who is complaining of facial pain d. Patient with right femoral shaft fracture whose thigh is swollen and ecchymotic

ANS: D Swelling and bruising after a femoral shaft fracture suggest hemorrhage and risk for compartment syndrome. The nurse should assess the patient rapidly and then notify the health care provider. The other patients have symptoms that are typical for their injuries, but do not require immediate intervention

Which finding in a patient with a Colles' fracture of the left wrist is most important to communicate to the health care provider? a. Swelling is noted around the wrist. b. The patient is reporting severe pain. c. The wrist has a deformed appearance. d. Capillary refill to the fingers is prolonged.

ANS: D Swelling, pain, and deformity are common findings with a Colles' fracture. Prolonged capillary refill indicates decreased circulation and risk for ischemia. This is not an expected finding and should be immediately reported

Which nursing action will be most useful in assisting a college student to adhere to a newly prescribed antiretroviral therapy (ART) regimen? a. Give the patient detailed information about possible medication side effects. b. Remind the patient of the importance of taking the medications as scheduled. c. Encourage the patient to join a support group for students who are HIV positive. d. Check the patient's class schedule to help decide when the drugs should be taken.

ANS: D The best approach to improve adherence is to learn about important activities in the patient's life and adjust the ART around those activities. The other actions also are useful, but they will not improve adherence as much as individualizing the ART to the patient's schedule

A patient who has ovarian cancer is crying and tells the nurse, "My husband rarely visits. He just doesn't care." The husband indicates to the nurse that he never knows what to say to help his wife. Which nursing diagnosis is most appropriate for the nurse to add to the plan of care? a. Compromised family coping related to disruption in lifestyle b. Impaired home maintenance related to perceived role changes c. Risk for caregiver role strain related to burdens of caregiving responsibilities d. Dysfunctional family processes related to effect of illness on family members

ANS: D The data indicate that this diagnosis is most appropriate because poor communication among the family members is affecting family processes. No data suggest a change in lifestyle or its role as an etiology. The data do not support impairment in home maintenance or a burden caused by caregiving responsibilities

After change-of-shift report on a ventilator weaning unit, which patient should the nurse assess first? a. Patient who failed a spontaneous breathing trial and has been placed in a rest mode on the ventilator b. Patient who is intubated and has continuous partial pressure end-tidal CO2 (PETCO2) monitoring c. Patient with a central venous oxygen saturation (ScvO2) of 69% while on bilevel positive airway pressure (BiPAP) d. Patient who was successfully weaned and extubated 4 hours ago and now has no urine output for the last 6 hours

ANS: D The decreased urine output may indicate acute kidney injury or that the patient's cardiac output and perfusion of vital organs have decreased. Any of these causes would require rapid action. The data about the other patients indicate that their conditions are stable and do not require immediate assessment or changes in their care. Continuous PETCO2 monitoring is frequently used when patients are intubated. The rest mode should be used to allow patient recovery after a failed SBT, and an ScvO2 of 69% is within normal limits

A patient with an enlarging, irregular mole that is 7 mm in diameter is scheduled for outpatient treatment. The nurse should plan to prepare the patient for which procedure? a. Curettage b. Cryosurgery c. Punch biopsy d. Surgical excision

ANS: D The description of the mole is consistent with malignancy, so excision and biopsy are indicated. Curettage and cryosurgery are not used if malignancy is suspected. A punch biopsy would not be done for a lesion greater than 5 mm in diameter

After change-of-shift report, which patient should the progressive care nurse assess first? a. Patient who was extubated in the morning and has a temperature of 101.4° F (38.6° C) b. Patient with bilevel positive airway pressure (BiPAP) for sleep apnea whose respiratory rate is 16 c. Patient with arterial pressure monitoring who is 2 hours post-percutaneous coronary intervention who needs to void d. Patient who is receiving IV heparin for a venous thromboembolism and has a partial thromboplastin time (PTT) of 98 sec

ANS: D The findings for this patient indicate high risk for bleeding from an elevated (nontherapeutic) PTT. The nurse needs to adjust the rate of the infusion (dose) per the health care provider's parameters. The patient with BiPAP for sleep apnea has a normal respiratory rate. The patient recovering from the percutaneous coronary intervention will need to be assisted with voiding and this task could be delegated to unlicensed assistive personnel. The patient with a fever may be developing ventilator-associated pneumonia, but addressing the bleeding risk is a higher priority

A patient with chronic neck pain is seen in the pain clinic for follow-up. In order to evaluate whether the pain management is effective, which question is best for the nurse to ask? a. "Can you describe the quality of your pain?" b. "Has there been a change in the pain location?" c. "How would you rate your pain on a 0 to 10 scale?" d. "Does the pain keep you from doing things you enjoy?"

ANS: D The goal for the treatment of chronic pain usually is to enhance function and quality of life. The other questions are also appropriate to ask, but information about patient function is more useful in evaluating effectiveness

The nurse educator is evaluating the care that a new registered nurse (RN) provides to a patient receiving mechanical ventilation. Which action by the new RN indicates the need for more education? a. The RN increases the FIO2 to 100% before suctioning. b. The RN secures a bite block in place using adhesive tape. c. The RN asks for assistance to reposition the endotracheal tube. d. The RN positions the patient with the head of bed at 10 degrees.

ANS: D The head of the patient's bed should be positioned at 30 to 45 degrees to prevent ventilator-associated pneumonia. The other actions by the new RN are appropriate

A 68-year-old male patient tells the nurse that he is worried because he does not respond to sexual stimulation the same way he did when he was younger. The nurse's best response to the patient's concern is which of the following? a. "Interest in sex frequently decreases as men get older." b. "Many men need additional sexual stimulation with aging." c. "Erectile dysfunction is a common problem with older men." d. "Tell me more about how your sexual response has changed."

ANS: D The initial response by the nurse should be further assessment of the problem. The other statements by the nurse are accurate but may not respond to the patient's concerns

While waiting for cardiac transplantation, a patient with severe cardiomyopathy has a ventricular assist device (VAD) implanted. When planning care for this patient, the nurse should anticipate a. giving immunosuppressive medications. b. preparing the patient for a permanent VAD. c. teaching the patient the reason for complete bed rest. d. monitoring the surgical incision for signs of infection.

ANS: D The insertion site for the VAD provides a source for transmission of infection to the circulatory system and requires frequent monitoring. Patient's with VADs are able to have some mobility and may not be on bed rest. The VAD is a bridge to transplantation, not a permanent device. Immunosuppression is not necessary for nonbiologic devices like the VAD.

The nurse reviews the laboratory results of a patient who is receiving chemotherapy. Which laboratory result is most important to report to the health care provider? a. Hematocrit of 30% b. Platelets of 95,000/µL c. Hemoglobin of 10 g/L d. White blood cell (WBC) count of 2700/µL

ANS: D The low WBC count places the patient at risk for severe infection and is an indication that the chemotherapy dose may need to be lower or that WBC growth factors such as filgrastim (Neupogen) are needed. Although the other laboratory data indicate decreased levels, they do not indicate any immediate life-threatening adverse effects of the chemotherapy

A patient with human immunodeficiency virus (HIV) infection has developed Mycobacterium avium complex infection. Which outcome would be appropriate for the nurse to include in the plan of care? a. The patient will be free from injury. b. The patient will receive immunizations. c. The patient will have adequate oxygenation. d. The patient will maintain intact perineal skin.

ANS: D The major manifestation of M. avium infection is loose, watery stools, which would increase the risk for perineal skin breakdown. The other outcomes would be appropriate for other complications (pneumonia, dementia, influenza, etc.) associated with HIV infection

To maintain proper cuff pressure of an endotracheal tube (ET) when the patient is on mechanical ventilation, the nurse should a. inflate the cuff with a minimum of 10 mL of air. b. inflate the cuff until the pilot balloon is firm on palpation. c. inject air into the cuff until a manometer shows 15 mm Hg pressure. d. inject air into the cuff until a slight leak is heard only at peak inflation.

ANS: D The minimal occluding volume technique involves injecting air into the cuff until an air leak is present only at peak inflation. The volume to inflate the cuff varies with the ET and the patient's size. Cuff pressure should be maintained at 20 to 25 mm Hg. An accurate assessment of cuff pressure cannot be obtained by palpating the pilot balloon

The nurse responds to a ventilator alarm and finds the patient lying in bed holding the endotracheal tube (ET). Which action should the nurse take next? a. Activate the rapid response team. b. Provide reassurance to the patient. c. Call the health care provider to reinsert the tube. d. Manually ventilate the patient with 100% oxygen.

ANS: D The nurse should ensure maximal patient oxygenation by manually ventilating with a bag-valve-mask system. Offering reassurance to the patient, notifying the health care provider about the need to reinsert the tube, and activating the rapid response team are also appropriate after the nurse has stabilized the patient's oxygenation

When caring for a patient who has a radium implant for treatment of cancer of the cervix, the nurse will a. assist the patient to ambulate every 2 to 3 hours. b. use gloves and gown when changing the patient's bed. c. flush the toilet several times right after the patient voids. d. encourage the patient to discuss needs or concerns by telephone.

ANS: D The nurse should spend minimal time in the patient's room to avoid exposure to radiation. The patient and nurse can have longer conversations by telephone between the patient room and nursing station. To prevent displacement of the implant, absolute bed rest is required. Wearing of gloves and gown when changing linens, and flushing the toilet several times are not necessary because the isotope is confined to the implant

A patient who has severe refractory psoriasis on the face, neck, and extremities is socially withdrawn because of the appearance of the lesions. Which action should the nurse take first? a. Discuss the possibility of enrolling in a worker-retraining program. b. Encourage the patient to volunteer to work on community projects. c. Suggest that the patient use cosmetics to cover the psoriatic lesions. d. Ask the patient to describe the impact of psoriasis on quality of life.

ANS: D The nurse's initial actions should be to assess the impact of the disease on the patient's life and to allow the patient to verbalize feelings about the psoriasis. Depending on the assessment findings, other actions may be appropriate

A 48-year-old patient with a comminuted fracture of the left femur has Buck's traction in place while waiting for surgery. To assess for pressure areas on the patient's back and sacral area and to provide skin care, the nurse should a. loosen the traction and help the patient turn onto the unaffected side. b. place a pillow between the patient's legs and turn gently to each side. c. turn the patient partially to each side with the assistance of another nurse. d. have the patient lift the buttocks by bending and pushing with the right leg.

ANS: D The patient can lift the buttocks off the bed by using the left leg without changing the right-leg alignment. Turning the patient will tend to move the leg out of alignment. Disconnecting the traction will interrupt the weight needed to immobilize and align the fracture.

The nurse is caring for a patient who is to be discharged from the hospital 5 days after insertion of a femoral head prosthesis using a posterior approach. Which statement by the patient indicates a need for additional instruction? a. "I should not cross my legs while sitting." b. "I will use a toilet elevator on the toilet seat." c. "I will have someone else put on my shoes and socks." d. "I can sleep in any position that is comfortable for me."

ANS: D The patient needs to sleep in a position that prevents excessive internal rotation or flexion of the hip. The other patient statements indicate that the patient has understood the teaching

A patient with leukemia is considering whether to have hematopoietic stem cell transplantation (HSCT). The nurse will include which information in the patient's teaching plan? a. Transplant of the donated cells is painful because of the nerves in the tissue lining the bone. b. Donor bone marrow cells are transplanted through an incision into the sternum or hip bone. c. The transplant procedure takes place in a sterile operating room to minimize the risk for infection. d. Hospitalization will be required for several weeks after the stem cell transplant procedure is performed.

ANS: D The patient requires strict protective isolation to prevent infection for 2 to 4 weeks after HSCT while waiting for the transplanted marrow to start producing cells. The transplanted cells are infused through an IV line, so the transplant is not painful, nor is an operating room or incision required

A patient with atopic dermatitis has a new prescription for pimecrolimus (Elidel). After teaching the patient about the medication, which statement by the patient indicates that further teaching is needed? a. "After I apply the medication, I can go ahead and get dressed as usual." b. "I will need to minimize my time in the sun while I am using the Elidel." c. "I will rub the medication gently onto the skin every morning and night." d. "If the medication burns when I apply it, I will wipe it off and call the doctor."

ANS: D The patient should be taught that transient burning at the application site is an expected effect of pimecrolimus and that the medication should be left in place. The other statements by the patient are accurate and indicate that patient teaching has been effective

A patient receiving head and neck radiation for larynx cancer has ulcerations over the oral mucosa and tongue and thick, ropey saliva. Which instructions should the nurse give to this patient? a. Remove food debris from the teeth and oral mucosa with a stiff toothbrush. b. Use cotton-tipped applicators dipped in hydrogen peroxide to clean the teeth. c. Gargle and rinse the mouth several times a day with an antiseptic mouthwash. d. Rinse the mouth before and after each meal and at bedtime with a saline solution.

ANS: D The patient should rinse the mouth with a saline solution frequently. A soft toothbrush is used for oral care. Hydrogen peroxide may damage tissues. Antiseptic mouthwashes may irritate the oral mucosa and are not recommended

Four hours after mechanical ventilation is initiated for a patient with chronic obstructive pulmonary disease (COPD), the patient's arterial blood gas (ABG) results include a pH of 7.51, PaO2 of 82 mm Hg, PaCO2 of 26 mm Hg, and HCO3- of 23 mEq/L (23 mmol/L). The nurse will anticipate the need to a. increase the FIO2. b. increase the tidal volume. c. increase the respiratory rate. d. decrease the respiratory rate.

ANS: D The patient's PaCO2 and pH indicate respiratory alkalosis caused by too high a respiratory rate. The PaO2 is appropriate for a patient with COPD and increasing the respiratory rate and tidal volume would further lower the PaCO2.

The nurse in the women's health clinic has four patients who are waiting to be seen. Which patient should the nurse see first? a. 22-year-old with persistent red-brown vaginal drainage 3 days after having balloon thermotherapy b. 42-year-old with secondary amenorrhea who says that her last menstrual cycle was 3 months ago c. 35-year-old with heavy spotting after having a progestin-containing IUD (Mirena) inserted a month ago d. 19-year-old with menorrhagia who has been using superabsorbent tampons and has fever with weakness

ANS: D The patient's history and clinical manifestations suggest possible toxic shock syndrome, which will require rapid intervention. The symptoms for the other patients are consistent with their diagnoses and do not indicate life-threatening complications

The nurse is caring for a 78-year-old patient who was hospitalized 2 days earlier with community-acquired pneumonia. Which assessment information is most important to communicate to the health care provider? a. Scattered crackles bilaterally in the posterior lung bases. b. Persistent cough that is productive of blood-tinged sputum. c. Temperature of 101.5° F (38.6° C) after 2 days of IV antibiotic therapy. d. Decreased oxygen saturation to 90% with 100% O2 by non-rebreather mask.

ANS: D The patient's low SpO2 despite receiving a high fraction of inspired oxygen (FIO2) indicates the possibility of acute respiratory distress syndrome (ARDS). The patient's blood-tinged sputum and scattered crackles are not unusual in a patient with pneumonia, although they do require continued monitoring. The continued temperature elevation indicates a possible need to change antibiotics, but this is not as urgent a concern as the progression toward hypoxemia despite an increase in O2 flow rate

When assisting with the placement of a pulmonary artery (PA) catheter, the nurse notes that the catheter is correctly placed when the monitor shows a a. typical PA pressure waveform. b. tracing of the systemic arterial pressure. c. tracing of the systemic vascular resistance. d. typical PA wedge pressure (PAWP) tracing.

ANS: D The purpose of a PA line is to measure PAWP, so the catheter is floated through the pulmonary artery until the dilated balloon wedges in a distal branch of the pulmonary artery, and the PAWP readings are available. After insertion, the balloon is deflated and the PA waveform will be observed. Systemic arterial pressures are obtained using an arterial line and the systemic vascular resistance is a calculated value, not a waveform

Which information about continuous bladder irrigation will the nurse teach to a patient who is being admitted for a transurethral resection of the prostate (TURP)? a. Bladder irrigation decreases the risk of postoperative bleeding. b. Hydration and urine output are maintained by bladder irrigation. c. Antibiotics are infused continuously through the bladder irrigation. d. Bladder irrigation prevents obstruction of the catheter after surgery.

ANS: D The purpose of bladder irrigation is to remove clots from the bladder and to prevent obstruction of the catheter by clots. The irrigation does not decrease bleeding or improve hydration. Antibiotics are given by the IV route, not through the bladder irrigation.

Which information should the nurse include when teaching patients about decreasing the risk for sun damage to the skin? a. Use a sunscreen with an SPF of at least 8 to 10 for adequate protection. b. Water resistant sunscreens will provide good protection when swimming. c. Increase sun exposure by no more than 10 minutes a day to avoid skin damage. d. Try to stay out of the sun between the hours of 10 AM and 2 PM (regular time).

ANS: D The risk for skin damage from the sun is highest with exposure between 10 AM and 2 PM. No sunscreen is completely water resistant. Sunscreens classified as water resistant sunscreens still need to be reapplied after swimming. Sunscreen with an SPF of at least 15 is recommended for people at normal risk for skin cancer. Although gradually increasing sun exposure may decrease the risk for burning, the risk for skin cancer is not decreased

The nurse teaches a postmenopausal patient with stage III breast cancer about the expected outcomes of cancer treatment. Which patient statement indicates that the teaching has been effective? a. "After cancer has not recurred for 5 years, it is considered cured." b. "The cancer will be cured if the entire tumor is surgically removed." c. "Cancer is never considered cured, but the tumor can be controlled with surgery, chemotherapy, and radiation." d. "I will need to have follow-up examinations for many years after I have treatment before I can be considered cured."

ANS: D The risk of recurrence varies by the type of cancer. Some cancers are considered cured after a shorter time span or after surgery, but stage III breast cancer will require additional therapies and ongoing follow-up

A patient who is receiving sustained-release morphine sulfate (MS Contin) every 12 hours for chronic pain experiences level 9 (0 to 10 scale) breakthrough pain and anxiety. Which action by the nurse is best? a. Provide amitriptyline (Elavil) 10 mg orally. b. Administer lorazepam (Ativan) 1 mg orally. c. Offer ibuprofen (Motrin) 400 to 800 mg orally. d. Give immediate-release morphine 30 mg orally.

ANS: D The severe breakthrough pain indicates that the initial therapy should be a rapidly acting opioid, such as the immediate-release morphine. Lorazepam and amitriptyline may be appropriate to use as adjuvant therapy, but they are not likely to block severe breakthrough pain. Use of antianxiety agents for pain control is inappropriate because this patient's anxiety is caused by the pain

After receiving change-of-shift report on a medical unit, which patient should the nurse assess first? a. A patient with cystic fibrosis who has thick, green-colored sputum b. A patient with pneumonia who has crackles bilaterally in the lung bases c. A patient with emphysema who has an oxygen saturation of 90% to 92% d. A patient with septicemia who has intercostal and suprasternal retractions

ANS: D This patient's history of septicemia and labored breathing suggest the onset of ARDS, which will require rapid interventions such as administration of oxygen and use of positive pressure ventilation. The other patients should also be assessed as quickly as possible, but their assessment data are typical of their disease processes and do not suggest deterioration in their status

Which patient with pain should the nurse assess first? a. Patient with postoperative pain who received morphine sulfate IV 15 minutes ago b. Patient with neuropathic pain who has a dose of hydrocodone (Lortab) scheduled now c. Patient who received hydromorphone (Dilaudid) 1 hour ago and currently has a sedation scale of 2 d. Patient who returned from the postanesthesia care unit 2 hours ago and has a respiratory rate of 10

ANS: D This patient's respiratory rate indicates possible respiratory depression. The risk for oversedation is greatest in the first 4 hours after transfer from the postanesthesia care unit. Patients should be reassessed 30 minutes after receiving IV opioids for pain. A scheduled oral mediation does not need to be administered exactly at the scheduled time. A sedation scale of 2 indicates adequate pain control from hydromorphone

The health care provider prescribes topical 5-FU for a patient with actinic keratosis on the left cheek. The nurse should include which statement in the patient's instructions? a. "5-FU will shrink the lesion so that less scarring occurs once the lesion is excised." b. "You may develop nausea and anorexia, but good nutrition is important during treatment." c. "You will need to avoid crowds because of the risk for infection caused by chemotherapy." d. "Your cheek area will be painful and develop eroded areas that will take weeks to heal."

ANS: D Topical 5-FU causes an initial reaction of erythema, itching, and erosion that lasts 4 weeks after application of the medication is stopped. The medication is topical, so there are no systemic effects such as increased infection risk, anorexia, or nausea

Which action should the nurse take when caring for a patient who is receiving chemotherapy and complains of problems with concentration? a. Teach the patient to rest the brain by avoiding new activities. b. Teach that "chemo-brain" is a short-term effect of chemotherapy. c. Report patient symptoms immediately to the health care provider. d. Suggest use of a daily planner and encourage adequate rest and sleep.

ANS: D Use of tools to enhance memory and concentration such as a daily planner and adequate rest are helpful for patients who develop "chemo-brain" while receiving chemotherapy. Patients should be encouraged to exercise the brain through new activities. Chemo-brain may be short- or long-term. There is no urgent need to report common chemotherapy side effects to the provider

When caring for the patient with a pulmonary artery (PA) pressure catheter, the nurse observes that the PA waveform indicates that the catheter is in the wedged position. Which action should the nurse take next? a. Zero balance the transducer. b. Activate the fast flush system. c. Notify the health care provider. d. Deflate and reinflate the PA balloon.

ANS: D When the catheter is in the wedge position, blood flow past the catheter is obstructed, placing the patient at risk for pulmonary infarction. A health care provider or advanced practice nurse should be called to reposition the catheter. The other actions will not correct the wedging of the PA catheter


संबंधित स्टडी सेट्स

Finance Final, Fk Pirim (Exams1-3)

View Set

JavaScript Objects and Prototypes

View Set

OPMA 3306 Chapter 3 (Forecasting)

View Set